APUSH Exam Prac

Lakukan tugas rumah & ujian kamu dengan baik sekarang menggunakan Quizwiz!

The trend depicted in the table most directly contributed to which of the following developments in British North America? A Disagreement over the enforcement of mercantilist restrictions B Debates regarding the enactment of religious toleration in some colonies C Discussions about whether colonists believed themselves to be British subjects D Conflict over whether to allow slavery in the northern colonies

A Disagreement over the enforcement of mercantilist restrictions As the British North American colonies became increasingly reliant on imported goods from England, some colonists sought ways to work around mercantilist policies that required the colonies to trade only with Britain.

How were European economic systems in the American colonies in the 1500s and 1600s different from existing economic systems in Europe?

C Spanish colonists used enslaved Africans to work on plantations. During the colonization of the Americas, Spanish and later other European settlers relied on enslaved African labor, which was a significant departure from labor practices in European society.

"The existence of [colonial] subregions leads us to another question: whether the Middle Colonies in fact represented a coherent region at all. . . . In important respects, the Middle Colonies can be divided into separate societies focused around the cities of New York and Philadelphia. Thus the economies of [New York] and northern New Jersey were tied closely to that of New York City, while those of southern New Jersey, Pennsylvania, and northern Delaware were linked to Philadelphia. Those areas grew at very different rates, and they possessed quite distinct characteristics. . . . "Nonetheless, the Middle Colonies did share a number of things. One was their geography, a combination of climate and topography and setting, which determined some of the ways the land could be put to use, its accessibility to both intra-regional and international commerce, and its strategic importance in imperial competition. It was a region organized around extensive inland waterways, which gave merchants an almost unparalleled access to the American interior, building upon trade routes that pre-dated European settlement. . . . "Perhaps the most important argument for the coherence of the mid-Atlantic as a region is the extent to which those colonies shared a common history. . . . "The most often-noted characteristic of the region was the diversity of its peoples. . . . The society of the Middle Colonies surely was 'America's first plural society.' . . . There were two principal sources of the growing diversity of the European settlements. One was historical: New York, New Jersey, and Delaware were all conquered colonies, with Dutch, Swedish, Finnish, and many other populations already resident at the time of English conquest. The other was the consolidation that occurred as the colonies of six European nations along the Atlantic coast in the early seventeenth century were reduced to two by century's end, those of [Protestant] England and those of [Catholic] France. The result was that [diverse] European Protestants heading for the New World were concentrated within English colonies, a situation that virtually mandated some form of toleration. . . . Toleration and pluralism, it turns out, were not based solely on enlightened benevolence." Ned C. Landsman, historian, Crossroads of Empire: The Middle Colonies in British North America, published in 2010 Which of the following describes Landsman's overall argument in the excerpt? A The Middle Colonies differed from French colonies because they depended on Native American commerce. B The Middle Colonies were similar to each other because they developed plantation agriculture. C The Middle Colonies were more different from each other than the English colonies in other regions. D The Middle Colonies faced similar challenges in governing diverse colonists after they became English.

D The Middle Colonies faced similar challenges in governing diverse colonists after they became English. In the excerpt, Landsman makes the overall argument that the Middle Colonies shared a common history in governing the religiously and ethnically diverse colonial populations of the mid-Atlantic region.

Which of the following was a similarity between the encomienda system and slavery in the Spanish colonies?

The Spanish used both labor systems for plantation agriculture as well as for mining in their American colonies. Although the encomienda system was first designed to extract precious metals, it was also eventually used for agriculture, and African slaves were imported to work in mines as well as on plantations.

"With regard to the northwestern States, to which the ordinance of 1787 was applied—Ohio, Indiana, Illinois, and Michigan—no one now believes that any one of those States, if they thought proper to do it, has not just as much a right to introduce slavery within her borders as Virginia has a right to maintain the existence of slavery within hers. "Then, if in this struggle of power and empire between the two classes of states a decision of California has taken place adverse to the wishes of the southern States, it is a decision not made by the General [federal] Government; it is a decision respecting which they cannot complain to the General Government. It is a decision made by California herself, and which California had incontestably a right to make under the Constitution of the United States. . . . The question of slavery, either of its introduction or interdiction, is silent as respects the action of this [federal] Government; and if it has been decided, it has been by a different body—by a different power—by California herself, who had a right to make that decision." Senator Henry Clay, speech in the United States Senate, 1850 The excerpt best reflects which of the following historical situations? A Congressional leaders sought political compromise to resolve discord between the North and the South. B States in the Great Lakes region advocated to legalize slavery within their borders. C Senators appealed to the idea of American exceptionalism to encourage national unity. D The Supreme Court decision in the Dred Scott case reduced sectional conflict within the United States.

A Congressional leaders sought political compromise to resolve discord between the North and the South. The excerpt reflects growing discord between the North and the South over the expansion of slavery in new western territories. Congressional leaders such as Clay sought to defuse these sectional tensions through the Compromise of 1850, which among other provisions, admitted California as a free state, allowed popular sovereignty to determine the status of slavery in the Mexican Cession, and provided for a strengthened Fugitive Slave Act.

Which of the following best explains the cause of the emergence of new political parties in the early nineteenth century? A Continued debates over the proper role of the federal government B Growing agreement regarding the issue of slavery C Declining support for westward territorial expansion D Persistent concern over the influence of foreign powers

A Continued debates over the proper role of the federal government Debates over federal issues such as internal improvements, tariffs, and the abolition of slavery fueled the rise—and sometimes decline—of parties such as the Jacksonian Democrats, and the Whigs.

"The United States [under the Articles of Confederation] has an indefinite discretion to make [requests] for men and money; but they have no authority to raise either, by regulations extending to the individual citizens of America. The consequence of this is, that though in theory their resolutions concerning those objects are laws, constitutionally binding on the members of the Union, yet in practice they are mere recommendations which the States observe or disregard at their option. "There is nothing absurd or impracticable in the idea of a league or alliance between independent nations for certain defined purposes . . . depending for its execution on the good faith of the parties. . . . In the early part of the present century there was an [enthusiasm] in Europe for [leagues or alliances]. . . . They were scarcely formed before they were broken, giving an instructive but afflicting lesson to mankind, how little dependence is to be placed on treaties which have no other sanction than the obligations of good faith. . . . "There was a time when we were told that breaches, by the States, of the regulations of the [Confederation's] authority were not be expected. . . . "In our case, the [agreement] of thirteen distinct sovereign wills is requisite, under the Confederation, to the complete execution of every important measure that proceeds from the Union. . . . The measures of the Union have not been executed. . . . Each State, yielding to the persuasive voice of immediate interest or convenience, has successively withdrawn its support." Alexander Hamilton, The Federalist paper number 15, published in 1787 Which of the following was a piece of evidence Hamilton used to support his argument in the excerpt? A Earlier attempts to form associations of states in Europe had failed. B The United States had the authority to mandate funding under the Articles of Confederation. C Some people believed that the states would agree to follow the congressional directives. D States sometimes needed to form alliances to achieve shared goals.

A Earlier attempts to form associations of states in Europe had failed. In the second paragraph of the excerpt, Hamilton discussed earlier European experience with confederations of sovereign states as evidence that a stronger governing authority was necessary to prevent the political dissolution of the United States.

"It is not only important, but, in a degree necessary, that the people of this country, should have an American Dictionary of the English language; for, although the body of the language is the same as in England, . . . yet some differences must exist. Language is the expression of ideas; and if the people of one country cannot preserve an identity of ideas, they cannot retain an identity of language. . . . But the principal differences between the people of this country and of all others, arise from different forms of government, different laws, institutions and customs. Thus the . . . feudal system of England originated terms which formed . . . a necessary part of the language of that country; but, in the United States, many of these terms are no part of our present language,—and they cannot be, for the things which they express do not exist in this country. . . . The institutions in this country which are new and peculiar, give rise to new terms or to new applications of old terms, unknown to the people of England; which cannot be explained by them and which will not be inserted in their dictionaries, unless copied from ours. . . . No person in this country will be satisfied with the English definitions of the words congress, senate, and assembly, court, [etc.] for although these are words used in England, yet they are applied in this country to express ideas which they do not express in that country." Noah Webster, "Preface," An American Dictionary of the English Language, 1828 The national identity described in the excerpt most strongly reflects the influence of which of the following? A European precedents along with an American national culture B The English feudal system in which lords and landowners dominated vassals and farmers C Independence movements and revolutions in Europe and Latin America D Antislavery activism in the United States and Europe

A European precedents along with an American national culture Webster discusses the English origins of the language along with the unique American institutions that generated linguistic differences.

"The Declaration of Independence, drawn up by the Continental Congress, was actually a declaration by 'thirteen united States of America' proclaiming that as 'Free and Independent States they have full power to levy war, conclude peace, contract alliances, establish commerce, and to do all other acts and things which independent States may of right do.' And the Articles of Confederation, for all the powers it theoretically gave to the Congress, did not in fact alter this independence. . . . Congressional resolutions continued to be mere recommendations which the states were left to enforce. . . . The Confederation was intended to be, and remained, a Confederation of sovereign states." Gordon S. Wood, historian, The Creation of the American Republic, 1776-1787, published in 1969 Which of the following does the author use to support his argument about the power of the states under the Confederation? A Foundational political documents written during the American Revolution B Journals written by leaders of the Continental Army C Arguments eventually published as The Federalist papers D Treaties signed with Great Britain and France

A Foundational political documents written during the American Revolution The author cites language from the Declaration of Independence as evidence of the states being established as independent sovereign governments, a situation that was unaltered by the Articles of Confederation.

"The great increase of drunkenness, within the last half century, among the people of the United States, led a number of philanthropic individuals . . . to consult together, upon the duty of making more united, systematic, and extended efforts for the prevention of this evil. Its cause was at once seen to be, the use of intoxicating liquor; and its appropriate remedy, abstinence. It was also known, that the use of such liquor, as a beverage, is not only needless, but injurious to the health, the virtue, and the happiness of men. It was believed, that the facts which had been . . . collected would prove this . . . ; and that if the knowledge of them were universally disseminated it would, with the divine blessing, do much toward changing the habits of the nation. . . . [The American Temperance Society's] object is . . . the exertion of kind moral influence . . . to effect such a change of sentiment and practice, that drunkenness and all its evils will cease." Introduction to a book of reports from the American Temperance Society, 1835 The American Temperance Society used which of the following evidence in the excerpt to explain why it believed the temperance movement would be successful? A Its supporters held the moral and religious high ground. B Its supporters were willing to use violence to get people to stop drinking alcohol. C Its supporters were more likely to vote in upcoming elections. D Its supporters used new transportation routes to spread their message.

A Its supporters held the moral and religious high ground. The use of religious language in the excerpt reflects the belief among many reform movements during this period that their movement represented an effort to improve overall morality in the United States.

"Forces committed to restoring White supremacy launched a ruthless, bloody campaign of terror and intimidation against freedpeople and their White allies in the South. As young southern units of the Republican Party broke under those blows and the Republicans of the North retreated and grew more conservative, Reconstruction collapsed. With it went many . . . gains. A resurgent southern elite once again set about imposing White supremacy and tyrannical labor discipline while stripping freedpeople of many of their civic and political rights." Bruce Levine, historian, The Fall of the House of Dixie, 2013 "For many poor Whites throughout the South, Jim Crow laws alone could not ease their most persistent fear. In regions like northern Louisiana, with little but pine trees rising from its barren soil, White men found themselves competing with [formerly enslaved people], and during the dozen years of Reconstruction they had not known which race would prevail. "Such men had dropped away from the Ku Klux Klan after President Grant's crackdown, but their simmering resentments had grown. With control of the South passing again to the Democrats, powerless Whites were joining plantation owners to ensure that Black workers remained without their basic rights." A. J. Langguth, historian, After Lincoln, 2014 Which of the following claims is supported by the arguments made by both Levine and Langguth? A Local political tactics served to deny African Americans their rights. B White southerners accepted racial and political equality. C Republicans permanently changed the balance of political power in the South. D African Americans gained property rights while becoming self-sufficient.

A Local political tactics served to deny African Americans their rights. The authors agree that African Americans in the South were denied their newly won rights through intimidation, violence, and local political manipulation during the period.

"Jackson truly believed that, compared to his predecessors' combination of high-minded rhetoric, treachery, and abandonment, his Indian policy was 'just and humane.' . . . ". . . Jackson's paternalism was predicated on his assumption, then widely but not universally shared by white Americans, that all Indians . . . were [irrational] and inferior to all whites. His promises about voluntary and compensated relocation . . . were constantly undermined by delays and by sharp dealing by War Department negotiators—actions Jackson condoned. . . . Jackson tried to head off outright fraud, but the removal bill's allotment scheme invited an influx of outside speculators, who wound up buying between 80 and 90 percent of the land owned by Indians who wished to stay at a fraction of its actual worth. At no point did Jackson consider allowing even a small number of Georgia Cherokees who preferred to stay to do so in select enclaves, an option permitted to small numbers of Iroquois in upstate New York and Cherokees in western North Carolina. . . . Bereft of long-term planning and a full-scale federal commitment, the realities of Indian removal belied Jackson's rhetoric. Although the worst suffering was inflicted after he left office, Jackson cannot escape responsibility for setting in motion an insidious policy that uprooted tens of thousands of Choctaws and Creeks [from the Southeast] during his presidency." Sean Wilentz, historian, The Rise of American Democracy: Jefferson to Lincoln, published in 2005 Which of the following describes a context that most influenced the implementation of the government policy discussed in the excerpt? A Many Americans desired the United States to expand its western land claims. B Many Americans asserted the separation of public and private spheres. C Some Americans were influenced by European literary and cultural models. D Some Americans began to oppose the spread of slavery.

A Many Americans desired the United States to expand its western land claims. The context of the American desire to extend United States western land claims in the early nineteenth century most influenced the implementation of the Indian removal policies described in the excerpt.

"Your Memorialist . . . represents to your honorable body, that he has devoted much time and attention to the subject of a railroad from Lake Michigan through the Rocky Mountains to the Pacific Ocean, and that he finds such a route practicable, the results from which would be incalculable—far beyond the imagination of man to estimate. . . . "It would enable us, in the short space of eight days (and perhaps less) to concentrate all the forces of our vast country at any point from Maine to Oregon. . . . Such easy and rapid communication with such facilities for exchanging the different products of the different parts would bring all our immensely wide spread population together. . . . "[W]ith a railroad to the Pacific, and thence to China by steamers, can be performed in thirty days, being now a distance of nearly seventeen thousand miles. . . Then the drills and sheetings of Connecticut, Rhode Island, and Massachusetts, and other manufactures of the United States, may be transported to China in thirty days; and the teas and rich silks of China, in exchange, come back to New Orleans, to Charleston, to Washington, to Baltimore, to Philadelphia, New York, and to Boston, in thirty days more." Asa Whitney, merchant, "National Railroad, Connecting the Atlantic and Pacific Ocean," memorial to the Senate and House of Representatives of the United States, 1845 The excerpt best reflects which of the following developments? A Popular support for the idea of Manifest Destiny B The emergence of nativist political parties C The collapse of the Second Party System D The increase in sectional tensions before the Civil War

A Popular support for the idea of Manifest Destiny Whitney's boosterism in the excerpt for a transcontinental railroad to facilitate domestic and international trade was supported and furthered by popular belief in the mid-1840s in Manifest Destiny—the idea that the United States had a providential right to spread its land claims and influence from the Atlantic Ocean to the Pacific Ocean.

"In 1739 arrived among us from Ireland the Reverend Mr. Whitefield, who had made himself remarkable there as [a traveling] preacher. He was at first permitted to preach in some of our churches; but the clergy, taking a dislike to him, soon refused him their pulpits, and he was obliged to preach in the fields. The multitudes of all [members of different religious groups] that attended his sermons were enormous, and it was a matter of speculation to me . . . to observe the extraordinary influence of his oratory on his hearers, and how much they admired and respected him. . . . It was wonderful to see the change soon made in the manners of our inhabitants. From being thoughtless or indifferent about religion, it seemed as if all the world were growing religious, so that one could not walk through the town in an evening without hearing psalms sung in different families of every street. "And it being found inconvenient to assemble in the open air, subject to its [harsh conditions], the building of a house to meet in was no sooner proposed . . . and the work [of erecting the building] was carried on with such spirit as to be finished in a much shorter time than could have been expected. Both house and ground were vested in trustees, expressly for the use of any preacher of any religious persuasion who might desire to say something to the people at Philadelphia." Benjamin Franklin, from his autobiography, describing events in 1739 Which of the following developments most directly contributed to the events described in the excerpt? A Protestant evangelism came to the colonies from Great Britain and Europe. B The beliefs of Puritan leaders had an increased impact on colonial religious practices. C Colonists became less interested in religion as they became more involved with trans-Atlantic trade. D Local clergy members in the colonies routinely welcomed the preaching of diverse religious doctrines.

A Protestant evangelism came to the colonies from Great Britain and Europe. Reverend Whitefield came to the British North American colonies from England as a part of the exchange of religious and philosophical ideas that had begun to be shared across the Atlantic, sparking the First Great Awakening.

"What fault has there been on the part of the General Government of the United States? Why break up this Union? Will any gentleman be so kind as to particularize a single instance worthy of debate, in which the Federal Government has been derelict [negligent] in the discharge of its duty, or has failed to accomplish the purposes of its organization? . . . "I am not here . . . to defend the election of Abraham Lincoln. I believe that his election was virtually a fraud upon the people of the United States . . . nominated, as he was, by a sectional party, and upon a sectional platform, with no representation in the body which nominated him from the South; but he was nominated and elected according to the forms of law. . . . "Let us look . . . at the evils that must result from secession. The first, in my opinion, would be that our country would not only be divided into a Northern Confederacy and into Southern Confederacy, but, soon or later it would be divided into sundry [several] petty Confederacies. We would have a Central Confederacy, a Confederacy of the States of the Mississippi Valley, a Pacific Confederacy, a Western Confederacy, an Eastern Confederacy, a Northern and a Southern Confederacy. ". . . It is easy perhaps to break down this Government; but, sir, when we break it down it will not be so easy a matter to build it up. . . . Gentlemen cry out against the tyranny of their own government, and yet denounce [those opposed to secession] because we hesitate to allow ourselves to be thrust into the embraces of such a military dictatorship." Waitman T. Willey, addressing the Virginia State Secession Convention, March 4, 1861 Which of the following conclusions can best be reached based on the sentiments expressed in the excerpt? A Sectional tensions erupted because most Southerners did not support Abraham Lincoln. B Sectional divisions were showing signs of diminishing. C The Compromise of 1850 prevented the outbreak of long-term conflict. D The election of 1860 was a success for the idea that territories should vote whether or not to have slavery.

A Sectional tensions erupted because most Southerners did not support Abraham Lincoln. The excerpt shows that Virginia was considering following the other Southern states that seceded from the United States after the election of 1860, as Lincoln won the election without any electoral college votes from the South.

"I do not belong, said Mr. [Calhoun], to the school which holds that aggression is to be met by concession. . . . If we concede an inch, concession would follow concession—compromise would follow compromise, until our ranks would be so broken that effectual resistance would be impossible. . . . ". . . A large portion of the Northern States believed slavery to be a sin, and would believe it to be an obligation of conscience to abolish it if they should feel themselves in any degree responsible for its continuance. . . . ". . . Abolition and the Union cannot coexist. As the friend of the Union, I openly proclaim it—and the sooner it is known the better. The former may now be controlled, but in a short time it will be beyond the power of man to arrest the course of events. We of the South will not, cannot, surrender our institutions. To maintain the existing relations between the two races, inhabiting that section of the Union, is indispensable to the peace and happiness of both. . . . But let me not be understood as admitting, even by implication, that the existing relations between the two races in the slaveholding States is an evil—far otherwise; I hold it to be a good, as it has thus far proved itself to be to both, and will continue to prove so if not disturbed by the fell spirit of abolition." Source: South Carolina senator John C. Calhoun, speech in the United States Senate, 1837. Which of the following most contributed to slaveholders such as Calhoun arguing in the 1830s and 1840s that slavery should be viewed as part of the Southern way of life? A Slave rebellions in Haiti, South Carolina, and Virginia had made many leaders in the South fear that enslaved African Americans could harm them. B Abolitionists made up a majority of members of Congress in the 1840s, and Southerners feared that they would pass emancipation legislation. C Most Southern states had begun to allow slaves to learn to read and write, and Southerners wanted people in other states to know that the slave system was changing. D The sudden growth of Southern manufacturing had created a new demand for cotton, so men like Calhoun saw the preservation of slave labor as critical.

A Slave rebellions in Haiti, South Carolina, and Virginia had made many leaders in the South fear that enslaved African Americans could harm them. Many Southerners were terrified by slave rebellions, most recently Nat Turner's 1831 revolt in Virginia. They felt that preserving the slave system was the only way to protect themselves from slaves who would seek revenge if they were able to do so. As a consequence, they formulated justifications to defend the system of slavery.

"I do not belong, said Mr. [Calhoun], to the school which holds that aggression is to be met by concession. . . . If we concede an inch, concession would follow concession—compromise would follow compromise, until our ranks would be so broken that effectual resistance would be impossible. . . . ". . . A large portion of the Northern States believed slavery to be a sin, and would believe it to be an obligation of conscience to abolish it if they should feel themselves in any degree responsible for its continuance. . . . ". . . Abolition and the Union cannot coexist. As the friend of the Union, I openly proclaim it—and the sooner it is known the better. The former may now be controlled, but in a short time it will be beyond the power of man to arrest the course of events. We of the South will not, cannot, surrender our institutions. To maintain the existing relations between the two races, inhabiting that section of the Union, is indispensable to the peace and happiness of both. . . . But let me not be understood as admitting, even by implication, that the existing relations between the two races in the slaveholding States is an evil—far otherwise; I hold it to be a good, as it has thus far proved itself to be to both, and will continue to prove so if not disturbed by the fell spirit of abolition." Source: South Carolina senator John C. Calhoun, speech in the United States Senate, 1837. Which of the following resulted from arguments made by Southern politicians, such as the one in the excerpt, in the years prior to the Civil War? A Slaveholders became more insistent that maintaining the slave system was essential to protecting the South and its way of life. B Many people in the South who depended on the labor of enslaved people nevertheless became more willing to admit that slavery was a sin. C Abolitionists scheduled numerous speaking tours throughout the South to counter proslavery attitudes like that expressed by Calhoun. D Congress passed legislation guaranteeing slavery in the South to show that the slave system was not threatened.

A Slaveholders became more insistent that maintaining the slave system was essential to protecting the South and its way of life. In the decades preceding the Civil War, slaveholders began to echo Calhoun's speech and to describe slavery as a "positive good" in an effort to justify maintaining the slave economy.

"Antebellum planters . . . were very interested in the control of black movement. They were also keen to master their slaves' senses of pleasure. Seeking to contain [African Americans] even further than laws, curfews, bells, horns, and patrols already did, some planters used plantation [parties] as a paternalist mechanism of social control. Plantation parties, which carefully doled out joy on Saturday nights and holidays, were intended to seem benevolent and to inspire respect, gratitude, deference, and importantly, obedience. . . . The most important component of paternalistic plantation parties was the legitimating presence of the master. ". . . [Yet] again and again, slaves sought out illicit, secular gatherings of their own creation. They disregarded curfews and pass laws to escape to secret parties where . . . pleasures such as drinking, eating, dancing, and dressing up were the main amusements. . . . ". . . In the context of enslavement, such exhilarating pleasure . . . must be understood as important and meaningful enjoyment, as personal expression, and as oppositional." Stephanie M. H. Camp, historian, Closer to Freedom: Enslaved Women and Everyday Resistance in the Plantation South, 2004 Which of the following does the author use as evidence to support her argument that slaveholders were "keen to master their slaves' senses of pleasure"? A Slaveholders held parties to encourage the loyalty of the enslaved. B Slaveholders regularly listened to and sang slave spirituals. C Enslaved African Americans held their own illicit parties. D Enslaved African Americans regularly broke curfews and violated pass laws.

A Slaveholders held parties to encourage the loyalty of the enslaved. The author cites plantation parties held for enslaved people on special occasions as broader evidence of slaveholder's interest in using slaves' sense of pleasure as a further means of controlling them

"I am filled with deep emotion at finding myself standing here in the place . . . from which sprang the institutions under which we live. . . . I have never had a feeling politically that did not spring from the sentiments embodied in the Declaration of Independence. . . . It was not the mere matter of the separation of the colonies from the mother land; but something in that Declaration giving liberty, not alone to the people of this country, but hope to the world for all future time. It was that which gave promise that in due time the weights should be lifted from the shoulders of all men. . . . "Now, my friends, can this country be saved upon that basis? If it can, I will consider myself one of the happiest men in the world if I can help to save it. If it can't be saved upon that principle, it will be truly awful. "Now, in my view of the present aspect of affairs, there need be no bloodshed and war. . . . And I may say in advance, there will be no blood shed unless it be forced upon the Government. . . . "My friends, this is a wholly unprepared speech. I did not expect to be called upon to say a word when I came here. . . . I may, therefore, have said something indiscreet, but I have said nothing but what I am willing to live by, and, in the pleasure of Almighty God, die by." President-elect Abraham Lincoln, speaking at Independence Hall in Philadelphia, February 22, 1861 Evidence in the excerpt best corroborates which of the following? A Southern politicians would not abandon slavery, and they believed Lincoln was a threat to that system. B Sectional divisions were not as deep as many thought, and there was agreement about gradual abolition. C Political leadership in previous administrations had successfully limited sectional conflict. D Lincoln viewed foreign influences as serious threats to the stability of the United States.

A Southern politicians would not abandon slavery, and they believed Lincoln was a threat to that system. In the excerpt, Lincoln explains why it is necessary to be optimistic that conditions would improve. This is a reaction to the objectives that Southern states expressed in seceding from the Union, in which they declared that they wanted to preserve slavery at all costs.

The Kansas-Nebraska Act of 1854 was most similar in intent to which of the following earlier legislative initiatives? A The Missouri Compromise in 1820 B The forced removal of American Indians C The funding of internal improvements under the American System D The annexation of Texas in 1836

A The Missouri Compromise in 1820 Both the Missouri Compromise and the Kansas-Nebraska Act were attempts to find compromise between Northerners and Southerners over the issue of the expansion of slavery.

"The preservation of the states in a certain degree of agency is indispensable. It will produce that collision between the different authorities which should be wished for in order to check each other. To attempt to abolish the states altogether, would degrade the councils of our country, would be impracticable, would be ruinous. [John Dickinson] compared the proposed national system to the solar system, in which the states were the planets, and ought to be left to move freely in their proper orbits. . . . If the state governments were excluded from all agency in the national one, and all power drawn from the people at large, the consequence would be, that the national government would move in the same direction as the state governments now do, and would run into all the same mischiefs [troubles]." John Dickinson, delegate from Delaware, summary of a speech at the Constitutional Convention from the notes of James Madison, 1787 The principle of federalism embodied in the United States Constitution had most in common with which of the following earlier aspects of British colonial government? A The autonomy of colonial legislatures from Great Britain B The absence of colonial representation in Parliament C The enforcement of commercial regulations by British officials D The authority of the king over his colonial governors

A The autonomy of colonial legislatures from Great Britain During the period of British salutary neglect of its North American colonies before 1763, colonial legislatures enjoyed relative autonomy from direct control by central authorities in London. This was similar to the relationship the framers of the Constitution hoped to establish between the state governments and the federal government under the Constitution.

"It was not automatically apparent how any of the filibustering targets of the post-1848 period could 'fit' into an American republic, or even into an American empire. . . . While it seemed only logical to some to simply take all of Mexico as booty [spoils] of the war, cut Mexico up, and turn it into new territories and states, most Americans rejected this idea. They did so because central Mexico was densely populated. . . . Many Americans feared the result of the integration of Mexico's people into the United States. Critics also doubted whether Americans could be happy in the alien landscape of central and southern Mexico." Amy Greenberg, historian, Manifest Manhood and the Antebellum American Empire, 2005 "American settlers had eclipsed the Mexicans in Texas and, with ample aid from southern Whites, had rebelled and won their independence. . . . A small band of Americans, many of them merchants, lived in Mexican California when war broke out in 1846. This dispersion of hardy migrants inspired observers to insist that pioneers and not politicians won the West. . . . "Pioneers played a role in expansion, but the historical record points to politicians and propagandists as the primary agents of empire. Racial, economic, social, and political factors coalesced [combined] to make territorial and commercial expansion enticing to American leaders. . . . "Denying any parallels between earlier empires and their own, expansionists insisted that democracy and dominion were complementary, not contradictory. Since leaders intended to transform [territorial] cessions into states and their inhabitants (at least Whites) into citizens, they scoffed at misgivings about governing a vast domain." Thomas Hietala, historian, Manifest Design: American Exceptionalism and Empire, 2003 Both authors would most likely suggest that the historical situation described in the excerpts contributed to which of the following? A The continued alteration of Native American culture and society B The failure of Reconstruction policies to enforce constitutional amendments C The debate over the rights of states to nullify federal laws D The controversy over the Supreme Court decision in Dred Scott v. Sandford

A The continued alteration of Native American culture and society The United States victory and territorial gains as a result of the Mexican-American War increased conflict between Americans and Native Americans and put increased pressure on Native American culture and society by further taking their land.

"To the Commanders of armed vessels belonging to the United States: "WHEREAS it is declared by the act entitled 'An act for the protection of the commerce and seamen of the United States, against the Tripolitan cruisers,' That it shall be lawful fully to equip, officer, man, and employ such of the armed vessels of the United States, as may be judged requisite by the President of the United States, for protecting effectually the commerce and seamen thereof, on the Atlantic ocean, the Mediterranean and adjoining seas: and also, that it shall be lawful for the President of the United States to instruct the commanders of the respective public vessels, to subdue, seize, and make prize, of all vessels, goods, and effects, belonging to the Bey [Sultan] of Tripoli [in North Africa], or to his subjects. "THEREFORE, And in pursuance of the said statute, you are hereby authorized and directed to subdue, seize, and make prize, of all vessels, goods, and effects, belonging to the Bey of Tripoli, or to his subjects, and to bring or send the same into port, to be proceeded against and distributed according to law. "By command of the President of the United States of America." Thomas Jefferson, 1802 The excerpt could best be used by historians studying which of the following? A The creation of the Monroe Doctrine B The approval of the Louisiana Purchase C The abolition of the international slave trade D The passage of the Missouri Compromise

A The creation of the Monroe Doctrine The excerpt reflects the establishment of policies meant to ensure United States interests in the Mediterranean, much like the Monroe Doctrine claimed the right of the United States to protect its interests in the Western Hemisphere.

Which of the following best explains the depiction of George Washington in the painting? A The development of a sense of American identity among Patriots B The attempts of colonists to gain support for declaring independence C The desire of colonists to commemorate conquests over Native Americans D The tensions between northern and southern colonies during the Revolutionary War

A The development of a sense of American identity among Patriots As a hero celebrated for his leadership of the Continental Army, George Washington came to represent patriotism and American republican values for many Americans during the Revolutionary War and in the early United States. Images of Washington were reproduced throughout the country beginning in the later 1770s.

"Mr. President, it was solemnly asserted on this floor, some time ago, that all parties in the non-slaveholding States had come to a fixed and solemn determination upon two propositions. One was that there should be no further admission of any States into this Union which permitted, by their constitutions, the existence of slavery; and the other was that slavery shall not hereafter exist in any of the territories of the United States, the effect of which would be to give to the non-slaveholding States the monopoly of the public domain. . . . The subject has been agitated in the other House [of Congress], and they have sent up a bill 'prohibiting the extension of slavery . . . to any territory which may be acquired by the United States hereafter.' At the same time, two resolutions which have been moved to extend the compromise line from the Rocky Mountains to the Pacific, during the present session, have been rejected by a decided majority. "Sir, there is no mistaking the signs of the times; and it is high time that the Southern States—the slaveholding States—should inquire what is now their relative strength in this Union, and what it will be if this determination is carried into effect hereafter." John C. Calhoun, senator, speech in the United States Senate, 1847 The speech given by Calhoun relates to which of the following? A The effect of regional attitudes on federal policy making B The ways in which immigration changed American culture C The efforts by national leaders to expand the Pacific trade D The widespread support for the immediate end of slavery

A The effect of regional attitudes on federal policy making The excerpt depicts arguments about the expansion of slavery and how regional attitudes had a profound effect on the implementation of government policies and an eventual, but temporary, compromise in 1850.

The rise in manufacturing beginning in the early 1800s eventually resulted in which of the following by 1848? A The emergence of a larger middle class in the North B A decline in economic inequality in urban areas C An increased demand for agricultural workers in the Midwest D The improvement of working conditions in factories

A The emergence of a larger middle class in the North The growth in manufacturing in the northern and western portions of the United States in the early nineteenth century contributed to the emergence of a growing middle class, which made a living from trade and industry.

Which of the following developments was most directly connected to the collapse of the Whig Party in United States politics during the 1850s? A The escalation of tensions between proslavery and antislavery factions B Abraham Lincoln's debates with Stephen Douglas over popular sovereignty C The rejection of Manifest Destiny by a growing percentage of American people D Disagreement over the constitutionality of federal internal improvements funding

A The escalation of tensions between proslavery and antislavery factions The escalation of tensions between proslavery and antislavery factions within the United States created internal strife in the Whig Party, leading directly to its collapse as a political force in the 1850s. By the end of the decade, most party leaders and voters abandoned the Whig Party in favor of the Democrats or the newly founded Republican Party.

People who shared the views expressed in the image most likely supported which of the following? A The extension of political opportunities to formerly enslaved people B The rejection of suffrage rights for women C The expansion of the power of Southern Democrats D The enforcement of temperance laws in the North

A The extension of political opportunities to formerly enslaved people In addition to the voting rights extended to African American men following the Fifteenth Amendment, supporters of the argument presented in the image most likely would have favored allowing those men to exert their new political power.

"A bank of the United States is in many respects convenient for the Government and useful to the people. Entertaining this opinion, and deeply impressed with the belief that some of the powers and privileges possessed by the existing bank are unauthorized by the Constitution, subversive of the rights of the States, and dangerous to the liberties of the people, I felt it my duty at an early period of my Administration to call the attention of Congress to the practicability of organizing an institution combining all its advantages and obviating [removing] these objections. I sincerely regret that in the act before me I can perceive none of those modifications of the bank charter which are necessary, in my opinion, to make it compatible with justice, with sound policy, or with the Constitution of our country. . . . "Experience should teach us wisdom. Most of the difficulties our Government now encounters and most of the dangers which impend over our Union have sprung from an abandonment of the legitimate objects of Government by our national legislation. . . . Many of our rich men have not been content with equal protection and equal benefits, but have besought us to make them richer by act of Congress. By attempting to gratify their desires we have in the results of our legislation arrayed section against section, interest against interest, and man against man, in a fearful commotion which threatens to shake the foundations of our Union." President Andrew Jackson, Veto Message Regarding the Bank of the United States, 1832 The excerpt best reflects which of the following developments during the first half of the nineteenth century? A The formation of new political parties B The establishment of abolitionist groups C The increase of immigration from Europe D The emergence of regional cultures

A The formation of new political parties Debates over the role of the federal government, such as the constitutionality of the Bank of the United States, reflected one of the clear divisions between the emerging Democratic Party and the Whig Party in the 1820s and 1830s.

The expansion of suffrage to most adult White men in the early nineteenth century most directly resulted in which of the following? A The growth of new political parties B The abolition of slavery in the northern states C The signing of peace treaties with Native Americans D The employment of women in textile manufacturing

A The growth of new political parties As the status of being an adult White male, rather than property ownership, became the basis for voting rights in the early nineteenth century, civic participation increased and political parties more actively sought to speak for newly enfranchised voters.

Anti-immigrant nativism of the 1840s and 1850s had the most in common with which of the following earlier developments? A The passage of the Alien and Sedition Acts (1798), which limited rights for foreign-born residents B The conflict between Patriots and Loyalists during the American Revolution C The persecution of religious dissenters in the Massachusetts Bay Colony D The signing of the Treaty of Greenville (1794) that ended wars between the United States and Native Americans in the Northwest Territory

A The passage of the Alien and Sedition Acts (1798), which limited rights for foreign-born residents The passage of the Alien and Sedition Acts in 1798, which made it more difficult for immigrants to obtain naturalized citizenship and gave the president increased authority to imprison and deport dangerous noncitizens, shared the most continuity with the resurgence of anti-immigrant nativism against Irish, Germans, and Catholics in the 1840s and 1850s.

"The creation of a home market is not only necessary to procure for our agriculture a just reward of its labors, but it is indispensable to obtain a supply of our necessary wants. . . . Suppose no actual abandonment of farming, but, what is most likely, a gradual and imperceptible employment of population in the business of manufacturing, instead of being compelled to resort to agriculture. . . . Is any part of our common country likely to be injured by a transfer of the theatre of [manufacturing] for our own consumption from Europe to America? ". . . Suppose it were even true that Great Britain had abolished all restrictions upon trade, and allowed the freest introduction of the [products] of foreign labor, would that prove it unwise for us to adopt the protecting system? The object of protection is the establishment and perfection of the [manufacturing] arts. In England it, has accomplished its purpose, fulfilled its end. . . . The adoption of the restrictive system, on the part of the United States, by excluding the [products] of foreign labor, would extend the [purchasing] of American [products], unable, in the infancy and unprotected state of the arts, to sustain a competition with foreign fabrics. Let our arts breathe under the shade of protection; let them be perfected as they are in England, and [then] we shall be ready . . . to put aside protection, and enter upon the freest exchanges." Henry Clay, speaker of the House of Representatives, speech in Congress, 1824 The excerpt could best be used by historians studying which of the following in the early 1800s? A The political debates over economic development B The lives of women working in new factories C The effects of new technologies on commerce D The value of British-manufactured imports

A The political debates over economic development In the excerpt from Clay's speech, he sought to convince his congressional colleagues to support his plan to encourage United States manufacturing, which could best be used by historians studying political debates over economic development.

The trend in the population percent increase from 1790 to 1800 most likely indicates which of the following? A The rapid movement of migrants to newly settled states such as Kentucky and Tennessee B The increased arrival of indentured servants in established areas in Virginia and Maryland C The restriction of the international slave trade to North and South Carolina D The growth of new industrial cities in Georgia

A The rapid movement of migrants to newly settled states such as Kentucky and Tennessee The significant percent population increase in the southern interior from 1790 to 1800—199 percent in Kentucky, 194 percent in Tennessee—can best be explained by the movement of settlers into areas with limited prior White settlement.

"The American Republicans of the city and county of Philadelphia, who are determined to support the NATIVE [White, Protestant] AMERICANS in their Constitutional Rights of peaceably assembling to express their opinions on any question of Public Policy, and to SUSTAIN THEM AGAINST THE ASSAULTS OF ALIENS AND FOREIGNERS are requested to assemble on MONDAY AFTERNOON, May 6th, 1844 at 4 o'clock, at the corner of Master and Second street, Kensington [a section of Philadelphia], to express their indignation [anger] at the outrage on Friday evening last, which was perpetrated by the Irish Catholics." Text from a poster announcing a meeting of the American Republican Party, later renamed the American Party, Philadelphia, 1844 Which of the following historical situations can best be used to explain how the excerpt would have been interpreted at the time? A The rise in immigration to the United States B The decline of the international slave trade C The expansion of manufacturing in the South D The start of the women's rights movement

A The rise in immigration to the United States The rise in European immigration by Catholics from Ireland and Germany fueled nativist sentiments in the United States during the 1830s and 1840s and best explains how the excerpt would have been interpreted.

Which of the following best explains why some European colonists intermarried with Native Americans?

A To create economic and diplomatic relationships between Europeans and Native Americans French and Dutch colonists relied less on European inhabitants seizing land and more on intermarriage with Native Americans to develop trade alliances and diplomatic relationships that sought to strengthen mutual partnerships.

"I have already intimated [warned] to you the danger of parties in the State, with particular reference to the founding of them on geographical discriminations. Let me now take a more comprehensive view, and warn you in the most solemn manner against the baneful effects of the spirit of party generally. . . . The alternate domination of one faction over another, sharpened by the spirit of revenge, natural to party dissension . . . is itself a frightful despotism. . . . "The great rule of conduct for us in regard to foreign nations is in extending our commercial relations, to have with them as little political connection as possible. So far as we have already formed engagements, let them be fulfilled with perfect good faith. Here let us stop. . . . Taking care always to keep ourselves by suitable establishments on a respectable defensive posture, we may safely trust to temporary alliances for extraordinary emergencies. "Harmony, liberal intercourse with all nations, are recommended by policy, humanity, and interest. But even our commercial policy should hold an equal and impartial hand; neither seeking nor granting exclusive favors or preferences; consulting the natural course of things; diffusing and diversifying by gentle means the streams of commerce, but forcing nothing." George Washington, Farewell Address, 1796 Washington most likely wrote about political parties for which of the following purposes? A To warn the public that political parties result in national divisions B To explain how political parties are good for the economy C To assert that political parties only work in democratic republics D To argue that political parties encourage foreign interference

A To warn the public that political parties result in national divisions In the excerpt, Washington warned that the continual "domination of one faction over another" would eventually result in strong political divisions among Americans.

The expansion of suffrage to most adult White men by the 1820s and 1830s most directly contributed to the A emergence of political rallies and events to encourage people to vote for particular parties B increase in the autonomy of state legislatures C expansion of labor union activism that demanded better conditions for workers D establishment of representative democracy through the United States Constitution

A emergence of political rallies and events to encourage people to vote for particular parties As political participation came to be more often based on being an adult White male rather than property ownership, parties sought to promote voting by holding large-scale rallies and events and offering incentives including money and alcohol to support their sides.

"To the Commanders of armed vessels belonging to the United States: "WHEREAS it is declared by the act entitled 'An act for the protection of the commerce and seamen of the United States, against the Tripolitan cruisers,' That it shall be lawful fully to equip, officer, man, and employ such of the armed vessels of the United States, as may be judged requisite by the President of the United States, for protecting effectually the commerce and seamen thereof, on the Atlantic ocean, the Mediterranean and adjoining seas: and also, that it shall be lawful for the President of the United States to instruct the commanders of the respective public vessels, to subdue, seize, and make prize, of all vessels, goods, and effects, belonging to the Bey [Sultan] of Tripoli [in North Africa], or to his subjects. "THEREFORE, And in pursuance of the said statute, you are hereby authorized and directed to subdue, seize, and make prize, of all vessels, goods, and effects, belonging to the Bey of Tripoli, or to his subjects, and to bring or send the same into port, to be proceeded against and distributed according to law. "By command of the President of the United States of America." Thomas Jefferson, 1802 President Jefferson sought the protections described in the excerpt most likely for the purpose of A establishing trade routes B enlarging the size of the navy C supporting alliances with European nations D creating United States colonies

A establishing trade routes As the actions of the Barbary pirates threatened United States trade in the Mediterranean, the orders outlined in the excerpt indicate Jefferson's goal to secure and extend those trade routes.

The expansion of participatory democracy in the Jacksonian era most likely influenced the Second Great Awakening by A giving rise to individualistic beliefs B transforming gender roles in the family C increasing membership in the national political parties D generating opposition to the abolitionist movement

A giving rise to individualistic beliefs As more White male Americans saw their political participation expand, they began to believe that individuals should have a greater role in democracy and political decision making, which in turn influenced the emphasis in the Second Great Awakening on personal salvation.

"The committee of the president and directors of the Chesapeake and Delaware Canal Company [in Delaware] . . . beg leave respectfully to offer to the members of the Senate and House of Representatives, the following facts and observations relative to the said canal. . . . ". . . The island of Great Britain furnishes proof of the advantages of canals, beyond any other country. That nation has now become the maritime rival, and almost controller of every commercial people; her superiority has arisen from her unbounded commerce, and the vast wealth it has introduced, the basis of which wealth is her immense manufactures . . . : the foundation of these manufactures has again been formed by her internal improvements. . . . "The United States, both from their present political and natural situation, demand from their government every aid it can furnish. . . . Her rapid increase in prosperity, has already drawn upon her the envy, the jealousy, and the hostility of other nations, which alone can be counteracted by improving her internal strength, supplying her wants as far as possible by her own [products] and manufactures, and extending her agriculture so as to gain from its surplus the wealth of other nations." The Chesapeake and Delaware Canal Company, petition to the United States Congress, 1809 At the time the petition was produced, Congress most likely interpreted the petition's purpose as A requesting federal funding for transportation construction projects B rallying support to end commercial relations with Great Britain C encouraging White settlers to use canals to migrate to new states D promoting the development of agriculture at the expense of industry

A requesting federal funding for transportation construction projects As indicated by the request for aid from the government in the third paragraph, Congress at that time would have interpreted that petition as a request funding for internal improvements.

"National gratitude—national pride—every high and generous feeling that attaches us to the land of our birth, or that [elevates] our characters as individuals, ask[s] of us that we should foster the . . . literature of our country. . . . On the other hand, it is not necessary for these purposes—it is even detrimental to bestow on mediocrity the praise due to excellence, and still more so is the attempt to persuade ourselves and others into an admiration of the faults of [our writers]. . . . "It must however be allowed, that the poetry of the United States, though it has not reached that perfection to which some other countries have carried theirs, is yet even better than it could have been expected to produce, considering that our nation has scarcely seen two centuries since its founders erected their cabins on its soil. . . . "The fondness for literature is fast increasing in our country—and if this were not the case, the patrons of literature have multiplied, of course, and will continue to multiply with the mere growth of our population. The popular English works of the day are often reprinted in our country—they are dispersed all over the union. . . . What should hinder our native works, if equal in merit, from meeting an equally favorable reception?" William Cullen Bryant, book review in the North American Review, 1818 Which of the following can be concluded about the United States based on the author's descriptions in the excerpt? A Regional political interests dominated political debates. B A common national culture was developing. C Educational reforms contributed to increased literacy. D New transportation routes made shipping books easier.

B A common national culture was developing. The author describes the emergence of interest in literature across the United States as a whole, reflecting the development of a common national culture.

"Mississippi planter and agricultural reformer M. W. Phillips, a regular contributor to the American Cotton Planter, wrote about soil exhaustion and crop rotation, and extolled the virtues of manuring and self-provisioning. In one of his most widely reproduced articles, Phillips condemned planters before whom 'everything has to bend [and] give way to large crops of cotton.' . . . "Phillips imagined the cotton economy in terms of flows of energy, nutrients, and fertility, all of which he was convinced were being expended at an unsustainable rate. He used images of human, animal, and mineral depletion to represent an onrushing ecological catastrophe. But he did so within the incised [limited] terms allowed him by his culture—the culture of cotton. Phillips was arguing that the slaveholding South needed to slow the rate at which it was converting human beings into cotton plants." Walter Johnson, historian, River of Dark Dreams: Slavery and Empire in the Cotton Kingdom, 2013 In the first half of the 1800s, which of the following resulted from the debates about the cotton economy described in the excerpt? A Northerners began to frame antislavery arguments in ecological terms. B A distinct Southern economic and cultural identity emerged. C Large numbers of immigrants moved to Southern cities to pursue economic opportunities. D The federal government built an extensive network of roads, canals, and railroads to support cotton agriculture.

B A distinct Southern economic and cultural identity emerged. As the Southern economy became based around the production and export of a few agricultural staples like cotton, a distinctive southern culture and identity emerged in stark contrast to that of the North.

Changes in ideas about men's and women's gender roles in the family, resulting from the market revolution, most directly contributed to which of the following shifts in American social practices during the same period? A The rise of widespread support for women's right to vote in national elections B A new emphasis on the separation between the public and private spheres C Calls for mothers to guide their children's education in republican values and citizenship D The growth of a political culture blending European and uniquely American elements

B A new emphasis on the separation between the public and private spheres The growth of the separate spheres ideology, which made distinctions between the domestic or private sphere of the home associated with women and the public sphere of work and politics associated with men, became a major result of the market revolution.

"Mr. President, it was solemnly asserted on this floor, some time ago, that all parties in the non-slaveholding States had come to a fixed and solemn determination upon two propositions. One was that there should be no further admission of any States into this Union which permitted, by their constitutions, the existence of slavery; and the other was that slavery shall not hereafter exist in any of the territories of the United States, the effect of which would be to give to the non-slaveholding States the monopoly of the public domain. . . . The subject has been agitated in the other House [of Congress], and they have sent up a bill 'prohibiting the extension of slavery . . . to any territory which may be acquired by the United States hereafter.' At the same time, two resolutions which have been moved to extend the compromise line from the Rocky Mountains to the Pacific, during the present session, have been rejected by a decided majority. "Sir, there is no mistaking the signs of the times; and it is high time that the Southern States—the slaveholding States—should inquire what is now their relative strength in this Union, and what it will be if this determination is carried into effect hereafter." John C. Calhoun, senator, speech in the United States Senate, 1847 Which of the following can be concluded based on the situation in which Calhoun gave this speech? A The United States attempted to establish trade with western American Indian nations. B Americans debated how to integrate conquered territories into the United States. C Americans wanted to access natural resources in the western North America. D The United States sought to gain markets for its manufactured goods in East Asia.

B Americans debated how to integrate conquered territories into the United States. Calhoun's speech discussing the status of slavery in territories provides evidence to help conclude that Americans debated how the United States could integrate territories conquered from Mexico during the Mexican-American War.

Innovations in shipping and the growth of commercial networks were most directly related to which of the following other developments of the first half of the nineteenth century? A A decrease in the availability of jobs for recent immigrants B An increase in the number of Americans moving west of the Appalachian Mountains C The spread of industrialization to most cities in the South D An increase in the production in the home of goods used by families

B An increase in the number of Americans moving west of the Appalachian Mountains Innovations in shipping and transportation technology, including steamboats and canals, and the concurrent growth of national commercial networks allowed Americans to migrate and settle farther westward beyond the Appalachian Mountains.

"Yes: Mexico must be thoroughly chastised! . . . The news of yesterday [at the southern border] has added the last argument wanted to prove the necessity of an immediate Declaration of War by our government toward its southern neighbor. "We are justified in the face of the world, in having treated Mexico with more forbearance [tolerance] than we have ever yet treated an enemy. . . . We have . . . submitted thus far to a most offensive rejection of an Ambassador personifying the American nation, and waited for years without payment of the claims of our injured merchants. We have sought peace through every avenue, and shut our eyes to many things, which, had they come from England or France, the President would not have dared to pass over without stern and speedy resentment. We have dammed up our memory, of what had passed in the South [Texas] years ago—of devilish massacres of some of our bravest and noblest sons . . . in violation of all the rules of war. . . . "We think there can be no doubt of the truth of yesterday's news; and we are sure the people here, ten to one, are for prompt and hostilities. . . . Let our arms now be carried with a spirit which shall teach the world that, while we are not forward for a quarrel, America knows how to crush, as well as how to expand!" Walt Whitman, journalist and poet, editorial in the Brooklyn Eagle, 1846 "President [James K. Polk] in his message, as a pretext for sending our army to invade and conquer the country upon the Rio Grande, says: "Texas by its [legislative] act of December 19, 1836, had declared the [Rio Grande] to be the boundary of that [formerly independent] republic.' . . . The truth is that Texas had agreed upon the Nueces [River] as her boundary. . . . "If [Mexico] be ours, why does he seek to justify the taking possession of it by references to the fact that Mexico is indebted to some of our people? If it be not ours, and he has taken possession of it in order to compel Mexico to pay those debts, why not say so? The fact that Mexico has not paid the debts due to our citizens can have no legitimate connection with taking possession of [it as] our own soil. But [the president] was obviously conscious that this invasion of the Mexican territory could not be justified. . . . "When the Executive and Congress openly and avowedly took upon themselves the responsibility of extending and perpetuating slavery by the annexation of Texas, and by the total overthrow and subversion of the Constitution, . . . my confidence in the stability of our institutions was shaken, destroyed. . . . Our Union continues, but our Constitution is gone. . . . ". . . No man regards this war as just. We know, the country knows, and the civilized world are conscious, that it has resulted from a desire to extend and sustain an institution on which the curse of the Almighty most visibly rests." Joshua Giddings, congressman from Ohio, speech in the United States House of Representatives, 1846 Which of the following is a similarity between how Whitman and Giddings made their arguments? A Both used examples of Mexico's behavior to support their points. B Both sought to justify their positions to international observers. C Both drew on principles from the Constitution to make claims. D Both made religious appeals to prove their assertions.

B Both sought to justify their positions to international observers. Both Whitman and Giddings sought to justify their positions to international observers. In the second paragraph of the first excerpt, Whitman claimed that the United States was "justified in the face of the world," while in the last paragraph of the second excerpt, Giddings appealed to the "civilized world."

"I have already intimated [warned] to you the danger of parties in the State, with particular reference to the founding of them on geographical discriminations. Let me now take a more comprehensive view, and warn you in the most solemn manner against the baneful effects of the spirit of party generally. . . . The alternate domination of one faction over another, sharpened by the spirit of revenge, natural to party dissension . . . is itself a frightful despotism. . . . "The great rule of conduct for us in regard to foreign nations is in extending our commercial relations, to have with them as little political connection as possible. So far as we have already formed engagements, let them be fulfilled with perfect good faith. Here let us stop. . . . Taking care always to keep ourselves by suitable establishments on a respectable defensive posture, we may safely trust to temporary alliances for extraordinary emergencies. "Harmony, liberal intercourse with all nations, are recommended by policy, humanity, and interest. But even our commercial policy should hold an equal and impartial hand; neither seeking nor granting exclusive favors or preferences; consulting the natural course of things; diffusing and diversifying by gentle means the streams of commerce, but forcing nothing." George Washington, Farewell Address, 1796 Which of the following best explains why Washington warned against foreign alliances? A No nations attempted to sign commercial agreements with the United States. B Britain and France were at war with each other, and both threatened United States interests. C Federalists and Jeffersonian Republicans forced the president to agree to a policy of neutrality. D The power of European empires in the Americas had already begun to decline.

B Britain and France were at war with each other, and both threatened United States interests. Several nations had already established commercial agreements with the United States, including the Jay Treaty (1794) with Britain.

"Antebellum planters . . . were very interested in the control of black movement. They were also keen to master their slaves' senses of pleasure. Seeking to contain [African Americans] even further than laws, curfews, bells, horns, and patrols already did, some planters used plantation [parties] as a paternalist mechanism of social control. Plantation parties, which carefully doled out joy on Saturday nights and holidays, were intended to seem benevolent and to inspire respect, gratitude, deference, and importantly, obedience. . . . The most important component of paternalistic plantation parties was the legitimating presence of the master. ". . . [Yet] again and again, slaves sought out illicit, secular gatherings of their own creation. They disregarded curfews and pass laws to escape to secret parties where . . . pleasures such as drinking, eating, dancing, and dressing up were the main amusements. . . . ". . . In the context of enslavement, such exhilarating pleasure . . . must be understood as important and meaningful enjoyment, as personal expression, and as oppositional." Stephanie M. H. Camp, historian, Closer to Freedom: Enslaved Women and Everyday Resistance in the Plantation South, 2004 Which of the following could best be used as evidence to support the argument in the third paragraph of the excerpt that enslaved people engaged in oppositional activities? A Slaveholders allowed some enslaved African Americans to work unsupervised. B Enslaved African Americans routinely caused tools to break or worked more slowly as means of resistance. C Abolitionists criticized slaveholders for separating enslaved children from their parents. D Enslaved African Americans assigned to plantation homes had easier working conditions than those who worked in the fields.

B Enslaved African Americans routinely caused tools to break or worked more slowly as means of resistance. The breaking of tools was another documented example of slave resistance like the illicit parties described in the excerpt.

"Let us, then, with courage and confidence, pursue our own Federal and [Democratic-] Republican principles, our attachment to union and representative government. Kindly separated by nature and a wide ocean from the exterminating havoc of one quarter of the globe; too high-minded to endure the degradations of the others; possessing a chosen country, with room enough for our descendants . . . ; entertaining a due sense of our equal right to the use of our own faculties, to the acquisitions of our own industry, to honor and confidence from our fellow citizens, resulting not from birth, but from our actions and their sense of them; enlightened by a benign religion . . . —with all these blessings, what more is necessary to make us a happy and a prosperous people? Still one thing more, fellow citizens—a wise and frugal Government, which shall restrain men from injuring one another, shall leave them otherwise free to regulate their own pursuits of industry and improvement, and shall not take from the mouth of labor the bread it has earned. This is the sum of good government; and this is necessary to close the circle of our felicities." President Thomas Jefferson, first inaugural address, 1801 Which of the following best describes Jefferson's point of view about government as expressed in the excerpt? A The federal government should financially support internal improvements. B Government should limit interference in the lives of its citizens. C Governments should provide economic assistance to all citizens. D State governments should have more authority than the federal government.

B Government should limit interference in the lives of its citizens. Jefferson's views of government saw limited involvement with the lives of United States citizens. He argued in the excerpt that "a wise and frugal Government . . . shall leave [its people] otherwise free to regulate their own pursuits of industry and improvement."

"Whether you are or are not, entitled to all the rights of citizenship in this country has long been a matter of dispute to your prejudice. By enlisting in the service of your country at this trial hour, and upholding the National Flag, you stop the mouths of [cynics] and win applause even from the iron lips of ingratitude. Enlist and you make this your country in common with all other men born in the country or out of it. . . . He who fights the battles of America may claim America as his country—and have that claim respected. Thus in defending your country now against rebels and traitors you are defending your own liberty, honor, manhood and self-respect. . . . . . . [H]istory shall record the names of heroes and martyrs who bravely answered the call of patriotism and Liberty—against traitors, thieves and assassins—let it not be said that in the long list of glory, composed of men of all nations—there appears the name of no colored man." Frederick Douglass, excerpt from an editorial, April 1863 Douglass' rhetoric in the excerpt was most likely interpreted as promoting which of the following? A The need for more soldiers in the Union Army B His advocacy for African American equal rights C His support for Abraham Lincoln's reelection in 1864 D Criticism of the limits of the Emancipation Proclamation

B His advocacy for African American equal rights The excerpt explains why Douglass believes African Americans should join the Union Army and fight for the United States. His main reason for advocating this is so that African Americans can more easily legitimize their claims to the same constitutional rights as White citizens.

The development of the Second Great Awakening can best be linked to which of the following historical situations? A The market revolution led to a larger number of Americans working for wages. B Increased geographical mobility aided travel to new regions and the sharing of ideas. C Ideals of Romanticism caused more people to question the principles of the nation's founders. D The growing abolitionist movement was predominately influenced by northern Protestant Christians.

B Increased geographical mobility aided travel to new regions and the sharing of ideas. One of the main contributors to the success of the Second Great Awakening was the ability of Americans to reach different regions and to share ideas, thanks to innovations in transportation such as railroads, canals, and more roads.

"It was not automatically apparent how any of the filibustering targets of the post-1848 period could 'fit' into an American republic, or even into an American empire. . . . While it seemed only logical to some to simply take all of Mexico as booty [spoils] of the war, cut Mexico up, and turn it into new territories and states, most Americans rejected this idea. They did so because central Mexico was densely populated. . . . Many Americans feared the result of the integration of Mexico's people into the United States. Critics also doubted whether Americans could be happy in the alien landscape of central and southern Mexico." Amy Greenberg, historian, Manifest Manhood and the Antebellum American Empire, 2005 "American settlers had eclipsed the Mexicans in Texas and, with ample aid from southern Whites, had rebelled and won their independence. . . . A small band of Americans, many of them merchants, lived in Mexican California when war broke out in 1846. This dispersion of hardy migrants inspired observers to insist that pioneers and not politicians won the West. . . . "Pioneers played a role in expansion, but the historical record points to politicians and propagandists as the primary agents of empire. Racial, economic, social, and political factors coalesced [combined] to make territorial and commercial expansion enticing to American leaders. . . . "Denying any parallels between earlier empires and their own, expansionists insisted that democracy and dominion were complementary, not contradictory. Since leaders intended to transform [territorial] cessions into states and their inhabitants (at least Whites) into citizens, they scoffed at misgivings about governing a vast domain." Thomas Hietala, historian, Manifest Design: American Exceptionalism and Empire, 2003 Which of the following arguments about the Mexican-American War do the excerpts best support? A It resulted in the first efforts at western expansion. B It generated debates over citizenship. C It ended sectional tensions between the North and South. D It contributed to the elimination of the domestic slave trade.

B It generated debates over citizenship. As a result of the war, the United States added to its territorial holdings, raising questions about the status of enslaved people, Native Americans, and Mexicans in the newly acquired land.

"The American Republicans of the city and county of Philadelphia, who are determined to support the NATIVE [White, Protestant] AMERICANS in their Constitutional Rights of peaceably assembling to express their opinions on any question of Public Policy, and to SUSTAIN THEM AGAINST THE ASSAULTS OF ALIENS AND FOREIGNERS are requested to assemble on MONDAY AFTERNOON, May 6th, 1844 at 4 o'clock, at the corner of Master and Second street, Kensington [a section of Philadelphia], to express their indignation [anger] at the outrage on Friday evening last, which was perpetrated by the Irish Catholics." Text from a poster announcing a meeting of the American Republican Party, later renamed the American Party, Philadelphia, 1844 The language in the excerpt was most likely interpreted as promoting which of the following? A States' rights B Nativist sentiment C Religious pluralism D Abolitionist activism

B Nativist sentiment The excerpt depicts strong language vilifying Irish Catholics, who along with German Catholics migrated to the United States in substantial numbers during the 1830s and 1840s.

"Let us, then, with courage and confidence, pursue our own Federal and [Democratic-] Republican principles, our attachment to union and representative government. Kindly separated by nature and a wide ocean from the exterminating havoc of one quarter of the globe; too high-minded to endure the degradations of the others; possessing a chosen country, with room enough for our descendants . . . ; entertaining a due sense of our equal right to the use of our own faculties, to the acquisitions of our own industry, to honor and confidence from our fellow citizens, resulting not from birth, but from our actions and their sense of them; enlightened by a benign religion . . . —with all these blessings, what more is necessary to make us a happy and a prosperous people? Still one thing more, fellow citizens—a wise and frugal Government, which shall restrain men from injuring one another, shall leave them otherwise free to regulate their own pursuits of industry and improvement, and shall not take from the mouth of labor the bread it has earned. This is the sum of good government; and this is necessary to close the circle of our felicities." President Thomas Jefferson, first inaugural address, 1801 Which of the following best describes the context from which the ideas expressed in the excerpt emerged? A Popular opinion supported intervention in Europe against France. B Political leaders sought to encourage domestic economic development. C Voters pressured state governments to drop property restrictions on voting. D Religious revivals encouraged the widespread development of reform movements.

B Political leaders sought to encourage domestic economic development. Jefferson's discussion in his inaugural address of the advantages of the United States in the early 1800s occurred in the context of the desire of many political leaders to encourage expanded agriculture and increased trade.

"Jackson truly believed that, compared to his predecessors' combination of high-minded rhetoric, treachery, and abandonment, his Indian policy was 'just and humane.' . . . ". . . Jackson's paternalism was predicated on his assumption, then widely but not universally shared by white Americans, that all Indians . . . were [irrational] and inferior to all whites. His promises about voluntary and compensated relocation . . . were constantly undermined by delays and by sharp dealing by War Department negotiators—actions Jackson condoned. . . . Jackson tried to head off outright fraud, but the removal bill's allotment scheme invited an influx of outside speculators, who wound up buying between 80 and 90 percent of the land owned by Indians who wished to stay at a fraction of its actual worth. At no point did Jackson consider allowing even a small number of Georgia Cherokees who preferred to stay to do so in select enclaves, an option permitted to small numbers of Iroquois in upstate New York and Cherokees in western North Carolina. . . . Bereft of long-term planning and a full-scale federal commitment, the realities of Indian removal belied Jackson's rhetoric. Although the worst suffering was inflicted after he left office, Jackson cannot escape responsibility for setting in motion an insidious policy that uprooted tens of thousands of Choctaws and Creeks [from the Southeast] during his presidency." Sean Wilentz, historian, The Rise of American Democracy: Jefferson to Lincoln, published in 2005 Which of the following pieces of evidence would best refute Jackson's claim about his predecessors' policies toward American Indians, as described in the first paragraph of the excerpt? A President James Madison used the federal army to defeat an American Indian confederacy in the Northwest Territory. B President George Washington enforced treaties guaranteeing American Indians in New York rights to their land. C President James Monroe forced American Indians in Florida to move to a reservation after the First Seminole War. D President Thomas Jefferson suggested purchasing territory from indebted American Indian groups.

B President George Washington enforced treaties guaranteeing American Indians in New York rights to their land. Evidence that George Washington enforced treaties to guarantee American Indians rights to their land in New York would refute Jackson's claim that earlier presidents had mistreated American Indians.

"The committee of the president and directors of the Chesapeake and Delaware Canal Company [in Delaware] . . . beg leave respectfully to offer to the members of the Senate and House of Representatives, the following facts and observations relative to the said canal. . . . ". . . The island of Great Britain furnishes proof of the advantages of canals, beyond any other country. That nation has now become the maritime rival, and almost controller of every commercial people; her superiority has arisen from her unbounded commerce, and the vast wealth it has introduced, the basis of which wealth is her immense manufactures . . . : the foundation of these manufactures has again been formed by her internal improvements. . . . "The United States, both from their present political and natural situation, demand from their government every aid it can furnish. . . . Her rapid increase in prosperity, has already drawn upon her the envy, the jealousy, and the hostility of other nations, which alone can be counteracted by improving her internal strength, supplying her wants as far as possible by her own [products] and manufactures, and extending her agriculture so as to gain from its surplus the wealth of other nations." The Chesapeake and Delaware Canal Company, petition to the United States Congress, 1809 The claims in the excerpt were most likely interpreted as opposing which of the following existing federal government policies at the time? A Financing a national banking system B Promoting economic development through foreign trade C Acquiring western Native American land D Levying tariffs on imported manufactured goods

B Promoting economic development through foreign trade The claims in the excerpt about developing manufacturing and the domestic economy would have been interpreted as opposing government policies focused on encouraging foreign trade.

"Brother, listen to what we say. There was a time when our forefathers owned this great [land]. . . . Your forefathers crossed the great water and landed upon this [land]. Their numbers were small. They found friends, not enemies. They told us they had fled from their own country for fear of wicked men, and had come here to enjoy their religion. They asked for a small seat. We took pity on them, we granted their request, and they sat down amongst us. We gave them corn and meat; they gave us poison in return. ". . . Our seats were once large and yours were small. You have now become a great people, and we have scarcely a place left to spread our blankets. You have got our country, but are not satisfied; you want to force your religion upon us. . . . ". . . The Great Spirit has made us all, but he has made a great difference between his white and red children. . . . Since he has made so great a difference between us in other things, why may we not conclude that he has given us a different religion according to our understanding? The Great Spirit does right. He knows what is best for his children; we are satisfied." Red Jacket, Iroquois American Indian chief in New York, speech to a missionary from Massachusetts and a United States diplomat, 1805 Which of the following best explains how the purpose of the speech in the excerpt was interpreted by federal officials? A Red Jacket wanted to increase Iroquois commerce with the United States. B Red Jacket sought to protect Iroquois independence from the United States. C Red Jacket desired to abandon traditional Iroquois religious practices for Christianity. D Red Jacket wished to help United States migrants form new settlements on Iroquois land.

B Red Jacket sought to protect Iroquois independence from the United States. Red Jacket's expression in the excerpt of his desire to reject Christianization by United States officials would have been interpreted as an attempt to maintain Iroquois independence from the United States. Engaging in missionary efforts was one strategy pursued by the United States in the nineteenth century to reduce the autonomy of American Indian nations.

"The petition of a great number of Blacks detained in a state of slavery in the bowels of a free and Christian country humbly showeth that your petitioners apprehend that they have in common with all other men a natural and inalienable right to that freedom which the Great Parent of the Universe hath bestowed equally on all mankind. . . . They were unjustly dragged by the hand of cruel power . . . from a populous, pleasant, and plentiful country and in violation of laws of nature and of nations. . . . ". . . Your petitioners . . . cannot but express their astonishment that it has never been considered that every principle from which America has acted in the course of their unhappy difficulties with Great Britain pleads stronger than a thousand arguments in favor of your petitioners. They therefore humble beseech your honors to give this petition its due weight and consideration and cause an act of the legislature to be passed whereby they may be restored to the enjoyments of that which is the natural right of all men—and their children who were born in this land of liberty may not be held as slaves after they arrive at the age of twenty one years." Petition to the Massachusetts state legislature, 1777 Which of the following describes an overall argument of the excerpt? A Slavery is not economically important for the northern colonies. B Slavery is contrary to the ideals of the American Revolution. C Enslaved Africans should be rewarded for their loyalty to Great Britain. D Enslaved Africans should be returned to Africa.

B Slavery is contrary to the ideals of the American Revolution. In the excerpt, the African American petitioners claim in the first paragraph that the ideals of the American Revolution apply to them as well as to White men, and in the second paragraph, they claim that their desire for freedom is similar to that of the American revolutionaries fighting Great Britain.

"Mississippi planter and agricultural reformer M. W. Phillips, a regular contributor to the American Cotton Planter, wrote about soil exhaustion and crop rotation, and extolled the virtues of manuring and self-provisioning. In one of his most widely reproduced articles, Phillips condemned planters before whom 'everything has to bend [and] give way to large crops of cotton.' . . . "Phillips imagined the cotton economy in terms of flows of energy, nutrients, and fertility, all of which he was convinced were being expended at an unsustainable rate. He used images of human, animal, and mineral depletion to represent an onrushing ecological catastrophe. But he did so within the incised [limited] terms allowed him by his culture—the culture of cotton. Phillips was arguing that the slaveholding South needed to slow the rate at which it was converting human beings into cotton plants." Walter Johnson, historian, River of Dark Dreams: Slavery and Empire in the Cotton Kingdom, 2013 Which of the following resulted from the mass production of cotton described in the excerpt? A Southern planters eventually sought to diversify the crops they grew. B Some southerners relocated their plantations to the west of the Appalachian Mountains. C A cotton gin was developed that processed raw cotton more quickly. D Southerners supported protective tariffs to stimulate the United States economy.

B Some southerners relocated their plantations to the west of the Appalachian Mountains. Overproduction of cotton depleted the soil in eastern states of the South, leading to many southerners moving west and allowing the institution of slavery to grow.

The data in the first table most directly indicate which of the following about the professions of soldiers in the Civil War? A The Confederacy enlisted more factory workers to fill its armies than did the Union. B The Confederacy relied more heavily on agricultural workers to fill its armies than did the Union. C The majority of soldiers in both Confederate and Union forces were sharecroppers. D The Confederacy enlisted more skilled workers than did the Union.

B The Confederacy relied more heavily on agricultural workers to fill its armies than did the Union. According to the table, over 60 percent of the Confederacy's soldiers were farmworkers, whereas only 47.5 percent the Union's soldiers were.

Which of the following differences between the North and the South during the Civil War is depicted in the graph? A The South engaged in more international trade than the North. B The South relied more on plantation agriculture than the North. C The North had less developed transportation infrastructure than the South. D The North had fewer people available for labor and the military than the South.

B The South relied more on plantation agriculture than the North. The graph indicates that the South had more large farms (84 percent) than the North (16 percent) which was because it relied more on plantation agriculture.

"The United States [under the Articles of Confederation] has an indefinite discretion to make [requests] for men and money; but they have no authority to raise either, by regulations extending to the individual citizens of America. The consequence of this is, that though in theory their resolutions concerning those objects are laws, constitutionally binding on the members of the Union, yet in practice they are mere recommendations which the States observe or disregard at their option. "There is nothing absurd or impracticable in the idea of a league or alliance between independent nations for certain defined purposes . . . depending for its execution on the good faith of the parties. . . . In the early part of the present century there was an [enthusiasm] in Europe for [leagues or alliances]. . . . They were scarcely formed before they were broken, giving an instructive but afflicting lesson to mankind, how little dependence is to be placed on treaties which have no other sanction than the obligations of good faith. . . . "There was a time when we were told that breaches, by the States, of the regulations of the [Confederation's] authority were not be expected. . . . "In our case, the [agreement] of thirteen distinct sovereign wills is requisite, under the Confederation, to the complete execution of every important measure that proceeds from the Union. . . . The measures of the Union have not been executed. . . . Each State, yielding to the persuasive voice of immediate interest or convenience, has successively withdrawn its support." Alexander Hamilton, The Federalist paper number 15, published in 1787 Which of the following claims did Hamilton make in the excerpt about the powers of the United States under the Articles of Confederation? A The United States could not engage in diplomacy with foreign countries. B The United States was not empowered to raise sufficient money for the government. C The United States could act without the unanimous consent of the states. D The United States was able to raise military forces sufficient to defend the country.

B The United States was not empowered to raise sufficient money for the government. In the first paragraph of the excerpt, Hamilton claimed that while the United States under the Articles of Confederation could request money from the states, the states were not obliged to fulfill such requests to fund the national government.

"No roads marked the way to the traveler in California then: but, guided by the sun and well-known mountain peaks, we proceeded on our journey. . . . Some forty or fifty men were at work with the cradle machines, and were averaging about eight ounces [of gold] per day to the man. But a few moments passed before I was knee deep in water, with my wash-basin full of dirt, plunging it about endeavoring to separate the dirt from the gold. After washing some fifty pans of dirt, I found I had realized about four bits' worth of gold. Reader, do you know how [one] feels when the gold fever heat has suddenly fallen to about zero? I do. . . . The Indians who were working for Capts. Sutter and Weber gave them leading information, so that they were enabled to know the direction in which new discoveries were to be made. . . . "The morals of the miners of '48 should here be noticed. No person worked on Sunday at digging for gold. . . . We had ministers of the gospel amongst us, but they never preached. Religion had been forgotten, even by its ministers, and instead of their pointing out the narrow way which leads to eternal happiness . . . they might have been seen, with pick-axe and pan, traveling untrodden [untraveled] ways in search of . . . treasure . . . or drinking good health and prosperity with friends." James H. Carson, describing life in the early California gold fields, 1848 Which of the following developments most directly led to the activities described in the excerpt? A A prohibition on the northern extent of slavery in territories west of the Mississippi River B The acquisition of significant territory following the Mexican-American War C The vetoing of the rechartering of the Second Bank of the United States D The completion of the first transcontinental railroad to the Pacific Ocean

B The acquisition of significant territory following the Mexican-American War The United States gained control of California from Mexico as a result of the Mexican-American War, which enabled Americans, primarily, to profit from the gold rush activities described in the excerpt

"Your Memorialist . . . represents to your honorable body, that he has devoted much time and attention to the subject of a railroad from Lake Michigan through the Rocky Mountains to the Pacific Ocean, and that he finds such a route practicable, the results from which would be incalculable—far beyond the imagination of man to estimate. . . . "It would enable us, in the short space of eight days (and perhaps less) to concentrate all the forces of our vast country at any point from Maine to Oregon. . . . Such easy and rapid communication with such facilities for exchanging the different products of the different parts would bring all our immensely wide spread population together. . . . "[W]ith a railroad to the Pacific, and thence to China by steamers, can be performed in thirty days, being now a distance of nearly seventeen thousand miles. . . Then the drills and sheetings of Connecticut, Rhode Island, and Massachusetts, and other manufactures of the United States, may be transported to China in thirty days; and the teas and rich silks of China, in exchange, come back to New Orleans, to Charleston, to Washington, to Baltimore, to Philadelphia, New York, and to Boston, in thirty days more." Asa Whitney, merchant, "National Railroad, Connecting the Atlantic and Pacific Ocean," memorial to the Senate and House of Representatives of the United States, 1845 Sentiments of business leaders and politicians like that expressed in the excerpt most likely contributed to which of the following? A Widespread acceptance of moral reform movements B The creation of diplomatic ties with foreign nations C A sharp decline in immigration from European countries D A fostering of new religious understanding

B The creation of diplomatic ties with foreign nations The proposal in the excerpt sought to construct a railroad but also to create diplomatic ties through economic ventures. During this period the United States established diplomatic and trade relations with Japan and China in order to expand its global economic influence, and politicians and business leaders played central roles in advancing these interests.

"Since the surrender of the armies of the confederate States of America a little has been done toward establishing the Government upon true principles of liberty and justice; and but a little if we stop here. We have broken the material shackles of four million slaves. We have unchained them, from the stake so as to allow them locomotion, provided they do not walk in paths which are trod by white men. . . . But in what have we enlarged their liberty of thought? In what [ways] have we taught them the science and granted them the privilege of self-government? . . . "Unless the rebel states, before admission, should be made republican in spirit, and placed under the guardianship of loyal men, all our blood and treasure will have been spent in vain. . . . There is more reason why [African American] voters should be admitted in the rebel states. . . . In the states they form the great mass of the loyal men. Possibly with their aid loyal governments may be established in most of those states. Without it all are sure to be ruled by traitors; and loyal men, black and white, will be oppressed, exiled, or murdered. "I believe, on my conscience, that on the continued ascendency of [the Republican] party depends the safety of this great nation. [If there is not African American suffrage] in the rebel states then every one of them is sure to send a solid rebel representative . . . to Congress, and cast a solid rebel electoral vote. . . . I am for Negro suffrage in every rebel state. . . . every man, no matter what his race or color; every earthly being who has an immortal soul, has an equal right to justice, honesty, and fair play with every other man; and the law should secure him those rights." Thaddeus Stevens, member of Congress, speech to the House of Representatives, 1867 Which of the following pieces of evidence could best be used to refute Stevens' claim in the excerpt that the Union had done little for formerly enslaved people by 1867? A The assistance granted to formerly enslaved people to seek work in Northern factories B The creation of schools by the Freedmen's Bureau for formerly enslaved people C The widespread redistribution of land from former Confederates to formerly enslaved people D The adoption of work as sharecroppers by many formerly enslaved people

B The creation of schools by the Freedmen's Bureau for formerly enslaved people The creation of the Freedmen's Bureau, especially to help provide for the education of formerly enslaved people, could best be used to refute Stevens' claim that the Union had done little for formerly enslaved people.

"A bank of the United States is in many respects convenient for the Government and useful to the people. Entertaining this opinion, and deeply impressed with the belief that some of the powers and privileges possessed by the existing bank are unauthorized by the Constitution, subversive of the rights of the States, and dangerous to the liberties of the people, I felt it my duty at an early period of my Administration to call the attention of Congress to the practicability of organizing an institution combining all its advantages and obviating [removing] these objections. I sincerely regret that in the act before me I can perceive none of those modifications of the bank charter which are necessary, in my opinion, to make it compatible with justice, with sound policy, or with the Constitution of our country. . . . "Experience should teach us wisdom. Most of the difficulties our Government now encounters and most of the dangers which impend over our Union have sprung from an abandonment of the legitimate objects of Government by our national legislation. . . . Many of our rich men have not been content with equal protection and equal benefits, but have besought us to make them richer by act of Congress. By attempting to gratify their desires we have in the results of our legislation arrayed section against section, interest against interest, and man against man, in a fearful commotion which threatens to shake the foundations of our Union." President Andrew Jackson, Veto Message Regarding the Bank of the United States, 1832 Which of the following factors best supports the argument in the excerpt? A The rise in manufacturing had led to many people relocating to towns and cities as demand for labor increased. B The debates over the federal government's proper role had intensified during the early nineteenth century. C The public visibility of women as leaders of national social and political reform movements had increased. D The South had become increasingly reliant on producing and exporting cotton in the early nineteenth century.

B The debates over the federal government's proper role had intensified during the early nineteenth century. The excerpt displays one of the primary political tensions between Democrats and Whigs over the role of the federal government in issues like the economy. As president and the leader of the Democrats, Jackson espoused a common man vision of the nation, one which corresponded to his support for everyday people against the "rich men" the bank might benefit.

"It is not only important, but, in a degree necessary, that the people of this country, should have an American Dictionary of the English language; for, although the body of the language is the same as in England, . . . yet some differences must exist. Language is the expression of ideas; and if the people of one country cannot preserve an identity of ideas, they cannot retain an identity of language. . . . But the principal differences between the people of this country and of all others, arise from different forms of government, different laws, institutions and customs. Thus the . . . feudal system of England originated terms which formed . . . a necessary part of the language of that country; but, in the United States, many of these terms are no part of our present language,—and they cannot be, for the things which they express do not exist in this country. . . . The institutions in this country which are new and peculiar, give rise to new terms or to new applications of old terms, unknown to the people of England; which cannot be explained by them and which will not be inserted in their dictionaries, unless copied from ours. . . . No person in this country will be satisfied with the English definitions of the words congress, senate, and assembly, court, [etc.] for although these are words used in England, yet they are applied in this country to express ideas which they do not express in that country." Noah Webster, "Preface," An American Dictionary of the English Language, 1828 The excerpt best reflects which of the following historical situations in the early 1800s? A The innovations in the market revolution creating new wealth for Americans B The emergence of a new and distinctive American culture C The transition of the United States to a more participatory democracy D The importance of reading literacy among Americans

B The emergence of a new and distinctive American culture Webster explains why having an American dictionary is essential to promoting and maintaining a distinctive American culture and identity.

"The great increase of drunkenness, within the last half century, among the people of the United States, led a number of philanthropic individuals . . . to consult together, upon the duty of making more united, systematic, and extended efforts for the prevention of this evil. Its cause was at once seen to be, the use of intoxicating liquor; and its appropriate remedy, abstinence. It was also known, that the use of such liquor, as a beverage, is not only needless, but injurious to the health, the virtue, and the happiness of men. It was believed, that the facts which had been . . . collected would prove this . . . ; and that if the knowledge of them were universally disseminated it would, with the divine blessing, do much toward changing the habits of the nation. . . . [The American Temperance Society's] object is . . . the exertion of kind moral influence . . . to effect such a change of sentiment and practice, that drunkenness and all its evils will cease." Introduction to a book of reports from the American Temperance Society, 1835 The sentiments described in the excerpt best reflect which of the following developments? A The use of public protest to effect social change in the antebellum period B The emergence of reform movements during the Second Great Awakening C The establishment of trade relationships with East Asia D The expansion of United States control over territory held by American Indians

B The emergence of reform movements during the Second Great Awakening The sentiment of human perfectibility and techniques focusing on moral improvement promoted by the temperance movement reflect those of other reform movements during this time.

The passage of the Kansas-Nebraska Act was intended to resolve debates about which of the following issues in the 1850s? A The increase in immigration B The expansion of slavery C The fate of the Second Party system D The growth of low-wage factory labor

B The expansion of slavery The United States Congress intended the passage of the Kansas-Nebraska Act in 1854 to resolve the national debates about the expansion of slavery by allowing popular sovereignty to determine the status of slavery in the newly established territory. Ultimately, the law only further increased sectional tensions and strife by overturning the precedent set by the Missouri Compromise.

The efforts of Spanish colonists to convert Native Americans to Christianity were most directly influenced by which of the following simultaneous developments? A The effect of the Columbian Exchange on the population of Europe B The extraction of gold and other wealth from the land in the Americas C The arrival of English colonists seeking religious freedom in the Americas D The success of the French and the Dutch in their interactions with Native Americans

B The extraction of gold and other wealth from the land in the Americas As the Spanish sought to extract more gold and wealth from the land, they developed new institutions that converted many Native Americans to Christianity in order to subjugate the population and assimilate them into Spanish colonial society.

"Louisiana as ceded by France is made part of the United States. Congress may make part of the United States other adjacent territories which shall be justly acquired. "Congress may sever from the United States territory not heretofore within the United States, with consent of a majority of the free males above 21 years, inhabiting such territory." James Madison, secretary of state, proposed constitutional amendment [not passed], 1803 Which of the following best describes the historical situation in which the amendment was proposed? A The Anti-Federalists sought to add a bill of rights to the Constitution. B The federal government sought to acquire more western land in North America. C The United States sought to forcibly remove American Indians from their homelands. D The Constitutional Convention sought to establish the separation of powers between branches of government.

B The federal government sought to acquire more western land in North America. The excerpt explains that this amendment was proposed in the midst of negotiations with France over a proposed treaty that would result in the United States purchasing the Louisiana territory. The proposed amendment also alludes to future efforts to cede territory by the United States.

"I know not how to thank you for the deep and lively interest you have been pleased to take in the cause of . . . the emancipation of a people, who, for two long centuries, have endured, with the utmost patience, a bondage, one hour of which . . . is worse than ages of that which your fathers rose in rebellion to oppose. "It is such indications on the part of the press—which, happily, are multiplying throughout all the land—that kindle up within me an ardent hope that the curse of slavery will not much longer be permitted to make its iron foot-prints in the lacerated [deeply cut] hearts of my . . . brethren. . . . I am called, by way of reproach, a runaway slave. As if it were a crime—an unpardonable crime—for a man to take his inalienable rights! "But why [you,] a New-York editor, born and reared in the State of Maine, far removed from the contaminated . . . atmosphere of slavery, should pursue such a course [supporting abolition], is not so apparent. I will not, however, stop here to ascertain the cause, but deal with fact. . . . "The object . . . is simply to give such an exposition of the degrading influence of slavery upon the master and his [supporters] as well as upon the slave—to excite such an intelligent interest on the subject of American slavery—as may react upon that country, and tend to shame her out of her adhesion to a system which all must confess to disagree with justice. . . . "I am earnestly and anxiously laboring to wipe off this foul blot from the . . . American people, that they may accomplish in behalf of human freedom that which their exalted position among the nations of the earth amply fits them to do." Frederick Douglass to New York Tribune editor Horace Greeley, 1846 Rhetoric in the excerpt would most likely have been interpreted as promoting which of the following? A The creation of societies to send formerly enslaved people to Africa B The immediate end to the practice of slavery through legal reform C The expansion of slavery in new territories through popular sovereignty D The encouragement of enslaved people to take up arms and revolt

B The immediate end to the practice of slavery through legal reform In the excerpt Douglass advocates for bringing to light the issues surrounding slavery as a means to convince both Northerners and Southerners to end the system. Douglass' language in the excerpt does not call for mass rebellion or bloodshed to end the system of slavery, but rather appeals to sentiments that slavery was immoral and unjust, indicating his arguments better aligned with bringing about an end to slavery through legal means.

"Not far from this time Nat Turner's insurrection [a slave rebellion] broke out; and the news threw our town into great commotion. . . . "It was always the custom to have a muster every year. On that occasion every White man shouldered his musket. The citizens and the so-called country gentlemen wore military uniforms. . . . "I knew the houses were to be searched; and I expected it would be done by country bullies and the poor Whites. . . . "It was a grand opportunity for the low Whites, who had no Negroes of their own to scourge. They exulted in such a chance to exercise a little brief authority, and show their subserviency to the slaveholders; not reflecting that the power which trampled on the colored people also kept themselves in poverty, ignorance, and moral degradation. . . . Colored people and slaves who lived in remote parts of the town suffered in an especial manner. In some cases the searchers scattered [gun]powder and shot among their clothes, and then sent other parties to find them, and bring them forward as proof that they were plotting insurrection." Harriet Ann Jacobs, Incidents in the Life of a Slave Girl, published in 1861, describing events earlier in the nineteenth century Which of the following statements would an abolitionist claim supported the ideas expressed in the excerpt? A Southern states had the authority to restrict free African Americans from owning firearms. B The immorality of slavery had a widespread corrupting effect on Southern culture. C The Constitution held that enslaved people were legally considered property. D Southern interests depended on the labor of enslaved people, while the contributions of poor White citizens were minimal.

B The immorality of slavery had a widespread corrupting effect on Southern culture. The excerpt discusses how the prejudice of southern citizens often showed in their behavior toward free and enslaved African Americans. It also discusses the negative effects slavery had on United States culture.

"The preservation of the states in a certain degree of agency is indispensable. It will produce that collision between the different authorities which should be wished for in order to check each other. To attempt to abolish the states altogether, would degrade the councils of our country, would be impracticable, would be ruinous. [John Dickinson] compared the proposed national system to the solar system, in which the states were the planets, and ought to be left to move freely in their proper orbits. . . . If the state governments were excluded from all agency in the national one, and all power drawn from the people at large, the consequence would be, that the national government would move in the same direction as the state governments now do, and would run into all the same mischiefs [troubles]." John Dickinson, delegate from Delaware, summary of a speech at the Constitutional Convention from the notes of James Madison, 1787 Dickinson's desire to preserve "a certain degree of agency" for states is best explained by which of the following developments in the early United States? A The popularity of George Washington B The retention of regional cultural identity in conjunction with national unity C The proposals of Alexander Hamilton to address the financial system D The challenge of gaining support for infrastructure projects

B The retention of regional cultural identity in conjunction with national unity Dickinson's concern for the continued existence of the states after the Constitutional Convention was most reflective of the continuation of local American cultures after the American Revolution. Americans such as Dickinson wanted to preserve the particularity of the states at the same time that the states would become united to protect the gains of the American Revolution.

Historical developments such as that depicted in the image helped advance which of the following? A The conquest of the European continent by Spain B The spread of Spanish influence in the Western Hemisphere C The shift in European economies from capitalism to feudalism D The funding of Christopher Columbus's expeditions to the Caribbean

B The spread of Spanish influence in the Western Hemisphere The increased revenues from sugar cultivation in the Caribbean helped the Spanish fund new explorations that established new settlements in the Western Hemisphere, thereby expanding Spanish influence.

"No roads marked the way to the traveler in California then: but, guided by the sun and well-known mountain peaks, we proceeded on our journey. . . . Some forty or fifty men were at work with the cradle machines, and were averaging about eight ounces [of gold] per day to the man. But a few moments passed before I was knee deep in water, with my wash-basin full of dirt, plunging it about endeavoring to separate the dirt from the gold. After washing some fifty pans of dirt, I found I had realized about four bits' worth of gold. Reader, do you know how [one] feels when the gold fever heat has suddenly fallen to about zero? I do. . . . The Indians who were working for Capts. Sutter and Weber gave them leading information, so that they were enabled to know the direction in which new discoveries were to be made. . . . "The morals of the miners of '48 should here be noticed. No person worked on Sunday at digging for gold. . . . We had ministers of the gospel amongst us, but they never preached. Religion had been forgotten, even by its ministers, and instead of their pointing out the narrow way which leads to eternal happiness . . . they might have been seen, with pick-axe and pan, traveling untrodden [untraveled] ways in search of . . . treasure . . . or drinking good health and prosperity with friends." James H. Carson, describing life in the early California gold fields, 1848 The excerpt best reflects the development of which of the following? A The emergence of an abolitionist movement in the western territories B The widely held belief that the United States had a right to expand westward C The increasing importance of cotton exports to the United States economy D The debates about Native Americans and Mexican nationals dispossessed of land in California

B The widely held belief that the United States had a right to expand westward As part of the idea of Manifest Destiny, many Americans in the nineteenth century believed that the United States had the right and responsibility to expand control across the North American continent to the Pacific.

"The preservation of the states in a certain degree of agency is indispensable. It will produce that collision between the different authorities which should be wished for in order to check each other. To attempt to abolish the states altogether, would degrade the councils of our country, would be impracticable, would be ruinous. [John Dickinson] compared the proposed national system to the solar system, in which the states were the planets, and ought to be left to move freely in their proper orbits. . . . If the state governments were excluded from all agency in the national one, and all power drawn from the people at large, the consequence would be, that the national government would move in the same direction as the state governments now do, and would run into all the same mischiefs [troubles]." John Dickinson, delegate from Delaware, summary of a speech at the Constitutional Convention from the notes of James Madison, 1787 The framers of the United States Constitution initially responded to abuses of executive authority by the British monarch in which of the following ways? A They granted the Supreme Court the ability to veto treaties negotiated by the president. B They established the separation of powers between the president and Congress. C They provided for the popular election of the president by all voters. D They guaranteed that basic rights could not be limited by the president.

B They established the separation of powers between the president and Congress. Based on their experience of being unable to check the executive authority of the British monarch during the American Revolution, the framers of the Constitution established the separation of powers between the legislative and the executive branch, which enabled Congress to check the authority of the president.

"Let us, then, with courage and confidence, pursue our own Federal and [Democratic-] Republican principles, our attachment to union and representative government. Kindly separated by nature and a wide ocean from the exterminating havoc of one quarter of the globe; too high-minded to endure the degradations of the others; possessing a chosen country, with room enough for our descendants . . . ; entertaining a due sense of our equal right to the use of our own faculties, to the acquisitions of our own industry, to honor and confidence from our fellow citizens, resulting not from birth, but from our actions and their sense of them; enlightened by a benign religion . . . —with all these blessings, what more is necessary to make us a happy and a prosperous people? Still one thing more, fellow citizens—a wise and frugal Government, which shall restrain men from injuring one another, shall leave them otherwise free to regulate their own pursuits of industry and improvement, and shall not take from the mouth of labor the bread it has earned. This is the sum of good government; and this is necessary to close the circle of our felicities." President Thomas Jefferson, first inaugural address, 1801 Which of the following was most likely a main purpose of Jefferson's inaugural address? A To advocate and explain the constitutionality of certain laws B To summarize his beliefs about the ideal political system C To warn European countries against renewing conflicts with the United States D To justify the cost of the Louisiana Purchase

B To summarize his beliefs about the ideal political system Along with stating his support for limited government, Jefferson also touched on the ideals of republicanism in this excerpt from his inaugural address, which represented an expression of his ideal political system.

"The existence of [colonial] subregions leads us to another question: whether the Middle Colonies in fact represented a coherent region at all. . . . In important respects, the Middle Colonies can be divided into separate societies focused around the cities of New York and Philadelphia. Thus the economies of [New York] and northern New Jersey were tied closely to that of New York City, while those of southern New Jersey, Pennsylvania, and northern Delaware were linked to Philadelphia. Those areas grew at very different rates, and they possessed quite distinct characteristics. . . . "Nonetheless, the Middle Colonies did share a number of things. One was their geography, a combination of climate and topography and setting, which determined some of the ways the land could be put to use, its accessibility to both intra-regional and international commerce, and its strategic importance in imperial competition. It was a region organized around extensive inland waterways, which gave merchants an almost unparalleled access to the American interior, building upon trade routes that pre-dated European settlement. . . . "Perhaps the most important argument for the coherence of the mid-Atlantic as a region is the extent to which those colonies shared a common history. . . . "The most often-noted characteristic of the region was the diversity of its peoples. . . . The society of the Middle Colonies surely was 'America's first plural society.' . . . There were two principal sources of the growing diversity of the European settlements. One was historical: New York, New Jersey, and Delaware were all conquered colonies, with Dutch, Swedish, Finnish, and many other populations already resident at the time of English conquest. The other was the consolidation that occurred as the colonies of six European nations along the Atlantic coast in the early seventeenth century were reduced to two by century's end, those of [Protestant] England and those of [Catholic] France. The result was that [diverse] European Protestants heading for the New World were concentrated within English colonies, a situation that virtually mandated some form of toleration. . . . Toleration and pluralism, it turns out, were not based solely on enlightened benevolence." Ned C. Landsman, historian, Crossroads of Empire: The Middle Colonies in British North America, published in 2010 Which of the following best describes Landsman's argument in the last paragraph of the excerpt? A English conquests in the Middle Colonies were harsher than the conquests of other European empires in the Americas. B Toleration of religious diversity in the Middle Colonies was made a necessity because of patterns of migration. C The spread of Enlightenment values was the main reason that pluralism developed in the Middle Colonies. D The French colonies in North America and the Middle Colonies had very similar settler populations.

B Toleration of religious diversity in the Middle Colonies was made a necessity because of patterns of migration. In the last paragraph of the excerpt, Landsman argues that because of the religious diversity of the migrants who came to the Middle Colonies, religious toleration became a necessity for English colonial governments.

"Yes: Mexico must be thoroughly chastised! . . . The news of yesterday [at the southern border] has added the last argument wanted to prove the necessity of an immediate Declaration of War by our government toward its southern neighbor. "We are justified in the face of the world, in having treated Mexico with more forbearance [tolerance] than we have ever yet treated an enemy. . . . We have . . . submitted thus far to a most offensive rejection of an Ambassador personifying the American nation, and waited for years without payment of the claims of our injured merchants. We have sought peace through every avenue, and shut our eyes to many things, which, had they come from England or France, the President would not have dared to pass over without stern and speedy resentment. We have dammed up our memory, of what had passed in the South [Texas] years ago—of devilish massacres of some of our bravest and noblest sons . . . in violation of all the rules of war. . . . "We think there can be no doubt of the truth of yesterday's news; and we are sure the people here, ten to one, are for prompt and hostilities. . . . Let our arms now be carried with a spirit which shall teach the world that, while we are not forward for a quarrel, America knows how to crush, as well as how to expand!" Walt Whitman, journalist and poet, editorial in the Brooklyn Eagle, 1846 "President [James K. Polk] in his message, as a pretext for sending our army to invade and conquer the country upon the Rio Grande, says: "Texas by its [legislative] act of December 19, 1836, had declared the [Rio Grande] to be the boundary of that [formerly independent] republic.' . . . The truth is that Texas had agreed upon the Nueces [River] as her boundary. . . . "If [Mexico] be ours, why does he seek to justify the taking possession of it by references to the fact that Mexico is indebted to some of our people? If it be not ours, and he has taken possession of it in order to compel Mexico to pay those debts, why not say so? The fact that Mexico has not paid the debts due to our citizens can have no legitimate connection with taking possession of [it as] our own soil. But [the president] was obviously conscious that this invasion of the Mexican territory could not be justified. . . . "When the Executive and Congress openly and avowedly took upon themselves the responsibility of extending and perpetuating slavery by the annexation of Texas, and by the total overthrow and subversion of the Constitution, . . . my confidence in the stability of our institutions was shaken, destroyed. . . . Our Union continues, but our Constitution is gone. . . . ". . . No man regards this war as just. We know, the country knows, and the civilized world are conscious, that it has resulted from a desire to extend and sustain an institution on which the curse of the Almighty most visibly rests." Joshua Giddings, congressman from Ohio, speech in the United States House of Representatives, 1846 Which of the following comparisons best describes Whitman's and Giddings' arguments about the Mexican-American War? A Whitman believed the United States should show patience before war, while Giddings believed the country had tolerated abuses for too long without war. B Whitman argued that the war was intended to deter bad behavior, while Giddings argued that the war represented aggression by the United States. C Giddings asserted that the nation entered the war as a last resort, while Whitman asserted that the war was unnecessary to achieve the goals of the United States. D Giddings claimed that the war had popular support in the United States, while Whitman claimed that most Americans opposed the war.

B Whitman argued that the war was intended to deter bad behavior, while Giddings argued that the war represented aggression by the United States. In the excerpts, Whitman argued that only United States military force could stop unjust actions by Mexico, while Giddings argued, as he said in the first paragraph of the second excerpt, that the war was based on a false "pretext" so the United States could "conquer" parts of Mexico.

"Forces committed to restoring White supremacy launched a ruthless, bloody campaign of terror and intimidation against freedpeople and their White allies in the South. As young southern units of the Republican Party broke under those blows and the Republicans of the North retreated and grew more conservative, Reconstruction collapsed. With it went many . . . gains. A resurgent southern elite once again set about imposing White supremacy and tyrannical labor discipline while stripping freedpeople of many of their civic and political rights." Bruce Levine, historian, The Fall of the House of Dixie, 2013 "For many poor Whites throughout the South, Jim Crow laws alone could not ease their most persistent fear. In regions like northern Louisiana, with little but pine trees rising from its barren soil, White men found themselves competing with [formerly enslaved people], and during the dozen years of Reconstruction they had not known which race would prevail. "Such men had dropped away from the Ku Klux Klan after President Grant's crackdown, but their simmering resentments had grown. With control of the South passing again to the Democrats, powerless Whites were joining plantation owners to ensure that Black workers remained without their basic rights." A. J. Langguth, historian, After Lincoln, 2014 Which of the following arguments about Reconstruction policies would both authors most likely disagree with? A White Southerners across all economic classes rejected Reconstruction policies. B With Republicans in retreat, Southern Democrats grew more supportive of Reconstruction policies. C Reconstruction policies failed to prevent the spread of violence against formerly enslaved people. D Unfair labor conditions in the South persisted despite Reconstruction policies.

B With Republicans in retreat, Southern Democrats grew more supportive of Reconstruction policies. In the excerpts, both authors argue that declining Republican influence resulted in the dismantling of gains made during Reconstruction. Neither author supports the claim that Southern Democrats began to support Reconstruction policies in the vacuum created by Republican withdrawal.

"What fault has there been on the part of the General Government of the United States? Why break up this Union? Will any gentleman be so kind as to particularize a single instance worthy of debate, in which the Federal Government has been derelict [negligent] in the discharge of its duty, or has failed to accomplish the purposes of its organization? . . . "I am not here . . . to defend the election of Abraham Lincoln. I believe that his election was virtually a fraud upon the people of the United States . . . nominated, as he was, by a sectional party, and upon a sectional platform, with no representation in the body which nominated him from the South; but he was nominated and elected according to the forms of law. . . . "Let us look . . . at the evils that must result from secession. The first, in my opinion, would be that our country would not only be divided into a Northern Confederacy and into Southern Confederacy, but, soon or later it would be divided into sundry [several] petty Confederacies. We would have a Central Confederacy, a Confederacy of the States of the Mississippi Valley, a Pacific Confederacy, a Western Confederacy, an Eastern Confederacy, a Northern and a Southern Confederacy. ". . . It is easy perhaps to break down this Government; but, sir, when we break it down it will not be so easy a matter to build it up. . . . Gentlemen cry out against the tyranny of their own government, and yet denounce [those opposed to secession] because we hesitate to allow ourselves to be thrust into the embraces of such a military dictatorship." Waitman T. Willey, addressing the Virginia State Secession Convention, March 4, 1861 The excerpt best serves as evidence that, in 1861, A citizens in the Northern states did not want Abraham Lincoln as president B citizens in the Southern states were deeply divided over secession C citizens in the Northern states were prepared to accommodate slavery D citizens in the Northern states would not accept a Confederate government

B citizens in the Southern states were deeply divided over secession In the excerpt, Willey argues that despite his severe disagreements with developments in the Northern states, he does not see how Virginia seceding from the Union would bring a better outcome. He warns that secession would likely be disastrous.

The percentages of White and African American populations in South Carolina shown in the table most directly suggest the A availability of land for settlement in the Deep South B expansion of slavery in the Deep South C growth of textile manufacturing in southern states D reliance on immigrant labor in southern states

B expansion of slavery in the Deep South The fact that in 1800 South Carolina had in its population the largest percentage of African Americans among southern states suggests the expansion of slavery in the Deep South to support plantation economies at this time.

"The laity [church members] . . . saw to it that the Second Great Awakening exerted much of its influence through purposeful voluntary associations, typically headed by boards of directors on which laypersons appeared prominently. . . . "Contemporaries called the interlocking, interdenominational directorates of these organizations "the Evangelical United Front" or "the Benevolent Empire." . . . "The social reforms embraced by the Evangelical United Front characteristically involved creating some form of personal discipline serving a goal or redemption. Prison reform serves as an example: No longer would the prison be intended only as a place to hold persons awaiting trial, coerce debt payment, or inflict retributive justice. Reformers reconceived the prison as corrective function, as a 'penitentiary' or 'reformatory,' in the vocabulary they invented. Besides prisoners, other people who did not function as free moral agents might become objects of the reformers' concern: alcoholics, children, slaves, the insane. The goal of the reformers in each case was to substitute for external constraints the inner discipline of morality. Some historians have interpreted the religious reformers as motivated simply by an impulse to impose 'social control,' but it seems more accurate to describe their concern as redemptive, and more specifically the creation of responsible personal autonomy. Liberation and control represented two sides of the redemptive process as they conceived it. Christians who had achieved self-liberation and self-control through conversion not surprisingly often turned to a concern with the liberation and discipline of others. . . . "The religious awakenings of the early nineteenth century marshaled powerful energies in an age when few other social agencies in the United States had the capacity to do so. [The] Evangelical United Front organized its voluntary associations on a national, indeed international, level, at a time when little else in American society was organized, when there existed no nationwide business corporation save the Second Bank of the United States and no nationwide government bureaucracy save the Post Office. Indeed, the four major evangelical denominations together employed twice as many people, occupied twice as many premises, and raised at least three times as much money as the Post Office." Daniel Walker Howe, historian, What Hath God Wrought: The Transformation of America, 1815-1848, published in 2007 A piece of evidence used by Howe in the second paragraph of the excerpt to support his argument about the goals of prison reform was that prison reformers A saw prisons primarily as a form of punishment B intended to use prisons to rehabilitate criminals C sought to expand prisons to force debt repayments D thought prisons were only to hold people before trial

B intended to use prisons to rehabilitate criminals In the second paragraph of the excerpt, Howe provides evidence that prison reformers believed prisons should serve a "corrective" function to support his argument that their goal was to encourage personal redemption.

"In 1739 arrived among us from Ireland the Reverend Mr. Whitefield, who had made himself remarkable there as [a traveling] preacher. He was at first permitted to preach in some of our churches; but the clergy, taking a dislike to him, soon refused him their pulpits, and he was obliged to preach in the fields. The multitudes of all [members of different religious groups] that attended his sermons were enormous, and it was a matter of speculation to me . . . to observe the extraordinary influence of his oratory on his hearers, and how much they admired and respected him. . . . It was wonderful to see the change soon made in the manners of our inhabitants. From being thoughtless or indifferent about religion, it seemed as if all the world were growing religious, so that one could not walk through the town in an evening without hearing psalms sung in different families of every street. "And it being found inconvenient to assemble in the open air, subject to its [harsh conditions], the building of a house to meet in was no sooner proposed . . . and the work [of erecting the building] was carried on with such spirit as to be finished in a much shorter time than could have been expected. Both house and ground were vested in trustees, expressly for the use of any preacher of any religious persuasion who might desire to say something to the people at Philadelphia." Benjamin Franklin, from his autobiography, describing events in 1739 The events of the First Great Awakening illustrated by the excerpt led to A established clergy strengthening their control over the religious and social life of colonial society B new denominations attracting followers who were drawn to the dynamic sermons of the new preachers C religious enthusiasm declining in cities where it had to complete more directly with Enlightenment ideas D few changes in most people's lives beyond increased church attendance and very little change in how they interacted with others

B new denominations attracting followers who were drawn to the dynamic sermons of the new preachers The First Great Awakening led to the growth of a number of Protestant denominations such as Methodists and Baptists in the colonies as people flocked to hear the dynamic sermons of the new preachers.

"Not far from this time Nat Turner's insurrection [a slave rebellion] broke out; and the news threw our town into great commotion. . . . "It was always the custom to have a muster every year. On that occasion every White man shouldered his musket. The citizens and the so-called country gentlemen wore military uniforms. . . . "I knew the houses were to be searched; and I expected it would be done by country bullies and the poor Whites. . . . "It was a grand opportunity for the low Whites, who had no Negroes of their own to scourge. They exulted in such a chance to exercise a little brief authority, and show their subserviency to the slaveholders; not reflecting that the power which trampled on the colored people also kept themselves in poverty, ignorance, and moral degradation. . . . Colored people and slaves who lived in remote parts of the town suffered in an especial manner. In some cases the searchers scattered [gun]powder and shot among their clothes, and then sent other parties to find them, and bring them forward as proof that they were plotting insurrection." Harriet Ann Jacobs, Incidents in the Life of a Slave Girl, published in 1861, describing events earlier in the nineteenth century Which of the following pieces of evidence best supports the excerpt's depiction of reactions to slave rebellions? A Southern states banned the importation of slaves from Africa. B Northerners agreed that slavery was a positive institution for society. C Additional restrictions were placed on enslaved and free African Americans. D The majority of slaveholders moved toward using alternative forms of labor.

C Additional restrictions were placed on enslaved and free African Americans. The excerpt references how the aftermath of Nat Turner's Rebellion affected the people, both free and enslaved. Legal restrictions were enacted forbidding enslaved people from socializing with each other, and laws were passed to strip free African Americans of specific legal rights.

"After [the Confederate surrender at] Appomattox the South's political leaders saw themselves entering an era of revolutionary changes imposed by the national government, which many viewed as an outside power. Continuing a long pattern of American . . . behavior, many whites found an outlet for their frustration by attacking those deemed responsible for their suffering: white Republicans and blacks. . . . "Frustrated at their inability to bring their states back to Democratic control, some southerners turned to the Ku Klux Klan and other white supremacist organizations, using terrorism to eliminate opposition leaders and to strike fear into the hearts of rank-and-file Republicans, both black and white. . . . "[Violence] in Alabama, Louisiana, Mississippi, and South Carolina exposed the impotence of the Republican party in the South and the determination of Democrats to defeat their opponents by any means necessary. The final triumph of the counterrevolution awaited the withdrawal of northern Republican support from the so-called 'carpetbag regimes' in 1877. The inconsistency of federal Reconstruction policy and the strength of southern resistance seem to have doomed the Reconstruction experiment to inevitable collapse. Although Americans have often been loathe to concede that violence may bring about [political] change, terrorism in the Reconstruction era was instrumental in achieving the ends desired by its perpetrators." George C. Rable, historian, But There Was No Peace: The Role of Violence in the Politics of Reconstruction, published in 1984 "In its pervasive impact and multiplicity of purposes, . . . the wave of counterrevolutionary terror that swept over large parts of the South between 1868 and 1871 lacks a counterpart . . . in the American experience. . . . "By 1870, the Ku Klux Klan . . . had become deeply entrenched in nearly every Southern state. . . . In effect, the Klan was a military force serving the interests of the Democratic party, the planter class, and all those who desired the restoration of white supremacy. . . . "Adopted in 1870 and 1871, a series of Enforcement Acts embodied the Congressional response to violence. . . . As violence persisted, Congress enacted a far more sweeping measure—the Ku Klux Klan Act of April 1871. This for the first time designated certain crimes committed by individuals as offenses punishable under federal law. . . . If states failed to act effectively against them, [these offenses could] be prosecuted by federal district attorneys, and even lead to military intervention. . . . "Judged by the percentage of Klansmen actually indicted and convicted, the fruits of 'enforcement' seem small indeed, a few hundred men among the thousands guilty of heinous crimes. But in terms of its larger purposes—restoring order, reinvigorating the morale of Southern Republicans, and enabling blacks to exercise their rights as citizens—the policy proved a success. . . . So ended the Reconstruction career of the Ku Klux Klan. . . . National power had achieved what most Southern governments had been unable, and Southern white public opinion unwilling, to accomplish: acquiescence in the rule of law." Eric Foner, historian, Reconstruction: America's Unfinished Revolution, 1863-1877, published in 1988 Which of the following pieces of evidence could best be used to modify Foner's main argument in the second excerpt? A Many African Americans were elected to the United States Congress for the first time during Reconstruction. B Southern states were not permitted to rejoin the Union until they had ratified the Fourteenth Amendment. C After 1877 Democrats in the South legislated restrictions on the ability of African Americans to vote. D Radical Republicans passed the Civil Rights Act of 1875 to protect African Americans against abuses.

C After 1877 Democrats in the South legislated restrictions on the ability of African Americans to vote Evidence that Democrats restricted African American voting rights after 1877 would modify Foner's argument that federal action ended the Ku Klux Klan in the early 1870s because this shows that the federal government allowed new forms of attack on African American civil rights after the end of Reconstruction.

"The great increase of drunkenness, within the last half century, among the people of the United States, led a number of philanthropic individuals . . . to consult together, upon the duty of making more united, systematic, and extended efforts for the prevention of this evil. Its cause was at once seen to be, the use of intoxicating liquor; and its appropriate remedy, abstinence. It was also known, that the use of such liquor, as a beverage, is not only needless, but injurious to the health, the virtue, and the happiness of men. It was believed, that the facts which had been . . . collected would prove this . . . ; and that if the knowledge of them were universally disseminated it would, with the divine blessing, do much toward changing the habits of the nation. . . . [The American Temperance Society's] object is . . . the exertion of kind moral influence . . . to effect such a change of sentiment and practice, that drunkenness and all its evils will cease." Introduction to a book of reports from the American Temperance Society, 1835 Which of the following evidence did the American Temperance Society in the excerpt use to support its argument about the need for the temperance movement? A Federal government excise taxes on alcoholic beverages were too high. B Restrictions on alcohol could discourage European immigrants. C Alcohol consumption damaged people's physical and emotional well-being. D Men wasted too much money buying alcohol, and their families became poor.

C Alcohol consumption damaged people's physical and emotional well-being. The excerpt refers to the negative effects on health caused by excessive alcohol consumption.

Which of the following explains how the growth of a market-based economy in the United States in the early 1800s most directly influenced changes in gender roles? A As both men and women took paid employment, social and political inequality between the sexes declined significantly. B As an economy based on commercial transactions encouraged society to become more secular, many young women gave up traditional families. C As home and the workplace became separated, women were increasingly expected to be responsible for housework and childcare while men took jobs outside the home. D As working for wages became more common, many people ceased to view the family as a refuge from the struggle for subsistence.

C As home and the workplace became separated, women were increasingly expected to be responsible for housework and childcare while men took jobs outside the home. During the market revolution, the divisions between men's and women's spheres became more established, with men taking jobs in public and women maintaining the home and children, leading to a hardening of gender expectations.

"There are those who are dissatisfied with me. To such I would say: You desire peace; and you blame me that we do not have it. But how can we attain it? . . . "But to be plain, you are dissatisfied with me about the Negro. . . . You dislike the emancipation proclamation; and, perhaps, would have it retracted. You say it is unconstitutional—I think differently. I think the Constitution invests its commander-in-chief, with the law of war, in time of war. The most that can be said, if so much, is, that slaves are property. Is there—has there ever been—any question that by the law of war, property, both of enemies and friends, may be taken when needed? And is it not needed whenever taking it, helps us, or hurts the enemy? . . . "You say you will not fight to free Negroes. Some of them seem willing to fight for you. . . . I issued the proclamation on purpose to aid you in saving the Union. . . . Why should they do anything for us, if we will do nothing for them? If they stake their lives for us, they must be prompted by the strongest motive—even the promise of freedom. And the promise being made, must be kept." President Abraham Lincoln, letter to James Conkling explaining why he issued the Emancipation Proclamation, 1863 Lincoln's rhetoric in the excerpt would most likely have been interpreted as promoting which of the following arguments? A Allowing slavery to exist was still a political option. B Maintaining the blockade of Southern states was difficult. C Changing the purpose of the war would strengthen the Union cause. D Achieving the Confederacy's unconditional surrender was the Union's main objective.

C Changing the purpose of the war would strengthen the Union cause. After a series of victories by the Confederate armies in the first two years of the war, Northern support was beginning to wane. Lincoln, realizing he needed more troops and looking to redefine the purpose of the war, issued the Emancipation Proclamation in part to encourage enslaved people and free African Americans to join the Union army and fight for the permanent abolition of slavery.

"The creation of a home market is not only necessary to procure for our agriculture a just reward of its labors, but it is indispensable to obtain a supply of our necessary wants. . . . Suppose no actual abandonment of farming, but, what is most likely, a gradual and imperceptible employment of population in the business of manufacturing, instead of being compelled to resort to agriculture. . . . Is any part of our common country likely to be injured by a transfer of the theatre of [manufacturing] for our own consumption from Europe to America? ". . . Suppose it were even true that Great Britain had abolished all restrictions upon trade, and allowed the freest introduction of the [products] of foreign labor, would that prove it unwise for us to adopt the protecting system? The object of protection is the establishment and perfection of the [manufacturing] arts. In England it, has accomplished its purpose, fulfilled its end. . . . The adoption of the restrictive system, on the part of the United States, by excluding the [products] of foreign labor, would extend the [purchasing] of American [products], unable, in the infancy and unprotected state of the arts, to sustain a competition with foreign fabrics. Let our arts breathe under the shade of protection; let them be perfected as they are in England, and [then] we shall be ready . . . to put aside protection, and enter upon the freest exchanges." Henry Clay, speaker of the House of Representatives, speech in Congress, 1824 Which of the following was an interpretation of the speech by opponents of the goals Clay expressed in the excerpt? A Clay was seeking to discourage factory production to protect traditional artisans. B Clay's proposal would require the expansion of slavery to supply factory workers. C Clay's manufacturing plan would benefit one section of the country more than others. D Clay was seeking to encourage an increase in imports of British manufactured goods.

C Clay's manufacturing plan would benefit one section of the country more than others. In the excerpt, Clay advocated the use of tariffs to protect the new industries of the North. Opponents of Clay's goals, often plantation holders and common farmers, believed this plan would hurt agriculturalists by making the manufactured goods they purchased more expensive. The opposition was especially pronounced in the South in the 1820s and 1830s, where many political leaders rejected tariffs as an attack on their way of life.

Which of the following describes a trend in exports from England to British North America between 1699 and 1749 indicated in the table? A British colonists began to export manufactured goods to compete with England. B England exported more leather to British North America in 1749 than in 1699. C Demand in the colonies for manufactured goods from England greatly increased. D France increased its imports from the colonies in order to undermine English influence.

C Demand in the colonies for manufactured goods from England greatly increased. The table shows that the value of exports to British North America, especially manufactured goods such as textiles, greatly increased during this period, which was a result of increased colonial demand.

"Since the surrender of the armies of the confederate States of America a little has been done toward establishing the Government upon true principles of liberty and justice; and but a little if we stop here. We have broken the material shackles of four million slaves. We have unchained them, from the stake so as to allow them locomotion, provided they do not walk in paths which are trod by white men. . . . But in what have we enlarged their liberty of thought? In what [ways] have we taught them the science and granted them the privilege of self-government? . . . "Unless the rebel states, before admission, should be made republican in spirit, and placed under the guardianship of loyal men, all our blood and treasure will have been spent in vain. . . . There is more reason why [African American] voters should be admitted in the rebel states. . . . In the states they form the great mass of the loyal men. Possibly with their aid loyal governments may be established in most of those states. Without it all are sure to be ruled by traitors; and loyal men, black and white, will be oppressed, exiled, or murdered. "I believe, on my conscience, that on the continued ascendency of [the Republican] party depends the safety of this great nation. [If there is not African American suffrage] in the rebel states then every one of them is sure to send a solid rebel representative . . . to Congress, and cast a solid rebel electoral vote. . . . I am for Negro suffrage in every rebel state. . . . every man, no matter what his race or color; every earthly being who has an immortal soul, has an equal right to justice, honesty, and fair play with every other man; and the law should secure him those rights." Thaddeus Stevens, member of Congress, speech to the House of Representatives, 1867 Which of the following developments could best be used as evidence to support Stevens' claim about African American suffrage in the last paragraph of the excerpt? A Republican candidates won most presidential electoral votes in the North in the late 1860s and 1870s. B Many Northern business leaders supported the Republican Party during Reconstruction. C Democrats dominated the Southern states after suppressing African American voting rights. D President Ulysses S. Grant used federal authority to counter voter intimidation in the South.

C Democrats dominated the Southern states after suppressing African American voting rights. The decline of the Republican Party in the South beginning in the 1870s and accelerating after the end of Reconstruction, partly because of Southern Democratic suppression of African American voting rights, best serves as evidence for Stevens' argument that African American suffrage was needed to support the Republican Party in the South.

"There are those who are dissatisfied with me. To such I would say: You desire peace; and you blame me that we do not have it. But how can we attain it? . . . "But to be plain, you are dissatisfied with me about the Negro. . . . You dislike the emancipation proclamation; and, perhaps, would have it retracted. You say it is unconstitutional—I think differently. I think the Constitution invests its commander-in-chief, with the law of war, in time of war. The most that can be said, if so much, is, that slaves are property. Is there—has there ever been—any question that by the law of war, property, both of enemies and friends, may be taken when needed? And is it not needed whenever taking it, helps us, or hurts the enemy? . . . "You say you will not fight to free Negroes. Some of them seem willing to fight for you. . . . I issued the proclamation on purpose to aid you in saving the Union. . . . Why should they do anything for us, if we will do nothing for them? If they stake their lives for us, they must be prompted by the strongest motive—even the promise of freedom. And the promise being made, must be kept." President Abraham Lincoln, letter to James Conkling explaining why he issued the Emancipation Proclamation, 1863 The excerpt could best be used by historians studying which of the following? A What prevented European powers from supporting the South B What motivated African Americans during the war C How Lincoln used executive powers to initiate wartime policy D How Confederate strategy prevented a rapid Union victory

C How Lincoln used executive powers to initiate wartime policy As Lincoln explains in the excerpt, he believed the Constitution gave the president broad powers during wartime. Lincoln sought to use these powers to frame the Emancipation Proclamation as necessary to achieving victory in the Civil War.

"Jackson truly believed that, compared to his predecessors' combination of high-minded rhetoric, treachery, and abandonment, his Indian policy was 'just and humane.' . . . ". . . Jackson's paternalism was predicated on his assumption, then widely but not universally shared by white Americans, that all Indians . . . were [irrational] and inferior to all whites. His promises about voluntary and compensated relocation . . . were constantly undermined by delays and by sharp dealing by War Department negotiators—actions Jackson condoned. . . . Jackson tried to head off outright fraud, but the removal bill's allotment scheme invited an influx of outside speculators, who wound up buying between 80 and 90 percent of the land owned by Indians who wished to stay at a fraction of its actual worth. At no point did Jackson consider allowing even a small number of Georgia Cherokees who preferred to stay to do so in select enclaves, an option permitted to small numbers of Iroquois in upstate New York and Cherokees in western North Carolina. . . . Bereft of long-term planning and a full-scale federal commitment, the realities of Indian removal belied Jackson's rhetoric. Although the worst suffering was inflicted after he left office, Jackson cannot escape responsibility for setting in motion an insidious policy that uprooted tens of thousands of Choctaws and Creeks [from the Southeast] during his presidency." Sean Wilentz, historian, The Rise of American Democracy: Jefferson to Lincoln, published in 2005 Which of the following pieces of evidence would help modify an argument in the excerpt about President Jackson's intentions toward American Indians? A Some representatives of Jackson profited by purchasing land from American Indians who were forcibly relocated. B Some of Jackson's agents encouraged American Indians from Ohio and Florida to leave their homelands as well. C Jackson had led United States armies that conquered American Indian peoples in the Southeast and forced land cessions. D Jackson believed that relocating American Indians was the only action that would enable them to preserve their way of life.

C Jackson had led United States armies that conquered American Indian peoples in the Southeast and forced land cessions. Evidence that Jackson had commanded United States armies in the conquest of American Indian peoples would modify the argument in the excerpt that Jackson believed that his intentions toward American Indians were paternalistic and humane.

"Antebellum planters . . . were very interested in the control of black movement. They were also keen to master their slaves' senses of pleasure. Seeking to contain [African Americans] even further than laws, curfews, bells, horns, and patrols already did, some planters used plantation [parties] as a paternalist mechanism of social control. Plantation parties, which carefully doled out joy on Saturday nights and holidays, were intended to seem benevolent and to inspire respect, gratitude, deference, and importantly, obedience. . . . The most important component of paternalistic plantation parties was the legitimating presence of the master. ". . . [Yet] again and again, slaves sought out illicit, secular gatherings of their own creation. They disregarded curfews and pass laws to escape to secret parties where . . . pleasures such as drinking, eating, dancing, and dressing up were the main amusements. . . . ". . . In the context of enslavement, such exhilarating pleasure . . . must be understood as important and meaningful enjoyment, as personal expression, and as oppositional." Stephanie M. H. Camp, historian, Closer to Freedom: Enslaved Women and Everyday Resistance in the Plantation South, 2004 Which of the following pieces of evidence could best be used to modify the argument in the excerpt that many enslaved people engaged in oppositional activities? A When possible, enslaved African Americans sought to escape to the North. B Some enslaved African Americans learned to read in spite of laws banning it. C Large-scale rebellions by enslaved African Americans in the first half of the 1800s were largely unsuccessful. D Many enslaved African Americans maintained family units, even with spouses and children on other plantations.

C Large-scale rebellions by enslaved African Americans in the first half of the 1800s were largely unsuccessful. Although the excerpt argues that many slaves engaged in oppositional activities, most of this opposition was hidden. Open opposition such as that expressed in slave rebellions was largely unsuccessful.

"I am filled with deep emotion at finding myself standing here in the place . . . from which sprang the institutions under which we live. . . . I have never had a feeling politically that did not spring from the sentiments embodied in the Declaration of Independence. . . . It was not the mere matter of the separation of the colonies from the mother land; but something in that Declaration giving liberty, not alone to the people of this country, but hope to the world for all future time. It was that which gave promise that in due time the weights should be lifted from the shoulders of all men. . . . "Now, my friends, can this country be saved upon that basis? If it can, I will consider myself one of the happiest men in the world if I can help to save it. If it can't be saved upon that principle, it will be truly awful. "Now, in my view of the present aspect of affairs, there need be no bloodshed and war. . . . And I may say in advance, there will be no blood shed unless it be forced upon the Government. . . . "My friends, this is a wholly unprepared speech. I did not expect to be called upon to say a word when I came here. . . . I may, therefore, have said something indiscreet, but I have said nothing but what I am willing to live by, and, in the pleasure of Almighty God, die by." President-elect Abraham Lincoln, speaking at Independence Hall in Philadelphia, February 22, 1861 The excerpt best serves as evidence for which of the following developments? A Abraham Lincoln attempted to increase the size of the army to regain Southern states. B Lincoln rejected allowing any additional states to join the United States. C Lincoln sought to avoid violence over the issues that divided the country. D Lincoln intended to enforce the Kansas-Nebraska Act.

C Lincoln sought to avoid violence over the issues that divided the country. In the excerpt Lincoln stated that, despite his belief in adhering to certain principles, the federal government would only show force if fired upon.

"The next matter I shall recommend to you is the providing more effectively for the security of your frontiers against [American] Indians, who notwithstanding the many parties of Rangers [militia, or local men who volunteered for colonial defense] have . . . killed and carried off at least twenty of our outward inhabitants and Indian allies; I have attempted by several ways to oppose those [invasions] but after some trouble and expense have only experienced that our people are not ready for warlike undertakings. . . . The [condition of our Indian allies has] of late approved themselves to be ready and faithfully allied, and I am persuaded that setting them along our frontiers without all our inhabitants . . . would be a better and cheaper safeguard to the country than the old method of Rangers." Virginia Governor Alexander Spotswood, addressing the members of the House of Burgesses, 1713 Which of the following groups would most likely oppose the ideas expressed in the excerpt? A White settlers on the western Virginia frontier B White settlers in coastal Virginia C Members of allied American Indian groups D Members of the House of Burgesses

C Members of allied American Indian groups Spotswood is suggesting putting allied American Indians on the front line of frontier conflict, so they would likely oppose his plan.

The share of White and African American populations in the various states shown in the table most strongly suggests which of the following? A African American migrants were moving from the southern to the northern states. B Small subsistence farms dominated older, more established states such as Virginia and Maryland. C Newly settled states such as Kentucky and Tennessee relied less on slave labor than did regions elsewhere in the South. D The use of slave labor was declining throughout the South by 1800.

C Newly settled states such as Kentucky and Tennessee relied less on slave labor than did regions elsewhere in the South. Fewer African Americans lived in newly established states such as Kentucky and Tennessee in part because White settlers in those states tended to have limited means and did not hold as many slaves as did farmers elsewhere. In addition, the climate and geography of Kentucky and Tennessee were less conducive to large plantations with large numbers of enslaved African Americans than were the climate and geography elsewhere.

"Forces committed to restoring White supremacy launched a ruthless, bloody campaign of terror and intimidation against freedpeople and their White allies in the South. As young southern units of the Republican Party broke under those blows and the Republicans of the North retreated and grew more conservative, Reconstruction collapsed. With it went many . . . gains. A resurgent southern elite once again set about imposing White supremacy and tyrannical labor discipline while stripping freedpeople of many of their civic and political rights." Bruce Levine, historian, The Fall of the House of Dixie, 2013 "For many poor Whites throughout the South, Jim Crow laws alone could not ease their most persistent fear. In regions like northern Louisiana, with little but pine trees rising from its barren soil, White men found themselves competing with [formerly enslaved people], and during the dozen years of Reconstruction they had not known which race would prevail. "Such men had dropped away from the Ku Klux Klan after President Grant's crackdown, but their simmering resentments had grown. With control of the South passing again to the Democrats, powerless Whites were joining plantation owners to ensure that Black workers remained without their basic rights." A. J. Langguth, historian, After Lincoln, 2014 Levine's argument about Reconstruction in the excerpt differs from that of Langguth in that Levine argues A African Americans were held in a perpetual state of slavery B White Southerners willingly worked alongside the emancipated population C Northern Republicans gradually withdrew their support for Reconstruction policies D White plantation owners conceded to Republican policies in the South

C Northern Republicans gradually withdrew their support for Reconstruction policies According to Levine, the Southern Republicans retreated under the campaign of terror and intimidation in the South, while the Northern wing of the Republican Party abandoned Reconstruction and its many gains as the party became more focused on economic development.

"No roads marked the way to the traveler in California then: but, guided by the sun and well-known mountain peaks, we proceeded on our journey. . . . Some forty or fifty men were at work with the cradle machines, and were averaging about eight ounces [of gold] per day to the man. But a few moments passed before I was knee deep in water, with my wash-basin full of dirt, plunging it about endeavoring to separate the dirt from the gold. After washing some fifty pans of dirt, I found I had realized about four bits' worth of gold. Reader, do you know how [one] feels when the gold fever heat has suddenly fallen to about zero? I do. . . . The Indians who were working for Capts. Sutter and Weber gave them leading information, so that they were enabled to know the direction in which new discoveries were to be made. . . . "The morals of the miners of '48 should here be noticed. No person worked on Sunday at digging for gold. . . . We had ministers of the gospel amongst us, but they never preached. Religion had been forgotten, even by its ministers, and instead of their pointing out the narrow way which leads to eternal happiness . . . they might have been seen, with pick-axe and pan, traveling untrodden [untraveled] ways in search of . . . treasure . . . or drinking good health and prosperity with friends." James H. Carson, describing life in the early California gold fields, 1848 Which of the following developments resulted most directly from the gold rush described in the excerpt? A An anti-Catholic movement arose in western mining communities. B Plantation agriculture spread from the South to the Pacific coast. C People from America, Europe, and Asia migrated to the region. D The West Coast became a major industrial center for the United States.

C People from America, Europe, and Asia migrated to the region. The gold rush drew migrants from the United States, Mexico, Europe, and Asia, dramatically increasing the population of California and setting the territory on a rapid path to statehood in the United States.

Which of the following most directly led to the expansion of participatory democracy in the first half of the nineteenth century? A Development of a canal system connecting different parts of the country B Encouragement of citizens to settle the western territories C Reduction of property ownership requirements for voting D Threats by South Carolina to nullify federal laws it deemed unconstitutional

C Reduction of property ownership requirements for voting In the 1820s and 1830s, many states weakened or ended requirements to own property to be eligible to vote, leading to an expansion of participatory democracy, particularly during the presidency of Andrew Jackson.

"In 1739 arrived among us from Ireland the Reverend Mr. Whitefield, who had made himself remarkable there as [a traveling] preacher. He was at first permitted to preach in some of our churches; but the clergy, taking a dislike to him, soon refused him their pulpits, and he was obliged to preach in the fields. The multitudes of all [members of different religious groups] that attended his sermons were enormous, and it was a matter of speculation to me . . . to observe the extraordinary influence of his oratory on his hearers, and how much they admired and respected him. . . . It was wonderful to see the change soon made in the manners of our inhabitants. From being thoughtless or indifferent about religion, it seemed as if all the world were growing religious, so that one could not walk through the town in an evening without hearing psalms sung in different families of every street. "And it being found inconvenient to assemble in the open air, subject to its [harsh conditions], the building of a house to meet in was no sooner proposed . . . and the work [of erecting the building] was carried on with such spirit as to be finished in a much shorter time than could have been expected. Both house and ground were vested in trustees, expressly for the use of any preacher of any religious persuasion who might desire to say something to the people at Philadelphia." Benjamin Franklin, from his autobiography, describing events in 1739 Which of the following most directly contributed to the decision in Philadelphia referenced in the excerpt to build a specific meeting house for the new preachers? A Leaders in cities such as Philadelphia wanted to do all they could to limit the spread of new Protestant denominations in the colony. B Local authorities worried about radical ideas and believed that a central meeting house would make it easier to censor the sermons of the new preachers. C Religious pluralism was more accepted in the middle colonies and particularly in the colony of Pennsylvania than elsewhere. D Commerce was more important than religion to most colonists, so building a small meeting house would keep the new preachers isolated.

C Religious pluralism was more accepted in the middle colonies and particularly in the colony of Pennsylvania than elsewhere. People settling in the middle colonies generally exhibited more toleration for ethnic and religious diversity than did those in either the New England or southern colonies and therefore were more willing to provide support for different religious practices, as shown in the excerpt.

The Fourteenth Amendment emerged from which of the following contexts? A Abolitionists' fears that African Americans would be barred from voting B Expectations that formerly enslaved people would run for political office C Republican concerns that African Americans would be denied citizenship rights D Former Confederate leaders' support for multiple new amendments to the Constitution

C Republican concerns that African Americans would be denied citizenship rights The Fourteenth Amendment specifically grants African Americans citizenship, ensuring their equal protection under the law. It was deemed necessary by Radical Republicans and abolitionists, who feared that Southern white politicians would pass local laws restricting African American rights.

"Whether you are or are not, entitled to all the rights of citizenship in this country has long been a matter of dispute to your prejudice. By enlisting in the service of your country at this trial hour, and upholding the National Flag, you stop the mouths of [cynics] and win applause even from the iron lips of ingratitude. Enlist and you make this your country in common with all other men born in the country or out of it. . . . He who fights the battles of America may claim America as his country—and have that claim respected. Thus in defending your country now against rebels and traitors you are defending your own liberty, honor, manhood and self-respect. . . . . . . [H]istory shall record the names of heroes and martyrs who bravely answered the call of patriotism and Liberty—against traitors, thieves and assassins—let it not be said that in the long list of glory, composed of men of all nations—there appears the name of no colored man." Frederick Douglass, excerpt from an editorial, April 1863 Which of the following best explains Douglass' point of view in the excerpt? A African American enlistment would enable the Union Army to prevail in the Civil War. B Once African American men enlisted, Northern White soldiers would accept them as equals. C Shared sacrifice would help advance African American men's claims to United States citizenship. D Northern politicians overwhelmingly favored enlistment of African Americans in the Union Army.

C Shared sacrifice would help advance African American men's claims to United States citizenship. Douglass argued that by enlisting and fighting in the Union Army, African American men's shared sacrifice and service to their country would help legitimize their claims to freedom and the rights of United States citizenship.

"The committee of the president and directors of the Chesapeake and Delaware Canal Company [in Delaware] . . . beg leave respectfully to offer to the members of the Senate and House of Representatives, the following facts and observations relative to the said canal. . . . ". . . The island of Great Britain furnishes proof of the advantages of canals, beyond any other country. That nation has now become the maritime rival, and almost controller of every commercial people; her superiority has arisen from her unbounded commerce, and the vast wealth it has introduced, the basis of which wealth is her immense manufactures . . . : the foundation of these manufactures has again been formed by her internal improvements. . . . "The United States, both from their present political and natural situation, demand from their government every aid it can furnish. . . . Her rapid increase in prosperity, has already drawn upon her the envy, the jealousy, and the hostility of other nations, which alone can be counteracted by improving her internal strength, supplying her wants as far as possible by her own [products] and manufactures, and extending her agriculture so as to gain from its surplus the wealth of other nations." The Chesapeake and Delaware Canal Company, petition to the United States Congress, 1809 Which of the following best describes a historian's likely interpretation of the situation in which the excerpt was produced in the early 1800s? A Political leaders agreed on policies for developing the economy. B Most Americans sought to turn away from European influences. C Some Americans promoted international strength through a unified national economy. D Business leaders argued that the Constitution limited federal spending.

C Some Americans promoted international strength through a unified national economy. Based on the discussion of the perceived "hostility" of other nations in the third paragraph, a historian would likely interpret the situation in which the excerpt was produced as one in which some Americans promoted measures to unify the national economy in order to project strength internationally.

Which of the following groups would most likely have supported secession from the United States after the 1860 presidential election? A Northern abolitionists B Members of the Free Soil Party C Southern Democrats D Northern Democrats

C Southern Democrats Southern Democrats would have been most likely to support secession from the United States owing to their support for maintaining and expanding slavery, which was in opposition to the Republican goal of limiting slavery. Southern Democrats strongly influenced South Carolina's decision to secede in 1860 following the election of Abraham Lincoln.

"With regard to the northwestern States, to which the ordinance of 1787 was applied—Ohio, Indiana, Illinois, and Michigan—no one now believes that any one of those States, if they thought proper to do it, has not just as much a right to introduce slavery within her borders as Virginia has a right to maintain the existence of slavery within hers. "Then, if in this struggle of power and empire between the two classes of states a decision of California has taken place adverse to the wishes of the southern States, it is a decision not made by the General [federal] Government; it is a decision respecting which they cannot complain to the General Government. It is a decision made by California herself, and which California had incontestably a right to make under the Constitution of the United States. . . . The question of slavery, either of its introduction or interdiction, is silent as respects the action of this [federal] Government; and if it has been decided, it has been by a different body—by a different power—by California herself, who had a right to make that decision." Senator Henry Clay, speech in the United States Senate, 1850 Evidence in the excerpt best corroborates which of the following broader historical contexts? A Expanded trade between the East and West Coasts and with Asia divided the country in new ways. B Abolitionist activity undermined the ability of new territories to achieve statehood. C Southern states sought more proslavery seats in the United States Congress. D New political parties such as the Know-Nothings threatened the system of slavery.

C Southern states sought more proslavery seats in the United States Congress. Southern interests sought new territories that would promote the expansion of slavery, allowing for the addition of proslavery members to the United States Congress when these territories eventually became states. Clay's compromise resolutions reflected this broader conflict between Southern interests and Northern interests over the issue of California.

"What fault has there been on the part of the General Government of the United States? Why break up this Union? Will any gentleman be so kind as to particularize a single instance worthy of debate, in which the Federal Government has been derelict [negligent] in the discharge of its duty, or has failed to accomplish the purposes of its organization? . . . "I am not here . . . to defend the election of Abraham Lincoln. I believe that his election was virtually a fraud upon the people of the United States . . . nominated, as he was, by a sectional party, and upon a sectional platform, with no representation in the body which nominated him from the South; but he was nominated and elected according to the forms of law. . . . "Let us look . . . at the evils that must result from secession. The first, in my opinion, would be that our country would not only be divided into a Northern Confederacy and into Southern Confederacy, but, soon or later it would be divided into sundry [several] petty Confederacies. We would have a Central Confederacy, a Confederacy of the States of the Mississippi Valley, a Pacific Confederacy, a Western Confederacy, an Eastern Confederacy, a Northern and a Southern Confederacy. ". . . It is easy perhaps to break down this Government; but, sir, when we break it down it will not be so easy a matter to build it up. . . . Gentlemen cry out against the tyranny of their own government, and yet denounce [those opposed to secession] because we hesitate to allow ourselves to be thrust into the embraces of such a military dictatorship." Waitman T. Willey, addressing the Virginia State Secession Convention, March 4, 1861 Evidence in the excerpt most strongly suggests which of the following? A Southern opinions regarding slavery were changing. B Southern businesses rejected paying federal taxes. C Southern voters viewed the presidential election with contempt. D Southern politicians were anxious to form alliances with European countries.

C Southern voters viewed the presidential election with contempt. As shown in the excerpt, Southern voters, including Willey, did not vote for Abraham Lincoln in the election of 1860 and often viewed his victory as a fraud. Willey, however, argued that despite their differences, Virginia must not secede from the Union.

"Let us, then, with courage and confidence, pursue our own Federal and [Democratic-] Republican principles, our attachment to union and representative government. Kindly separated by nature and a wide ocean from the exterminating havoc of one quarter of the globe; too high-minded to endure the degradations of the others; possessing a chosen country, with room enough for our descendants . . . ; entertaining a due sense of our equal right to the use of our own faculties, to the acquisitions of our own industry, to honor and confidence from our fellow citizens, resulting not from birth, but from our actions and their sense of them; enlightened by a benign religion . . . —with all these blessings, what more is necessary to make us a happy and a prosperous people? Still one thing more, fellow citizens—a wise and frugal Government, which shall restrain men from injuring one another, shall leave them otherwise free to regulate their own pursuits of industry and improvement, and shall not take from the mouth of labor the bread it has earned. This is the sum of good government; and this is necessary to close the circle of our felicities." President Thomas Jefferson, first inaugural address, 1801 Which of the following best describes the political situation in which Jefferson gave the address in the excerpt? A Federalists sought compromise with Democratic-Republican politicians. B The Federalist Party remained more powerful than the Democratic-Republicans. C The Democratic-Republican Party had won the presidency for the first time. D Democratic-Republicans had gained increased support from wealthy merchants.

C The Democratic-Republican Party had won the presidency for the first time. In 1800, Jefferson became the first Democratic-Republican to be elected president (after twelve years of Federalist administrations) and brought the first transition of executive authority from one party to another in United States history.

Which of the following describes a trend in sectional development by 1860 reflected in the data in the graph? A The South was becoming less dependent on exports to foreign markets than the North. B The North and the South were becoming more connected through railroads. C The North was becoming more diverse in its economic activities than the South. D The North and the South were becoming less reliant on agricultural products.

C The North was becoming more diverse in its economic activities than the South. The North's predominance in railroad mileage, iron and steel production, and number of factories, as indicated in the graph, reflects the trend of economic diversification by 1860.

The data in the tables most likely indicate which of the following? A The Union had more educated leadership in its army than did the Confederacy. B The Confederacy more typically enlisted enslaved African Americans to fill its army. C The Union had a larger and more diverse population of workers to enlist from than did the Confederacy. D The Confederacy had more wealthy people in its armed forces than did the Union.

C The Union had a larger and more diverse population of workers to enlist from than did the Confederacy. The tables indicate that the Union army had a larger number of soldiers and that only 47 percent were farmers. By contrast, the Confederate army had fewer soldiers but over 61 percent were farmers.

"Yes: Mexico must be thoroughly chastised! . . . The news of yesterday [at the southern border] has added the last argument wanted to prove the necessity of an immediate Declaration of War by our government toward its southern neighbor. "We are justified in the face of the world, in having treated Mexico with more forbearance [tolerance] than we have ever yet treated an enemy. . . . We have . . . submitted thus far to a most offensive rejection of an Ambassador personifying the American nation, and waited for years without payment of the claims of our injured merchants. We have sought peace through every avenue, and shut our eyes to many things, which, had they come from England or France, the President would not have dared to pass over without stern and speedy resentment. We have dammed up our memory, of what had passed in the South [Texas] years ago—of devilish massacres of some of our bravest and noblest sons . . . in violation of all the rules of war. . . . "We think there can be no doubt of the truth of yesterday's news; and we are sure the people here, ten to one, are for prompt and hostilities. . . . Let our arms now be carried with a spirit which shall teach the world that, while we are not forward for a quarrel, America knows how to crush, as well as how to expand!" Walt Whitman, journalist and poet, editorial in the Brooklyn Eagle, 1846 "President [James K. Polk] in his message, as a pretext for sending our army to invade and conquer the country upon the Rio Grande, says: "Texas by its [legislative] act of December 19, 1836, had declared the [Rio Grande] to be the boundary of that [formerly independent] republic.' . . . The truth is that Texas had agreed upon the Nueces [River] as her boundary. . . . "If [Mexico] be ours, why does he seek to justify the taking possession of it by references to the fact that Mexico is indebted to some of our people? If it be not ours, and he has taken possession of it in order to compel Mexico to pay those debts, why not say so? The fact that Mexico has not paid the debts due to our citizens can have no legitimate connection with taking possession of [it as] our own soil. But [the president] was obviously conscious that this invasion of the Mexican territory could not be justified. . . . "When the Executive and Congress openly and avowedly took upon themselves the responsibility of extending and perpetuating slavery by the annexation of Texas, and by the total overthrow and subversion of the Constitution, . . . my confidence in the stability of our institutions was shaken, destroyed. . . . Our Union continues, but our Constitution is gone. . . . ". . . No man regards this war as just. We know, the country knows, and the civilized world are conscious, that it has resulted from a desire to extend and sustain an institution on which the curse of the Almighty most visibly rests." Joshua Giddings, congressman from Ohio, speech in the United States House of Representatives, 1846 Based on their arguments in the excerpts, Giddings would likely agree with and Whitman would likely disagree with which of the following claims about the causes of the Mexican-American War? A The United States sought the repayment of Mexican debts. B Mexico had killed Americans during a previous conflict in Texas. C The United States desired to expand slavery to Mexican territory. D Mexico had sent soldiers across the southern border of the United States

C The United States desired to expand slavery to Mexican territory. Giddings would agree that the United States desired to expand slavery to Mexican territory, as claimed in the last paragraph of the second excerpt that the war was intended to "extend" the institution. Whitman would disagree with that claim, as he assigned other motives to the war in the first excerpt.

"The creation of a home market is not only necessary to procure for our agriculture a just reward of its labors, but it is indispensable to obtain a supply of our necessary wants. . . . Suppose no actual abandonment of farming, but, what is most likely, a gradual and imperceptible employment of population in the business of manufacturing, instead of being compelled to resort to agriculture. . . . Is any part of our common country likely to be injured by a transfer of the theatre of [manufacturing] for our own consumption from Europe to America? ". . . Suppose it were even true that Great Britain had abolished all restrictions upon trade, and allowed the freest introduction of the [products] of foreign labor, would that prove it unwise for us to adopt the protecting system? The object of protection is the establishment and perfection of the [manufacturing] arts. In England it, has accomplished its purpose, fulfilled its end. . . . The adoption of the restrictive system, on the part of the United States, by excluding the [products] of foreign labor, would extend the [purchasing] of American [products], unable, in the infancy and unprotected state of the arts, to sustain a competition with foreign fabrics. Let our arts breathe under the shade of protection; let them be perfected as they are in England, and [then] we shall be ready . . . to put aside protection, and enter upon the freest exchanges." Henry Clay, speaker of the House of Representatives, speech in Congress, 1824 Which of the following describes an interpretation of Clay's economic principles at the time as expressed in the excerpt? A The sections of the United States should retain separate regional economies. B The farmers of the United States should switch to manufacturing jobs. C The United States should increase domestic manufacturing to promote prosperity. D The United States should never engage in foreign commerce.

C The United States should increase domestic manufacturing to promote prosperity. Clay's promotion in the excerpt of protecting domestic manufacturing would have been interpreted at the time as representing the principle that the United States should increase domestic manufacturing to increase prosperity. This represented what he called the American System.

"With regard to the northwestern States, to which the ordinance of 1787 was applied—Ohio, Indiana, Illinois, and Michigan—no one now believes that any one of those States, if they thought proper to do it, has not just as much a right to introduce slavery within her borders as Virginia has a right to maintain the existence of slavery within hers. "Then, if in this struggle of power and empire between the two classes of states a decision of California has taken place adverse to the wishes of the southern States, it is a decision not made by the General [federal] Government; it is a decision respecting which they cannot complain to the General Government. It is a decision made by California herself, and which California had incontestably a right to make under the Constitution of the United States. . . . The question of slavery, either of its introduction or interdiction, is silent as respects the action of this [federal] Government; and if it has been decided, it has been by a different body—by a different power—by California herself, who had a right to make that decision." Senator Henry Clay, speech in the United States Senate, 1850 The position expressed by Clay in the excerpt best serves as evidence of which of the following? A Northern politicians showed less interest in California than Southern politicians. B The United States Senate could not agree on treaty terms with Mexico. C The acquisition of new territories created disputes over the expansion of slavery. D Nativist groups opposed incorporating new states into the Union.

C The acquisition of new territories created disputes over the expansion of slavery. In the excerpt, Clay is concerned that the issue of slavery in the new territories would further divide the country along sectional lines. The issue had been temporarily settled with the Missouri Compromise in 1820, but the problem resurfaced when the United States acquired new territory as a result of the Mexican-American War.

Which of the following developments most likely influenced the argument expressed in the image? A The Mexican-American War resulted in the expansion of United States territories. B The continued westward movement of settlers increased agricultural production. C The conclusion of the Civil War stirred debates over citizenship. D The invention of new sailing technologies made international trade easier.

C The conclusion of the Civil War stirred debates over citizenship. Through its depiction of people from around the world gathering at a Thanksgiving table, the image indicates the artist's support for the expansion of citizenship rights in the period immediately following the Civil War.

"I have already intimated [warned] to you the danger of parties in the State, with particular reference to the founding of them on geographical discriminations. Let me now take a more comprehensive view, and warn you in the most solemn manner against the baneful effects of the spirit of party generally. . . . The alternate domination of one faction over another, sharpened by the spirit of revenge, natural to party dissension . . . is itself a frightful despotism. . . . "The great rule of conduct for us in regard to foreign nations is in extending our commercial relations, to have with them as little political connection as possible. So far as we have already formed engagements, let them be fulfilled with perfect good faith. Here let us stop. . . . Taking care always to keep ourselves by suitable establishments on a respectable defensive posture, we may safely trust to temporary alliances for extraordinary emergencies. "Harmony, liberal intercourse with all nations, are recommended by policy, humanity, and interest. But even our commercial policy should hold an equal and impartial hand; neither seeking nor granting exclusive favors or preferences; consulting the natural course of things; diffusing and diversifying by gentle means the streams of commerce, but forcing nothing." George Washington, Farewell Address, 1796 George Washington's suggestions about United States foreign relations reflect which of the following situations? A The need to establish industries to export more goods B The desire to encourage immigration from northern and western Europe C The continued European colonial presence along United States borders D The challenge of spreading American revolutionary ideals abroad

C The continued European colonial presence along United States borders The excerpt reflects the continued presence of the British empire to the west and north of the current United States borders and of the Spanish empire to the south and southwest, as well as Washington's efforts to avoid conflicts with them.

"National gratitude—national pride—every high and generous feeling that attaches us to the land of our birth, or that [elevates] our characters as individuals, ask[s] of us that we should foster the . . . literature of our country. . . . On the other hand, it is not necessary for these purposes—it is even detrimental to bestow on mediocrity the praise due to excellence, and still more so is the attempt to persuade ourselves and others into an admiration of the faults of [our writers]. . . . "It must however be allowed, that the poetry of the United States, though it has not reached that perfection to which some other countries have carried theirs, is yet even better than it could have been expected to produce, considering that our nation has scarcely seen two centuries since its founders erected their cabins on its soil. . . . "The fondness for literature is fast increasing in our country—and if this were not the case, the patrons of literature have multiplied, of course, and will continue to multiply with the mere growth of our population. The popular English works of the day are often reprinted in our country—they are dispersed all over the union. . . . What should hinder our native works, if equal in merit, from meeting an equally favorable reception?" William Cullen Bryant, book review in the North American Review, 1818 The excerpt best serves as evidence of which of the following developments? A The termination of cultural connections with Great Britain B The popularization of the belief in human perfectibility C The creation of a unique American culture D The foundation of a trans-Atlantic print culture

C The creation of a unique American culture The excerpt highlights the increasing skill of United States authors and the desire to popularize United States literature, which provides evidence of the beginning of an American national culture.

Which of the following most likely contributed to the emergence of the Second Great Awakening? A The rise of the Whig Party B The election of President Andrew Jackson C The cultural responses to the Enlightenment D The emergence of a women's rights movement

C The cultural responses to the Enlightenment In its emphasis on personal salvation and emotion, the Second Great Awakening was a response or challenge to the ideals of rationalism, which were central to Enlightenment thought and its influence.

"Your Memorialist . . . represents to your honorable body, that he has devoted much time and attention to the subject of a railroad from Lake Michigan through the Rocky Mountains to the Pacific Ocean, and that he finds such a route practicable, the results from which would be incalculable—far beyond the imagination of man to estimate. . . . "It would enable us, in the short space of eight days (and perhaps less) to concentrate all the forces of our vast country at any point from Maine to Oregon. . . . Such easy and rapid communication with such facilities for exchanging the different products of the different parts would bring all our immensely wide spread population together. . . . "[W]ith a railroad to the Pacific, and thence to China by steamers, can be performed in thirty days, being now a distance of nearly seventeen thousand miles. . . Then the drills and sheetings of Connecticut, Rhode Island, and Massachusetts, and other manufactures of the United States, may be transported to China in thirty days; and the teas and rich silks of China, in exchange, come back to New Orleans, to Charleston, to Washington, to Baltimore, to Philadelphia, New York, and to Boston, in thirty days more." Asa Whitney, merchant, "National Railroad, Connecting the Atlantic and Pacific Ocean," memorial to the Senate and House of Representatives of the United States, 1845 Which of the following most directly contributed to the request expressed in the excerpt? A The removal of Native Americans east of the Mississippi B The passage of legislation ending the international slave trade C The desire for international trade and access to global markets D The expansion of westward settlement following the Civil War

C The desire for international trade and access to global markets The excerpt shows a proposal for the initiation of trade and market access between the United States and Asia through the creation of a westward path connecting the Atlantic coast to East Asia.

Which of the following best explains the spread of images of George Washington in the United States after the American Revolution? A The republican belief that political power should be inherited B The embrace of political party campaigning by electoral candidates C The desire of Americans to develop a new national culture D The refusal of former Loyalists to accept the outcome of the war

C The desire of Americans to develop a new national culture Following the American Revolution, many Americans desired to find ways to break away from dependence on British culture and to develop a new American national culture. The image and example of George Washington were used in such projects to define a new American nationalism.

The artist who created the image would have most likely opposed which of the following developments? A The creation of the Republican Party B The construction of canals and railroads C The ending of Reconstruction D The passage of laws promoting economic development in the West

C The ending of Reconstruction Because the end of Reconstruction resulted in the undoing of many policies advocated in the image such as the protection of civil rights, the artist most likely would have disapproved of that development.

Which of the following best explains the expansion of participatory democracy in the early nineteenth century? A The abolition of slavery in most northern states B The growth of manufacturing along rivers and canals C The extension of suffrage rights to most adult White men D The influence of the early women's rights movement

C The extension of suffrage rights to most adult White men The reduction of property requirements for White male suffrage resulted in increased voter participation and, thus, increased participation in political party activity.

"The committee of the president and directors of the Chesapeake and Delaware Canal Company [in Delaware] . . . beg leave respectfully to offer to the members of the Senate and House of Representatives, the following facts and observations relative to the said canal. . . . ". . . The island of Great Britain furnishes proof of the advantages of canals, beyond any other country. That nation has now become the maritime rival, and almost controller of every commercial people; her superiority has arisen from her unbounded commerce, and the vast wealth it has introduced, the basis of which wealth is her immense manufactures . . . : the foundation of these manufactures has again been formed by her internal improvements. . . . "The United States, both from their present political and natural situation, demand from their government every aid it can furnish. . . . Her rapid increase in prosperity, has already drawn upon her the envy, the jealousy, and the hostility of other nations, which alone can be counteracted by improving her internal strength, supplying her wants as far as possible by her own [products] and manufactures, and extending her agriculture so as to gain from its surplus the wealth of other nations." The Chesapeake and Delaware Canal Company, petition to the United States Congress, 1809 The petition could best be used as evidence by historians studying which of the following? A The effects of new transportation links on industry in the Northeast B The lives of immigrant laborers constructing new infrastructure C The ideas that led some Americans to advocate for improved transportation D The reasons that regional interests opposed internal improvements

C The ideas that led some Americans to advocate for improved transportation The discussion by the leaders of a canal company of the reasons to support canal construction could best be used by historians studying the ideas that led some Americans to advocate for improved transportation.

"The Declaration of Independence, drawn up by the Continental Congress, was actually a declaration by 'thirteen united States of America' proclaiming that as 'Free and Independent States they have full power to levy war, conclude peace, contract alliances, establish commerce, and to do all other acts and things which independent States may of right do.' And the Articles of Confederation, for all the powers it theoretically gave to the Congress, did not in fact alter this independence. . . . Congressional resolutions continued to be mere recommendations which the states were left to enforce. . . . The Confederation was intended to be, and remained, a Confederation of sovereign states." Gordon S. Wood, historian, The Creation of the American Republic, 1776-1787, published in 1969 The failure of the Articles of Confederation resulted in which of the following changes in the government of the United States? A The immediate end to property qualifications for White male suffrage B The ban on women's political participation under the ideas of republican motherhood C The ratification of a constitution that established a stronger central government D The enactment of the Northwest Ordinance, which outlined the admission process for new states

C The ratification of a constitution that established a stronger central government The ratification of the United States Constitution in 1787 led to the replacement of the Articles of Confederation and established a stronger central government that limited the sovereignty of the individual states.

Which of the following best explains a major reason for the emergence of the Second Great Awakening in the United States? A The expansion of participatory democracy fostered popular engagement in religion as well. B The arrival of large numbers of immigrants from Europe brought new religious ideas to the United States. C The rise of individualistic and evangelical spiritual beliefs inspired religious conversion. D The proclamation of the Monroe Doctrine in the Western Hemisphere created a sense of spiritual independence.

C The rise of individualistic and evangelical spiritual beliefs inspired religious conversion. In the early decades of United States independence, ideas of human equality with the divine promoted new egalitarian religious views. Many itinerant ministers engaged in evangelical preaching to attract new religious believers of any background or class.

"Louisiana as ceded by France is made part of the United States. Congress may make part of the United States other adjacent territories which shall be justly acquired. "Congress may sever from the United States territory not heretofore within the United States, with consent of a majority of the free males above 21 years, inhabiting such territory." James Madison, secretary of state, proposed constitutional amendment [not passed], 1803 Which of the following best describes a major purpose of the proposed amendment? A To strengthen the United States alliance with France B To give individual states the legal right to secede from the Union C To give Congress additional constitutional powers D To give the president more control over negotiating treaties

C To give Congress additional constitutional powers The proposed amendment was designed to add specific language to the Constitution regarding the powers of the federal government, particularly the legislative branch, to buy territory from foreign governments and incorporate new territory into the Union, because these powers were not explicitly stated.

Which of the following explains the most likely reason why English colonists wanted to come to North America?

C To seek economic opportunity and improved living conditions English colonists came to North America seeking social mobility, economic opportunity, religious freedom, and improved living conditions from what they experienced in England.

"To the Commanders of armed vessels belonging to the United States: "WHEREAS it is declared by the act entitled 'An act for the protection of the commerce and seamen of the United States, against the Tripolitan cruisers,' That it shall be lawful fully to equip, officer, man, and employ such of the armed vessels of the United States, as may be judged requisite by the President of the United States, for protecting effectually the commerce and seamen thereof, on the Atlantic ocean, the Mediterranean and adjoining seas: and also, that it shall be lawful for the President of the United States to instruct the commanders of the respective public vessels, to subdue, seize, and make prize, of all vessels, goods, and effects, belonging to the Bey [Sultan] of Tripoli [in North Africa], or to his subjects. "THEREFORE, And in pursuance of the said statute, you are hereby authorized and directed to subdue, seize, and make prize, of all vessels, goods, and effects, belonging to the Bey of Tripoli, or to his subjects, and to bring or send the same into port, to be proceeded against and distributed according to law. "By command of the President of the United States of America." Thomas Jefferson, 1802 The rhetorical purpose expressed in the excerpt would most likely have been interpreted as promoting which of the following? A Expanding suffrage rights to all White men B Challenging Great Britain's control of Canada C Using international commerce to expand United States influence D Investing in technology to make ocean voyages faster

C Using international commerce to expand United States influence The excerpt clearly outlines the desire of the United States to secure trade carried out by its citizens, with the intended impact of strengthening its influence abroad.

"The petition of a great number of Blacks detained in a state of slavery in the bowels of a free and Christian country humbly showeth that your petitioners apprehend that they have in common with all other men a natural and inalienable right to that freedom which the Great Parent of the Universe hath bestowed equally on all mankind. . . . They were unjustly dragged by the hand of cruel power . . . from a populous, pleasant, and plentiful country and in violation of laws of nature and of nations. . . . ". . . Your petitioners . . . cannot but express their astonishment that it has never been considered that every principle from which America has acted in the course of their unhappy difficulties with Great Britain pleads stronger than a thousand arguments in favor of your petitioners. They therefore humble beseech your honors to give this petition its due weight and consideration and cause an act of the legislature to be passed whereby they may be restored to the enjoyments of that which is the natural right of all men—and their children who were born in this land of liberty may not be held as slaves after they arrive at the age of twenty one years." Petition to the Massachusetts state legislature, 1777 The first paragraph of the excerpt makes the claim that A slavery should encourage religious conversion to Christianity B concepts of independence should apply only to White men C all people should have the same inherent liberties D the ideals of the American Revolution should also apply to other nations

C all people should have the same inherent liberties The petitioners state in the excerpt that they "have in common with all other men a natural and inalienable right to that freedom which the Great Parent of the Universe hath bestowed equally on all mankind." This suggests that Black and White people share the same natural rights.

The White and Black populations in New England shown in the table most directly suggest the A movement of indentured servants from the South to New England B expansion of plantation agriculture to New England C minimal use of enslaved labor in New England compared to other regions D spread of trans-Atlantic print culture among Puritans in New England

C minimal use of enslaved labor in New England compared to other regions The table shows that the White population in New England surpassed the Black population by a large margin. This was because the use of enslaved labor in the region remained limited and would eventually cease following the American Revolution.

"It was not automatically apparent how any of the filibustering targets of the post-1848 period could 'fit' into an American republic, or even into an American empire. . . . While it seemed only logical to some to simply take all of Mexico as booty [spoils] of the war, cut Mexico up, and turn it into new territories and states, most Americans rejected this idea. They did so because central Mexico was densely populated. . . . Many Americans feared the result of the integration of Mexico's people into the United States. Critics also doubted whether Americans could be happy in the alien landscape of central and southern Mexico." Amy Greenberg, historian, Manifest Manhood and the Antebellum American Empire, 2005 "American settlers had eclipsed the Mexicans in Texas and, with ample aid from southern Whites, had rebelled and won their independence. . . . A small band of Americans, many of them merchants, lived in Mexican California when war broke out in 1846. This dispersion of hardy migrants inspired observers to insist that pioneers and not politicians won the West. . . . "Pioneers played a role in expansion, but the historical record points to politicians and propagandists as the primary agents of empire. Racial, economic, social, and political factors coalesced [combined] to make territorial and commercial expansion enticing to American leaders. . . . "Denying any parallels between earlier empires and their own, expansionists insisted that democracy and dominion were complementary, not contradictory. Since leaders intended to transform [territorial] cessions into states and their inhabitants (at least Whites) into citizens, they scoffed at misgivings about governing a vast domain." Thomas Hietala, historian, Manifest Design: American Exceptionalism and Empire, 2003 Greenberg's argument most differs from Hietala's in that Greenberg claims that A race was a defining factor in the tensions leading up to the Mexican-American War B pioneers, not politicians, were a major factor in building the American empire C most Americans believed that Mexicans in the new territories could not assimilate D the granting of citizenship to people in the territories was welcomed by many Americans after the war

C most Americans believed that Mexicans in the new territories could not assimilate According to Greenberg's excerpt, many Americans opposed Mexican integration because they were fearful of the influence of foreign cultures and believed that assimilation would be difficult or impossible.

"Louisiana as ceded by France is made part of the United States. Congress may make part of the United States other adjacent territories which shall be justly acquired. "Congress may sever from the United States territory not heretofore within the United States, with consent of a majority of the free males above 21 years, inhabiting such territory." James Madison, secretary of state, proposed constitutional amendment [not passed], 1803 Which of the following groups would mostly likely have supported this proposed amendment? A Proponents of the supremacy of federal laws over state laws B Proponents of the expansion of the executive branch powers under the Constitution C Advocates of the power of the Supreme Court to review federal laws D Advocates of limiting federal powers to those specifically written into the Constitution

D Advocates of limiting federal powers to those specifically written into the Constitution This amendment was proposed by the Jeffersonians, those who emphasized a strict interpretation of the Constitution and who believed the power of the federal government to purchase and add territory to the Union needed to be specified in the Constitution in order for the Louisiana Purchase to take place.

The Second Great Awakening was most directly related to which of the following other historical developments of the early nineteenth century? A Innovations in manufacturing technology and agricultural equipment B Opposing political views on the powers of the federal government C Celebrations of American patriotism and national culture D Challenges to Enlightenment views of rationalism

D Challenges to Enlightenment views of rationalism The religious revivalism of the Second Great Awakening challenged existing Enlightenment views of rationalism, which sought to teach reason (rather than spiritual growth) as the single greatest purpose of having knowledge.

Which of the following factors best explains the increase in White male suffrage in the early nineteenth century? A Reaction to widespread political protest B Resistance to increased federal taxation C Amendments to the United States Constitution D Changes to property ownership requirements

D Changes to property ownership requirements The elimination of property ownership requirements by states throughout the nineteenth century resulted in most White men being able to vote.

The data in the tables indicate which of the following? A Confederate forces suffered more casualties than did Union forces. B The Confederacy lost more of its skilled workers than did the Union. C The Union lost a greater percentage of its population than did the Confederacy. D Confederate armies lost a greater proportion of its soldiers than did the Union.

D Confederate armies lost a greater proportion of its soldiers than did the Union. Although the Confederacy lost fewer soldiers, it lost almost 40 percent of its total forces, whereas the Union lost closer to 25 percent.

"National gratitude—national pride—every high and generous feeling that attaches us to the land of our birth, or that [elevates] our characters as individuals, ask[s] of us that we should foster the . . . literature of our country. . . . On the other hand, it is not necessary for these purposes—it is even detrimental to bestow on mediocrity the praise due to excellence, and still more so is the attempt to persuade ourselves and others into an admiration of the faults of [our writers]. . . . "It must however be allowed, that the poetry of the United States, though it has not reached that perfection to which some other countries have carried theirs, is yet even better than it could have been expected to produce, considering that our nation has scarcely seen two centuries since its founders erected their cabins on its soil. . . . "The fondness for literature is fast increasing in our country—and if this were not the case, the patrons of literature have multiplied, of course, and will continue to multiply with the mere growth of our population. The popular English works of the day are often reprinted in our country—they are dispersed all over the union. . . . What should hinder our native works, if equal in merit, from meeting an equally favorable reception?" William Cullen Bryant, book review in the North American Review, 1818 Which of the following can be concluded about the relationship between the United States and Europe based on the situation described in the excerpt? A American writing was considered more refined than European writing. B European artists traveled to the United States for inspiration. C Most American literature was written for European consumption. D European styles continued to influence American society.

D European styles continued to influence American society. The excerpt describes how books from England continued to be published and consumed across the United States.

Mr. President, it was solemnly asserted on this floor, some time ago, that all parties in the non-slaveholding States had come to a fixed and solemn determination upon two propositions. One was that there should be no further admission of any States into this Union which permitted, by their constitutions, the existence of slavery; and the other was that slavery shall not hereafter exist in any of the territories of the United States, the effect of which would be to give to the non-slaveholding States the monopoly of the public domain. . . . The subject has been agitated in the other House [of Congress], and they have sent up a bill 'prohibiting the extension of slavery . . . to any territory which may be acquired by the United States hereafter.' At the same time, two resolutions which have been moved to extend the compromise line from the Rocky Mountains to the Pacific, during the present session, have been rejected by a decided majority. "Sir, there is no mistaking the signs of the times; and it is high time that the Southern States—the slaveholding States—should inquire what is now their relative strength in this Union, and what it will be if this determination is carried into effect hereafter." John C. Calhoun, senator, speech in the United States Senate, 1847 The excerpt best provides evidence about which of the following historical situations in the late 1840s? A Increased conflict between urban immigrants and nativists B Compromises between the Supreme Court and national leaders C The expanding influence of the Northern abolitionist movement D Growing sectional tensions caused by the Mexican-American War

D Growing sectional tensions caused by the Mexican-American War The aftermath of the Mexican-American War led to heated controversies over whether to allow slavery in the newly acquired territories, which increased sectional tensions between the North and the South.

"There are those who are dissatisfied with me. To such I would say: You desire peace; and you blame me that we do not have it. But how can we attain it? . . . "But to be plain, you are dissatisfied with me about the Negro. . . . You dislike the emancipation proclamation; and, perhaps, would have it retracted. You say it is unconstitutional—I think differently. I think the Constitution invests its commander-in-chief, with the law of war, in time of war. The most that can be said, if so much, is, that slaves are property. Is there—has there ever been—any question that by the law of war, property, both of enemies and friends, may be taken when needed? And is it not needed whenever taking it, helps us, or hurts the enemy? . . . "You say you will not fight to free Negroes. Some of them seem willing to fight for you. . . . I issued the proclamation on purpose to aid you in saving the Union. . . . Why should they do anything for us, if we will do nothing for them? If they stake their lives for us, they must be prompted by the strongest motive—even the promise of freedom. And the promise being made, must be kept." President Abraham Lincoln, letter to James Conkling explaining why he issued the Emancipation Proclamation, 1863 The phrase in the excerpt "Some of them seem willing to fight for you" could most likely be interpreted as having which of the following purposes? A Authorizing the policy of total war by Union generals in fighting in the South B Addressing the difficulties of feeding runaway enslaved people at military encampments C Acknowledging the shuffling of commanding officers in the Union army D Highlighting the enlistment of formerly enslaved people into the Union army

D Highlighting the enlistment of formerly enslaved people into the Union army With the Emancipation Proclamation, Lincoln sought to redefine the purpose of the Civil War. Lincoln defended the proclamation's aims and supported the enlistment of tens of thousands of African Americans into the Union army.

The United States Supreme Court's decision in Dred Scott v. Sandford (1857) represented a departure from earlier practices in which of the following ways? A It established the principle of judicial review, the practice in which the Supreme Court has the authority to evaluate whether laws are consistent with the Constitution. B It introduced the idea of popular sovereignty, the arrangement in which residents of a federal territory could vote whether that territory would allow slavery. C It expanded fugitive slave laws that permitted slaveholders to recover formerly enslaved people even if they had escaped to free states. D It held that the federal government had no power to regulate slavery in federal territories acquired after the creation of the United States.

D It held that the federal government had no power to regulate slavery in federal territories acquired after the creation of the United States. The decision in the Dred Scott case that the federal government could not regulate slavery in federal territories overturned previous approaches to the issue established by the Missouri Compromise in 1820 and the Kansas-Nebraska Act in 1854.

The growth of manufacturing in the United States from 1800 to 1850 was most directly connected to which of the following broader historical processes? A Abolitionists advancing moral arguments to bring an end to slavery B Political parties debating about the need for a national bank C Supreme Court decisions confirming the primacy of federal laws over state laws D Large numbers of international migrants moving to northern cities

D Large numbers of international migrants moving to northern cities The growth of manufacturing in the northern states coincided with large numbers of international migrants settling in northern cities and urban areas.

"I have already intimated [warned] to you the danger of parties in the State, with particular reference to the founding of them on geographical discriminations. Let me now take a more comprehensive view, and warn you in the most solemn manner against the baneful effects of the spirit of party generally. . . . The alternate domination of one faction over another, sharpened by the spirit of revenge, natural to party dissension . . . is itself a frightful despotism. . . . "The great rule of conduct for us in regard to foreign nations is in extending our commercial relations, to have with them as little political connection as possible. So far as we have already formed engagements, let them be fulfilled with perfect good faith. Here let us stop. . . . Taking care always to keep ourselves by suitable establishments on a respectable defensive posture, we may safely trust to temporary alliances for extraordinary emergencies. "Harmony, liberal intercourse with all nations, are recommended by policy, humanity, and interest. But even our commercial policy should hold an equal and impartial hand; neither seeking nor granting exclusive favors or preferences; consulting the natural course of things; diffusing and diversifying by gentle means the streams of commerce, but forcing nothing." George Washington, Farewell Address, 1796 Supporters of Washington's comments would most likely have agreed with which of the following foreign policies? A Forming a large military B Expanding territorial control C Establishing mutual defense alliances D Maintaining economic relationships

D Maintaining economic relationships Washington argues that maintaining good economic relationships is the best foreign policy for the United States.

"The laity [church members] . . . saw to it that the Second Great Awakening exerted much of its influence through purposeful voluntary associations, typically headed by boards of directors on which laypersons appeared prominently. . . . "Contemporaries called the interlocking, interdenominational directorates of these organizations "the Evangelical United Front" or "the Benevolent Empire." . . . "The social reforms embraced by the Evangelical United Front characteristically involved creating some form of personal discipline serving a goal or redemption. Prison reform serves as an example: No longer would the prison be intended only as a place to hold persons awaiting trial, coerce debt payment, or inflict retributive justice. Reformers reconceived the prison as corrective function, as a 'penitentiary' or 'reformatory,' in the vocabulary they invented. Besides prisoners, other people who did not function as free moral agents might become objects of the reformers' concern: alcoholics, children, slaves, the insane. The goal of the reformers in each case was to substitute for external constraints the inner discipline of morality. Some historians have interpreted the religious reformers as motivated simply by an impulse to impose 'social control,' but it seems more accurate to describe their concern as redemptive, and more specifically the creation of responsible personal autonomy. Liberation and control represented two sides of the redemptive process as they conceived it. Christians who had achieved self-liberation and self-control through conversion not surprisingly often turned to a concern with the liberation and discipline of others. . . . "The religious awakenings of the early nineteenth century marshaled powerful energies in an age when few other social agencies in the United States had the capacity to do so. [The] Evangelical United Front organized its voluntary associations on a national, indeed international, level, at a time when little else in American society was organized, when there existed no nationwide business corporation save the Second Bank of the United States and no nationwide government bureaucracy save the Post Office. Indeed, the four major evangelical denominations together employed twice as many people, occupied twice as many premises, and raised at least three times as much money as the Post Office." Daniel Walker Howe, historian, What Hath God Wrought: The Transformation of America, 1815-1848, published in 2007 Which of the following is a piece of evidence used by Howe to support his claim in the third paragraph of the excerpt about religious organizations in the early nineteenth century? A Religious voluntary associations were limited to focusing on local efforts. B Many religious reformers also worked for large nationwide corporations. C The Second Bank of the United States was one of many national federal organizations. D Members of the Evangelical United Front employed more people than the Post Office did.

D Members of the Evangelical United Front employed more people than the Post Office did. In the third paragraph of the excerpt, Howe provides evidence that members of the Evangelical United Front employed more people than the Post Office did in order to support his claim that the Evangelical United Front was more extensively organized than the federal government was in the early nineteenth century.

English colonists in North America in the 1600s and 1700s most typically sought which of the following? A Chances to spread Christianity to Native Americans B Jobs working in factories C Political independence from England D Opportunities to improve their living conditions

D Opportunities to improve their living conditions English colonists came to North America for many reasons, but most common among these was to find economic opportunities and social mobility.

"The American Republicans of the city and county of Philadelphia, who are determined to support the NATIVE [White, Protestant] AMERICANS in their Constitutional Rights of peaceably assembling to express their opinions on any question of Public Policy, and to SUSTAIN THEM AGAINST THE ASSAULTS OF ALIENS AND FOREIGNERS are requested to assemble on MONDAY AFTERNOON, May 6th, 1844 at 4 o'clock, at the corner of Master and Second street, Kensington [a section of Philadelphia], to express their indignation [anger] at the outrage on Friday evening last, which was perpetrated by the Irish Catholics." Text from a poster announcing a meeting of the American Republican Party, later renamed the American Party, Philadelphia, 1844 Historians could best use the excerpt as an example of which of the following? A Expansion of voting rights during the Jackson administration B Debates over the separation of church and state in the United States C Advocacy for policies promoting the idea of Manifest Destiny D Political responses to changing demographics in the United States

D Political responses to changing demographics in the United States The excerpt depicts efforts by nativist groups who advocated for the primacy of White Protestantism to assemble to call for the expulsion of Catholics from their communities and changes in immigration laws to exclude Catholics.

"I am filled with deep emotion at finding myself standing here in the place . . . from which sprang the institutions under which we live. . . . I have never had a feeling politically that did not spring from the sentiments embodied in the Declaration of Independence. . . . It was not the mere matter of the separation of the colonies from the mother land; but something in that Declaration giving liberty, not alone to the people of this country, but hope to the world for all future time. It was that which gave promise that in due time the weights should be lifted from the shoulders of all men. . . . "Now, my friends, can this country be saved upon that basis? If it can, I will consider myself one of the happiest men in the world if I can help to save it. If it can't be saved upon that principle, it will be truly awful. "Now, in my view of the present aspect of affairs, there need be no bloodshed and war. . . . And I may say in advance, there will be no blood shed unless it be forced upon the Government. . . . "My friends, this is a wholly unprepared speech. I did not expect to be called upon to say a word when I came here. . . . I may, therefore, have said something indiscreet, but I have said nothing but what I am willing to live by, and, in the pleasure of Almighty God, die by." President-elect Abraham Lincoln, speaking at Independence Hall in Philadelphia, February 22, 1861 The excerpt most likely reflects which of the following historical situations? A Abraham Lincoln won all of the electoral college votes in the presidential election. B Formerly enslaved people were given the right to vote in presidential elections. C Southern states refused to participate in the presidential election. D States in the South had begun seceding after the presidential election.

D States in the South had begun seceding after the presidential election. The excerpt shows Lincoln's attempts to minimize the political impact of seven Southern states declaring their secession from the United States following his election between December 1860 and February 1861.

The table most directly suggests which of the following developments by 1749 ? A Native Americans refused to purchase British goods in order to achieve self-sufficiency. B Plantation owners began to rely on indentured servants to produce valuable cash crops. C Colonists became increasingly vulnerable to the transmission of epidemic diseases such as smallpox. D The British established increasingly extensive trade networks to provide goods to its colonies.

D The British established increasingly extensive trade networks to provide goods to its colonies. In order to gather sources of raw material and to expand markets for their products, English merchants extended their trade with British American colonists and with Native Americans across North America.

"The United States [under the Articles of Confederation] has an indefinite discretion to make [requests] for men and money; but they have no authority to raise either, by regulations extending to the individual citizens of America. The consequence of this is, that though in theory their resolutions concerning those objects are laws, constitutionally binding on the members of the Union, yet in practice they are mere recommendations which the States observe or disregard at their option. "There is nothing absurd or impracticable in the idea of a league or alliance between independent nations for certain defined purposes . . . depending for its execution on the good faith of the parties. . . . In the early part of the present century there was an [enthusiasm] in Europe for [leagues or alliances]. . . . They were scarcely formed before they were broken, giving an instructive but afflicting lesson to mankind, how little dependence is to be placed on treaties which have no other sanction than the obligations of good faith. . . . "There was a time when we were told that breaches, by the States, of the regulations of the [Confederation's] authority were not be expected. . . . "In our case, the [agreement] of thirteen distinct sovereign wills is requisite, under the Confederation, to the complete execution of every important measure that proceeds from the Union. . . . The measures of the Union have not been executed. . . . Each State, yielding to the persuasive voice of immediate interest or convenience, has successively withdrawn its support." Alexander Hamilton, The Federalist paper number 15, published in 1787 Which of the following overall arguments did Hamilton make about the Articles of Confederation? A The United States should invite other countries to ally with it under the terms of the Articles. B Americans should adopt the Articles as the constitution of the United States. C Americans should donate money to Congress because of flaws in the Articles. D The United States should abandon the Articles to form a stronger central government.

D The United States should abandon the Articles to form a stronger central government. Hamilton's overall argument in Federalist 15 is that since the national government could not compel state support, the United States should replace the Articles of Confederation with a constitution that provided a stronger central government.

"Not far from this time Nat Turner's insurrection [a slave rebellion] broke out; and the news threw our town into great commotion. . . . "It was always the custom to have a muster every year. On that occasion every White man shouldered his musket. The citizens and the so-called country gentlemen wore military uniforms. . . . "I knew the houses were to be searched; and I expected it would be done by country bullies and the poor Whites. . . . "It was a grand opportunity for the low Whites, who had no Negroes of their own to scourge. They exulted in such a chance to exercise a little brief authority, and show their subserviency to the slaveholders; not reflecting that the power which trampled on the colored people also kept themselves in poverty, ignorance, and moral degradation. . . . Colored people and slaves who lived in remote parts of the town suffered in an especial manner. In some cases the searchers scattered [gun]powder and shot among their clothes, and then sent other parties to find them, and bring them forward as proof that they were plotting insurrection." Harriet Ann Jacobs, Incidents in the Life of a Slave Girl, published in 1861, describing events earlier in the nineteenth century The excerpt best reflects which of the following developments? A The establishment of tariffs on textiles to protect domestic cotton production B The emergence of an abolitionist movement during the Second Great Awakening C The increase of prosperity caused by the production of manufactured goods D The development of regional cultures increasingly defined by the presence of enslaved people

D The development of regional cultures increasingly defined by the presence of enslaved people During the first half of the nineteenth century, southern culture became increasingly defined by the defense of slavery.

The industrial resources of the North during the Civil War most likely accounted for which of the following? A The Southern reliance on cotton production and export, along with the system of enslaved labor B The Confederate strategy that focused on the western theater of operations during the war C The willingness of African Americans to fight for the Union cause in the conflict D The disadvantage of the Confederacy in access to arms, munitions, and other supplies

D The disadvantage of the Confederacy in access to arms, munitions, and other supplies The industrialization of the North allowed for greater production of war materials for the Union army. In contrast, the Southern economy was more agrarian and did not have the resources to mobilize and adequately supply the Confederate army.

What was a major difference between the Spanish encomienda system and the Spanish caste system in the Americas?

D The encomienda system was based on using Native Americans for forced labor, while the caste system was based on a diverse and racially mixed population. The encomienda system was developed by the Spanish based on their experience in Europe of deriving forced labor from the indigenous population. The caste system developed after extensive interracial and ethnic mixing took place as a way to maintain Spanish control over a diverse population.

"I know not how to thank you for the deep and lively interest you have been pleased to take in the cause of . . . the emancipation of a people, who, for two long centuries, have endured, with the utmost patience, a bondage, one hour of which . . . is worse than ages of that which your fathers rose in rebellion to oppose. "It is such indications on the part of the press—which, happily, are multiplying throughout all the land—that kindle up within me an ardent hope that the curse of slavery will not much longer be permitted to make its iron foot-prints in the lacerated [deeply cut] hearts of my . . . brethren. . . . I am called, by way of reproach, a runaway slave. As if it were a crime—an unpardonable crime—for a man to take his inalienable rights! "But why [you,] a New-York editor, born and reared in the State of Maine, far removed from the contaminated . . . atmosphere of slavery, should pursue such a course [supporting abolition], is not so apparent. I will not, however, stop here to ascertain the cause, but deal with fact. . . . "The object . . . is simply to give such an exposition of the degrading influence of slavery upon the master and his [supporters] as well as upon the slave—to excite such an intelligent interest on the subject of American slavery—as may react upon that country, and tend to shame her out of her adhesion to a system which all must confess to disagree with justice. . . . "I am earnestly and anxiously laboring to wipe off this foul blot from the . . . American people, that they may accomplish in behalf of human freedom that which their exalted position among the nations of the earth amply fits them to do." Frederick Douglass to New York Tribune editor Horace Greeley, 1846 The excerpt could best be used by historians studying which of the following? A The development of new transportation technologies B The emergence of nativist political parties C The relocation of Native Americans from the South D The growth of the abolition movement in the United States

D The growth of the abolition movement in the United States The excerpt addresses the actions and writings of those, like Douglass, who had been formerly enslaved, aimed at ending the practice and institution of slavery in the United States. The excerpt also highlights the participation of people like Greeley in the abolition or antislavery movement.

"The Declaration of Independence, drawn up by the Continental Congress, was actually a declaration by 'thirteen united States of America' proclaiming that as 'Free and Independent States they have full power to levy war, conclude peace, contract alliances, establish commerce, and to do all other acts and things which independent States may of right do.' And the Articles of Confederation, for all the powers it theoretically gave to the Congress, did not in fact alter this independence. . . . Congressional resolutions continued to be mere recommendations which the states were left to enforce. . . . The Confederation was intended to be, and remained, a Confederation of sovereign states." Gordon S. Wood, historian, The Creation of the American Republic, 1776-1787, published in 1969 Which of the following evidence is used by the author to support his argument about state independence? A The ability of state governments to nullify the laws of other states B The ability of the central government to declare war C The inability of state governments to collect taxes D The inability of the central government to carry out many laws

D The inability of the central government to carry out many laws The author cites the inability of the central government to enforce many laws as an example of the continued independence of the states.

Which of the following developments most directly related to the increased sectional strife immediately prior to the election of 1860? A The addition of territory to the United States through the annexation of California B The rise of nativist political groups in Northern urban areas C The differing viewpoints over the use of the tariff to protect domestic industry D The legal ruling that denied African Americans rights of citizenship

D The legal ruling that denied African Americans rights of citizenship The 1857 Supreme Court decision in the Dred Scott case ruled that enslaved people were property and that African Americans did not have a Constitutional claim to rights of citizenship. This ruling further inflamed sectional tensions prior to 1860 by overturning earlier laws that had banned the expansion of slavery into certain territories and allowing slavery to stand as an institution in any territory in the United States.

Which of the following best explains a change in migration in United States society during the early 1800s? A The claims to land by American Indian groups prevented the migration of White settlers westward. B The system of indentured servitude allowed Europeans living in poverty to immigrate to North America. C The increased importance of Southern cotton coincided with substantial immigration from abroad to the region. D The rise in manufacturing in the North coincided with an increase of immigration from abroad to these urban areas.

D The rise in manufacturing in the North coincided with an increase of immigration from abroad to these urban areas. The rise of manufacturing in northern cities created a large demand for workers, which led to increased immigration from abroad to these cities.

"Not far from this time Nat Turner's insurrection [a slave rebellion] broke out; and the news threw our town into great commotion. . . . "It was always the custom to have a muster every year. On that occasion every White man shouldered his musket. The citizens and the so-called country gentlemen wore military uniforms. . . . "I knew the houses were to be searched; and I expected it would be done by country bullies and the poor Whites. . . . "It was a grand opportunity for the low Whites, who had no Negroes of their own to scourge. They exulted in such a chance to exercise a little brief authority, and show their subserviency to the slaveholders; not reflecting that the power which trampled on the colored people also kept themselves in poverty, ignorance, and moral degradation. . . . Colored people and slaves who lived in remote parts of the town suffered in an especial manner. In some cases the searchers scattered [gun]powder and shot among their clothes, and then sent other parties to find them, and bring them forward as proof that they were plotting insurrection." Harriet Ann Jacobs, Incidents in the Life of a Slave Girl, published in 1861, describing events earlier in the nineteenth century Which of the following claims best aligns with the evidence in the excerpt about the relationship between enslaved African Americans and White Southern citizens? A A majority of White citizens held slaves; therefore, they felt obligated to ensure the preservation of slavery. B The attitudes of White citizens regarding the motives of slave rebellions differed by region within the South. C A majority of poor White citizens benefited economically from having enslaved African Americans do the manual labor that White citizens did not want to do. D The slave system gave poor White citizens the feeling of social superiority over free and enslaved African Americans in a culture where African Americans held little power.

D The slave system gave poor White citizens the feeling of social superiority over free and enslaved African Americans in a culture where African Americans held little power. The excerpt discusses how poor White citizens showed an eagerness to exploit their superiority over African Americans (both free and enslaved), even though they themselves were near the bottom of the social ladder and often did not share in the spoils of the planter class.

"The preservation of the states in a certain degree of agency is indispensable. It will produce that collision between the different authorities which should be wished for in order to check each other. To attempt to abolish the states altogether, would degrade the councils of our country, would be impracticable, would be ruinous. [John Dickinson] compared the proposed national system to the solar system, in which the states were the planets, and ought to be left to move freely in their proper orbits. . . . If the state governments were excluded from all agency in the national one, and all power drawn from the people at large, the consequence would be, that the national government would move in the same direction as the state governments now do, and would run into all the same mischiefs [troubles]." John Dickinson, delegate from Delaware, summary of a speech at the Constitutional Convention from the notes of James Madison, 1787 Dickinson's concern for the "mischiefs" in the states is best understood in the context of which of the following? A Popular support in the states for making George Washington king B The extensive power of the executive in state governments C Continued warfare between some states and Great Britain D The threat to state governments from popular uprisings

D The threat to state governments from popular uprisings At the time of the Constitutional Convention, delegates such as Dickinson worried that popular revolts in some states, such as Shays' Rebellion in Massachusetts, stemmed from the policies of state legislatures and the powerlessness of the central government.

"A bank of the United States is in many respects convenient for the Government and useful to the people. Entertaining this opinion, and deeply impressed with the belief that some of the powers and privileges possessed by the existing bank are unauthorized by the Constitution, subversive of the rights of the States, and dangerous to the liberties of the people, I felt it my duty at an early period of my Administration to call the attention of Congress to the practicability of organizing an institution combining all its advantages and obviating [removing] these objections. I sincerely regret that in the act before me I can perceive none of those modifications of the bank charter which are necessary, in my opinion, to make it compatible with justice, with sound policy, or with the Constitution of our country. . . . "Experience should teach us wisdom. Most of the difficulties our Government now encounters and most of the dangers which impend over our Union have sprung from an abandonment of the legitimate objects of Government by our national legislation. . . . Many of our rich men have not been content with equal protection and equal benefits, but have besought us to make them richer by act of Congress. By attempting to gratify their desires we have in the results of our legislation arrayed section against section, interest against interest, and man against man, in a fearful commotion which threatens to shake the foundations of our Union." President Andrew Jackson, Veto Message Regarding the Bank of the United States, 1832 People who shared the views expressed in the excerpt most likely opposed which of the following? A The expansion of suffrage rights to most White men B The maintenance of low tariffs that allowed the export of goods to Europe C The expansion of United States territory through war D The use of federal government funding for internal improvements

D The use of federal government funding for internal improvements Unlike Whigs, who generally supported using federal funds for transportation projects, members of Jackson's constituency generally opposed projects of that kind as being an overreach of governmental power.

"The great increase of drunkenness, within the last half century, among the people of the United States, led a number of philanthropic individuals . . . to consult together, upon the duty of making more united, systematic, and extended efforts for the prevention of this evil. Its cause was at once seen to be, the use of intoxicating liquor; and its appropriate remedy, abstinence. It was also known, that the use of such liquor, as a beverage, is not only needless, but injurious to the health, the virtue, and the happiness of men. It was believed, that the facts which had been . . . collected would prove this . . . ; and that if the knowledge of them were universally disseminated it would, with the divine blessing, do much toward changing the habits of the nation. . . . [The American Temperance Society's] object is . . . the exertion of kind moral influence . . . to effect such a change of sentiment and practice, that drunkenness and all its evils will cease." Introduction to a book of reports from the American Temperance Society, 1835 Which of the following evidence was used by the American Temperance Society in the excerpt to explain why people would join the temperance movement? A The abstention from alcohol would extend American life expectancy. B The development of treatments for alcoholism would change the habits of men. C The formation of a national movement would eliminate the consumption of alcohol. D The use of specific studies would convince people to believe the movement's goals.

D The use of specific studies would convince people to believe the movement's goals. According to the excerpt, the American Temperance Society collected data in order to spread knowledge about excessive drinking and its consequences on society in order to convince people to join its movement.

"Whether you are or are not, entitled to all the rights of citizenship in this country has long been a matter of dispute to your prejudice. By enlisting in the service of your country at this trial hour, and upholding the National Flag, you stop the mouths of [cynics] and win applause even from the iron lips of ingratitude. Enlist and you make this your country in common with all other men born in the country or out of it. . . . He who fights the battles of America may claim America as his country—and have that claim respected. Thus in defending your country now against rebels and traitors you are defending your own liberty, honor, manhood and self-respect. . . . . . . [H]istory shall record the names of heroes and martyrs who bravely answered the call of patriotism and Liberty—against traitors, thieves and assassins—let it not be said that in the long list of glory, composed of men of all nations—there appears the name of no colored man." Frederick Douglass, excerpt from an editorial, April 1863 Ideas expressed by Douglass in the excerpt were most likely interpreted as supporting which of the following arguments? A The war would make African Americans feel free. B The war was the product of years of injustice against African Americans. C The war would take enormous sacrifice of military combat to achieve victory. D The war was no longer just about preserving the union of the states.

D The war was no longer just about preserving the union of the states. The ideas in the excerpt express a new reasoning that the Civil War could no longer be defined as an effort to preserve the union of the states, but rather that it sought to abolish slavery forever.

"The laity [church members] . . . saw to it that the Second Great Awakening exerted much of its influence through purposeful voluntary associations, typically headed by boards of directors on which laypersons appeared prominently. . . . "Contemporaries called the interlocking, interdenominational directorates of these organizations "the Evangelical United Front" or "the Benevolent Empire." . . . "The social reforms embraced by the Evangelical United Front characteristically involved creating some form of personal discipline serving a goal or redemption. Prison reform serves as an example: No longer would the prison be intended only as a place to hold persons awaiting trial, coerce debt payment, or inflict retributive justice. Reformers reconceived the prison as corrective function, as a 'penitentiary' or 'reformatory,' in the vocabulary they invented. Besides prisoners, other people who did not function as free moral agents might become objects of the reformers' concern: alcoholics, children, slaves, the insane. The goal of the reformers in each case was to substitute for external constraints the inner discipline of morality. Some historians have interpreted the religious reformers as motivated simply by an impulse to impose 'social control,' but it seems more accurate to describe their concern as redemptive, and more specifically the creation of responsible personal autonomy. Liberation and control represented two sides of the redemptive process as they conceived it. Christians who had achieved self-liberation and self-control through conversion not surprisingly often turned to a concern with the liberation and discipline of others. . . . "The religious awakenings of the early nineteenth century marshaled powerful energies in an age when few other social agencies in the United States had the capacity to do so. [The] Evangelical United Front organized its voluntary associations on a national, indeed international, level, at a time when little else in American society was organized, when there existed no nationwide business corporation save the Second Bank of the United States and no nationwide government bureaucracy save the Post Office. Indeed, the four major evangelical denominations together employed twice as many people, occupied twice as many premises, and raised at least three times as much money as the Post Office." Daniel Walker Howe, historian, What Hath God Wrought: The Transformation of America, 1815-1848, published in 2007 Which of the following describes a piece of evidence used by Howe to support his overall argument about the motivations of religious reformers? A They believed that they should focus their efforts only on the United States. B They sought to compete with other churches for religious converts. C They viewed reform mainly as a means of social control. D They desired to teach people personal autonomy.

D They desired to teach people personal autonomy. In the second paragraph, Howe provides evidence that prison reformers sought to cultivate personal autonomy in order to argue that religious reformers were motivated by a desire to redeem people by teaching self-discipline.

"The petition of a great number of Blacks detained in a state of slavery in the bowels of a free and Christian country humbly showeth that your petitioners apprehend that they have in common with all other men a natural and inalienable right to that freedom which the Great Parent of the Universe hath bestowed equally on all mankind. . . . They were unjustly dragged by the hand of cruel power . . . from a populous, pleasant, and plentiful country and in violation of laws of nature and of nations. . . . ". . . Your petitioners . . . cannot but express their astonishment that it has never been considered that every principle from which America has acted in the course of their unhappy difficulties with Great Britain pleads stronger than a thousand arguments in favor of your petitioners. They therefore humble beseech your honors to give this petition its due weight and consideration and cause an act of the legislature to be passed whereby they may be restored to the enjoyments of that which is the natural right of all men—and their children who were born in this land of liberty may not be held as slaves after they arrive at the age of twenty one years." Petition to the Massachusetts state legislature, 1777 The second paragraph of the excerpt proposes that the Massachusetts legislature should A allow enslaved Africans to serve in the state's militia B find a way to resolve its conflicts with Britain peacefully C write a constitution explaining how natural rights apply to enslaved people D abolish slavery and release enslaved African Americans upon adulthood

D abolish slavery and release enslaved African Americans upon adulthood The request in the excerpt that "an act of legislature be passed whereby [the petitioners] may be restored to the enjoyments of that which is the natural right of all men" means that the African American petitioners are asking for their freedom. The following line calls for children born in the United States to be released from slavery at age 21.

"The United States [under the Articles of Confederation] has an indefinite discretion to make [requests] for men and money; but they have no authority to raise either, by regulations extending to the individual citizens of America. The consequence of this is, that though in theory their resolutions concerning those objects are laws, constitutionally binding on the members of the Union, yet in practice they are mere recommendations which the States observe or disregard at their option. "There is nothing absurd or impracticable in the idea of a league or alliance between independent nations for certain defined purposes . . . depending for its execution on the good faith of the parties. . . . In the early part of the present century there was an [enthusiasm] in Europe for [leagues or alliances]. . . . They were scarcely formed before they were broken, giving an instructive but afflicting lesson to mankind, how little dependence is to be placed on treaties which have no other sanction than the obligations of good faith. . . . "There was a time when we were told that breaches, by the States, of the regulations of the [Confederation's] authority were not be expected. . . . "In our case, the [agreement] of thirteen distinct sovereign wills is requisite, under the Confederation, to the complete execution of every important measure that proceeds from the Union. . . . The measures of the Union have not been executed. . . . Each State, yielding to the persuasive voice of immediate interest or convenience, has successively withdrawn its support." Alexander Hamilton, The Federalist paper number 15, published in 1787 Hamilton claimed in the excerpt that state sovereignty A increased the unity of the United States B guaranteed Americans the protection of their liberties C encouraged Americans to pursue the common good in politics D allowed states to ignore the requests of the central government

D allowed states to ignore the requests of the central government In the first and third paragraphs of the excerpt, Hamilton claimed that the sovereignty retained by states under the Articles of Confederation allowed them to disregard laws passed by Congress as voluntary requests.

"It is not only important, but, in a degree necessary, that the people of this country, should have an American Dictionary of the English language; for, although the body of the language is the same as in England, . . . yet some differences must exist. Language is the expression of ideas; and if the people of one country cannot preserve an identity of ideas, they cannot retain an identity of language. . . . But the principal differences between the people of this country and of all others, arise from different forms of government, different laws, institutions and customs. Thus the . . . feudal system of England originated terms which formed . . . a necessary part of the language of that country; but, in the United States, many of these terms are no part of our present language,—and they cannot be, for the things which they express do not exist in this country. . . . The institutions in this country which are new and peculiar, give rise to new terms or to new applications of old terms, unknown to the people of England; which cannot be explained by them and which will not be inserted in their dictionaries, unless copied from ours. . . . No person in this country will be satisfied with the English definitions of the words congress, senate, and assembly, court, [etc.] for although these are words used in England, yet they are applied in this country to express ideas which they do not express in that country." Noah Webster, "Preface," An American Dictionary of the English Language, 1828 The historical concept of the American identity, as characterized in the excerpt, was most clearly distinguished from the identities of other nations by the A enthusiasm for copying English practices and institutions B movement toward establishing a capitalist economy C role women played in civic and public life D importance of liberal ideas about natural rights and liberties

D importance of liberal ideas about natural rights and liberties One of Webster's major points in creating an American dictionary was to establish a separate language that had new definitions for political institutions and new ways to express political ideas, particularly of natural rights and liberty.

"The existence of [colonial] subregions leads us to another question: whether the Middle Colonies in fact represented a coherent region at all. . . . In important respects, the Middle Colonies can be divided into separate societies focused around the cities of New York and Philadelphia. Thus the economies of [New York] and northern New Jersey were tied closely to that of New York City, while those of southern New Jersey, Pennsylvania, and northern Delaware were linked to Philadelphia. Those areas grew at very different rates, and they possessed quite distinct characteristics. . . . "Nonetheless, the Middle Colonies did share a number of things. One was their geography, a combination of climate and topography and setting, which determined some of the ways the land could be put to use, its accessibility to both intra-regional and international commerce, and its strategic importance in imperial competition. It was a region organized around extensive inland waterways, which gave merchants an almost unparalleled access to the American interior, building upon trade routes that pre-dated European settlement. . . . "Perhaps the most important argument for the coherence of the mid-Atlantic as a region is the extent to which those colonies shared a common history. . . . "The most often-noted characteristic of the region was the diversity of its peoples. . . . The society of the Middle Colonies surely was 'America's first plural society.' . . . There were two principal sources of the growing diversity of the European settlements. One was historical: New York, New Jersey, and Delaware were all conquered colonies, with Dutch, Swedish, Finnish, and many other populations already resident at the time of English conquest. The other was the consolidation that occurred as the colonies of six European nations along the Atlantic coast in the early seventeenth century were reduced to two by century's end, those of [Protestant] England and those of [Catholic] France. The result was that [diverse] European Protestants heading for the New World were concentrated within English colonies, a situation that virtually mandated some form of toleration. . . . Toleration and pluralism, it turns out, were not based solely on enlightened benevolence." Ned C. Landsman, historian, Crossroads of Empire: The Middle Colonies in British North America, published in 2010 Landsman claims that some historians might not consider the Middle Colonies a single British colonial region because the Middle Colonies A contained multiple inland waterways for commerce B were settled by ethnically diverse groups of Europeans C received European immigrants who practiced a variety of religions D had different local economies focused on Philadelphia and New York City

D had different local economies focused on Philadelphia and New York City In the first paragraph of the excerpt, Landsman argues that because they had different economies focused on New York City and Philadelphia, the Middle Colonies might not be considered a single colonial region.

"[In Virginia] the Negroes live in small cottages called quarters . . . under the direction of an overseer, who takes care that they tend such land as the owner allots and orders. . . . Their greatest hardship [is] consisting in that they and their posterity are not at their own liberty or disposal, but the property of their owners. . . . The children belong to the master of the woman that bears them. . . . "[The] abundance of [the] English entertain . . . that they are all fools and beggars that live in any [other] country but theirs. This home fondness has been very prejudicial [harmful] to the common sort of English, and has in a great measure [slowed] the plantations from being stocked with such inhabitants as are skillful, industrious, and laborious. . . . "These [English] servants are but an insignificant number, when compared with the vast shoals [mass] of Negroes who are employed as slaves there to do the hardest and most part of the work." Hugh Jones, The Present State of Virginia, 1724 The labor system described in the first paragraph of the excerpt was most similar to the labor system used for A mining in New Spain B whaling in New England C acquiring furs in New France D producing sugar in the Caribbean

D producing sugar in the Caribbean The labor system described in the first paragraph was most similar to the labor system of chattel slavery that was used to produce sugar in the Caribbean, in which enslaved people were held for their lifetimes by slaveholders.

"Brother, listen to what we say. There was a time when our forefathers owned this great [land]. . . . Your forefathers crossed the great water and landed upon this [land]. Their numbers were small. They found friends, not enemies. They told us they had fled from their own country for fear of wicked men, and had come here to enjoy their religion. They asked for a small seat. We took pity on them, we granted their request, and they sat down amongst us. We gave them corn and meat; they gave us poison in return. ". . . Our seats were once large and yours were small. You have now become a great people, and we have scarcely a place left to spread our blankets. You have got our country, but are not satisfied; you want to force your religion upon us. . . . ". . . The Great Spirit has made us all, but he has made a great difference between his white and red children. . . . Since he has made so great a difference between us in other things, why may we not conclude that he has given us a different religion according to our understanding? The Great Spirit does right. He knows what is best for his children; we are satisfied." Red Jacket, Iroquois American Indian chief in New York, speech to a missionary from Massachusetts and a United States diplomat, 1805 The excerpt could best be used by historians studying the A origins of the Second Great Awakening B effects of the market revolution on family roles C colonization of eastern North America by English settlers D resistance against the expansion of United States influence

D resistance against the expansion of United States influence Red Jacket's expression in the excerpt of his desire to maintain traditional Iroquois religious practices best helps historians interpret the ways in which American Indians resisted attempts by the United States to expand its influence over themselves and their homelands.

The encomienda and slavery systems both contributed to which of the following developments?

The Spanish developed a race-based caste system that defined the status of Europeans, Native Americans, Africans, and people of mixed race in their colonies. The Spanish empire maintained a diverse population and developed a caste system that could incorporate and categorize people by racial identity, including Europeans, Native Americans, Africans, and people of mixed race.

"It was painful for me, on a subject of such national importance, to differ from the respectable members who signed the Constitution; but conceiving, as I did, that the liberties of America were not secured by the system, it was my duty to oppose it. "My principal objections to the plan are, that there is no adequate provision for a representation of the people; . . . that some of the powers of the legislature are ambiguous . . . ; that the executive is blended with, and will have an undue influence over, the legislature; that the judicial department will be oppressive; . . . and that the system is without the security of a bill of rights. These are objections which are not local, but apply equally to all the states. "As the Convention was called for the 'sole and express purpose of revising the Articles of Confederation . . . ,' I did not conceive that these powers extend to the formation of the plan proposed; but the Convention being of a different opinion, I acquiesced [agreed] in it, being fully convinced that, to preserve the Union, an efficient government was indispensably necessary, and that it would be difficult to make proper amendments to the Articles of Confederation. "The Constitution proposed has few, if any, federal features, but is rather a system of national government. Nevertheless, in many respects, I think it has great merit, and, by proper amendments, may be adapted. . . . "Others may suppose that the Constitution may be safely adopted, because therein provision is made to amend it. But cannot this object be better attained before a ratification than after it? And should a free people adopt a form of government under conviction that it wants [needs] amendment?" Elbridge Gerry, letter to the Massachusetts state legislature, 1787 Gerry made which of the following arguments regarding amending the Constitution? A A Bill of Rights should be added before ratification. B Enslaved people should not be counted for representation. C The executive and legislative branches should share more power. D More populous states should have greater representation in Congress.

A A Bill of Rights should be added before ratification. In the last paragraph of the excerpt, Gerry argued that any modifications to the Constitution should be made while creating the new government rather than after establishing it.

"I have heard it asserted by some, that as America hath flourished under her former connection with Great Britain, that the same connection is necessary towards her future happiness, and will always have the same effect. Nothing can be more fallacious [untrue] than this kind of argument. We may as well assert that because a child has thrived upon milk, that it is never to have meat, or that the first twenty years of our lives is to become a precedent for the next twenty. But even this is admitting more than is true, for I answer roundly, that America would have flourished as much, and probably much more, had no European power had any thing to do with her. . . . "But Britain is the parent country, say some. Then the more shame upon her conduct. Even brutes do not devour their young, nor savages make war upon their families. . . . Europe, and not England, is the parent country of America. This new world hath been the asylum for the persecuted lovers of civil and religious liberty from every part of Europe. Hither have they fled, not from the tender embraces of the mother, but from the cruelty of the monster; and it is so far true of England, that the same tyranny which drove the first emigrants from home, pursues their descendants still." Thomas Paine, Common Sense, 1776 Paine's argument best provides evidence for which of the following developments resulting from the American Revolution? A The rapid creation of an industrialized economy in the United States B The increase in immigration from continental Europe and reduction in immigration from Great Britain C The emergence of a unique American national identity separate from that of Europe D The fear among the Spanish and French that that revolutionary ideas would spread, which prompted them to abandon their colonies

C The emergence of a unique American national identity separate from that of Europe Paine's argument of American economic independence and the desire to escape European "tyranny" best shows how the people living in the colonies could assert a new, separate identity following the American Revolution.

Which of the following political changes most likely influenced the Second Great Awakening? A The president called for the removal of American Indians from the southeastern states. B Southern politicians threatened to nullify the authority of the federal government. C A participatory democracy expanded belief in the importance of the individual. D A series of judicial rulings supported the development of infrastructure projects.

C A participatory democracy expanded belief in the importance of the individual. As more Americans participated in electing government officials, the growth of participatory democracy and expanded belief in the importance of the individual helped shape emphasis on personal religious salvation in the Second Great Awakening.

"On the western side of the ocean, movements of people and ideas . . . preceded the Atlantic connection. Great empires—in the Valley of Mexico, on the Mississippi River . . . —had collapsed or declined in the centuries before 1492. . . . As Columbus embarked on his first transatlantic voyage, the Mexica, or Aztecs, were consolidating their position [in Mexico]; their city was a center of both trade and military might. Tenochtitlán [the Aztec capital] . . . held 200,000 people, a population greater than in the largest city in contemporary Europe. ". . . The Mississippian culture spread east and west from its center, the city of Cahokia, on the Mississippi River near the site of modern St. Louis. It was a successor to earlier cultures, evidence of which can be seen in the great ceremonial mounds they built. Cahokia declined and was ultimately abandoned completely in the later thirteenth century. . . . Throughout the Southeast, smaller mound-building centers continued." Karen Ordahl Kupperman, historian, The Atlantic in World History, 2012 Which of the following most directly contributed to the advanced development of both pre-Columbian American societies described in the excerpt? A Creation of military forces stronger than those of most rival societies B Access to waterways to transport goods and trade with other societies C Adaptation to and use of the natural environment for their own benefit D Transfer of power through inheritance, which reinforced spiritual claims to authority

C Adaptation to and use of the natural environment for their own benefit Both the Aztec and Mississippian pre-Columbian societies adapted to their environments and exploited their surroundings in order to succeed. Both built large central cities and lasting cultural practices such as mound-building.

"I do not belong, said Mr. [Calhoun], to the school which holds that aggression is to be met by concession. . . . If we concede an inch, concession would follow concession—compromise would follow compromise, until our ranks would be so broken that effectual resistance would be impossible. . . . ". . . A large portion of the Northern States believed slavery to be a sin, and would believe it to be an obligation of conscience to abolish it if they should feel themselves in any degree responsible for its continuance. . . . ". . . Abolition and the Union cannot coexist. As the friend of the Union, I openly proclaim it—and the sooner it is known the better. The former may now be controlled, but in a short time it will be beyond the power of man to arrest the course of events. We of the South will not, cannot, surrender our institutions. To maintain the existing relations between the two races, inhabiting that section of the Union, is indispensable to the peace and happiness of both. . . . But let me not be understood as admitting, even by implication, that the existing relations between the two races in the slaveholding States is an evil—far otherwise; I hold it to be a good, as it has thus far proved itself to be to both, and will continue to prove so if not disturbed by the fell spirit of abolition." Source: South Carolina senator John C. Calhoun, speech in the United States Senate, 1837. The ideas expressed by John C. Calhoun and others who shared his views on slavery had which of the following effects on emerging abolitionist movements in the years leading up to the Civil War? A Many abolitionist groups in the North began to question the accounts of harsh treatment described by escaped slaves who made it to freedom. B Arguments describing slavery as a "positive good" weakened the impact of abolitionist efforts to encourage White northerners to support emancipation. C As many people came to see slavery as part of the Southern way of life, attitudes on both sides of the slavery argument hardened so that political compromise became difficult. D Very few members of Congress accepted Calhoun's "positive good" argument, and they became more open to passing laws limiting slaveholding and the internal slave trade.

C As many people came to see slavery as part of the Southern way of life, attitudes on both sides of the slavery argument hardened so that political compromise became difficult. Partly as a result of speeches like Calhoun's, attitudes both for and against slavery became hardened in the years leading up to the Civil War, and each side became less willing to consider political compromise.

"Antebellum planters . . . were very interested in the control of black movement. They were also keen to master their slaves' senses of pleasure. Seeking to contain [African Americans] even further than laws, curfews, bells, horns, and patrols already did, some planters used plantation [parties] as a paternalist mechanism of social control. Plantation parties, which carefully doled out joy on Saturday nights and holidays, were intended to seem benevolent and to inspire respect, gratitude, deference, and importantly, obedience. . . . The most important component of paternalistic plantation parties was the legitimating presence of the master. ". . . [Yet] again and again, slaves sought out illicit, secular gatherings of their own creation. They disregarded curfews and pass laws to escape to secret parties where . . . pleasures such as drinking, eating, dancing, and dressing up were the main amusements. . . . ". . . In the context of enslavement, such exhilarating pleasure . . . must be understood as important and meaningful enjoyment, as personal expression, and as oppositional." Stephanie M. H. Camp, historian, Closer to Freedom: Enslaved Women and Everyday Resistance in the Plantation South, 2004 Which of the following best describes a context in the first half of the 1800s that influenced the development of slavery as described in the excerpt? A The United States expanded its participation in the international slave trade. B Northern business leaders sought enslaved people as laborers for transportation projects. C Southern planters used enslaved people to produce cotton for international markets. D Protestant religious revivalists encouraged the growth of antislavery movements.

C Southern planters used enslaved people to produce cotton for international markets. The context of the use of enslaved laborers to produce cotton for international markets in the first half of the 1800s influenced the development of slavery as described in the excerpt.

"Mississippi planter and agricultural reformer M. W. Phillips, a regular contributor to the American Cotton Planter, wrote about soil exhaustion and crop rotation, and extolled the virtues of manuring and self-provisioning. In one of his most widely reproduced articles, Phillips condemned planters before whom 'everything has to bend [and] give way to large crops of cotton.' . . . "Phillips imagined the cotton economy in terms of flows of energy, nutrients, and fertility, all of which he was convinced were being expended at an unsustainable rate. He used images of human, animal, and mineral depletion to represent an onrushing ecological catastrophe. But he did so within the incised [limited] terms allowed him by his culture—the culture of cotton. Phillips was arguing that the slaveholding South needed to slow the rate at which it was converting human beings into cotton plants." Walter Johnson, historian, River of Dark Dreams: Slavery and Empire in the Cotton Kingdom, 2013 Which of the following most directly contributed to the development described in the excerpt? A The introduction of enslaved Africans in the 1600s into what is now the United States B The election of Andrew Jackson to the presidency and his decision to enforce tariff collections C A series of slave insurrections and rebellions in the first half of the 1800s D A belief by southern businessmen that the southern economy should focus on the export of select agricultural products

D A belief by southern businessmen that the southern economy should focus on the export of select agricultural products The overproduction of cotton that is described in the excerpt resulted directly from southern business leaders promoting its production and export to a global market.

Which of the following developments in labor systems in the United States before the Civil War is reflected by the data in the graph? A Indentured servants replaced wage laborers in the North. B Women laborers performed less industrial work in the North. C Large numbers of immigrants moved to urban areas in the South. D An internal trade in enslaved people spread throughout the South.

D An internal trade in enslaved people spread throughout the South. The export economy and number of large farms engaged in agriculture in the South, as indicated by the graph, reflect the growth of an internal slave trade to supply labor for plantations producing cotton for export.

"Jackson truly believed that, compared to his predecessors' combination of high-minded rhetoric, treachery, and abandonment, his Indian policy was 'just and humane.' . . . ". . . Jackson's paternalism was predicated on his assumption, then widely but not universally shared by white Americans, that all Indians . . . were [irrational] and inferior to all whites. His promises about voluntary and compensated relocation . . . were constantly undermined by delays and by sharp dealing by War Department negotiators—actions Jackson condoned. . . . Jackson tried to head off outright fraud, but the removal bill's allotment scheme invited an influx of outside speculators, who wound up buying between 80 and 90 percent of the land owned by Indians who wished to stay at a fraction of its actual worth. At no point did Jackson consider allowing even a small number of Georgia Cherokees who preferred to stay to do so in select enclaves, an option permitted to small numbers of Iroquois in upstate New York and Cherokees in western North Carolina. . . . Bereft of long-term planning and a full-scale federal commitment, the realities of Indian removal belied Jackson's rhetoric. Although the worst suffering was inflicted after he left office, Jackson cannot escape responsibility for setting in motion an insidious policy that uprooted tens of thousands of Choctaws and Creeks [from the Southeast] during his presidency." Sean Wilentz, historian, The Rise of American Democracy: Jefferson to Lincoln, published in 2005 Which of the following claims is supported by the author's main argument in the excerpt? A White settler demand for land was the principal cause of Indian removal policies. B Partnerships between White settlers and American Indians prevented most removals. C Andrew Jackson planned Indian removal to punish American Indians after conflicts. D Andrew Jackson can be blamed for the unintended effects of Indian removal.

D Andrew Jackson can be blamed for the unintended effects of Indian removal. The claim that Jackson can be blamed for unintended effects of Indian removal supports Wilentz's argument in the excerpt that Jackson bears responsibility for the suffering inflicted on American Indian peoples because of the policies he implemented.

"I know not how to thank you for the deep and lively interest you have been pleased to take in the cause of . . . the emancipation of a people, who, for two long centuries, have endured, with the utmost patience, a bondage, one hour of which . . . is worse than ages of that which your fathers rose in rebellion to oppose. "It is such indications on the part of the press—which, happily, are multiplying throughout all the land—that kindle up within me an ardent hope that the curse of slavery will not much longer be permitted to make its iron foot-prints in the lacerated [deeply cut] hearts of my . . . brethren. . . . I am called, by way of reproach, a runaway slave. As if it were a crime—an unpardonable crime—for a man to take his inalienable rights! "But why [you,] a New-York editor, born and reared in the State of Maine, far removed from the contaminated . . . atmosphere of slavery, should pursue such a course [supporting abolition], is not so apparent. I will not, however, stop here to ascertain the cause, but deal with fact. . . . "The object . . . is simply to give such an exposition of the degrading influence of slavery upon the master and his [supporters] as well as upon the slave—to excite such an intelligent interest on the subject of American slavery—as may react upon that country, and tend to shame her out of her adhesion to a system which all must confess to disagree with justice. . . . "I am earnestly and anxiously laboring to wipe off this foul blot from the . . . American people, that they may accomplish in behalf of human freedom that which their exalted position among the nations of the earth amply fits them to do." Frederick Douglass to New York Tribune editor Horace Greeley, 1846 Ideas in the excerpt would most likely have influenced which of the following? A Arguments supporting defining slavery on the grounds of states' rights B Claims that the United States should occupy all Mexican territory C Attempts to convince plantation owners to stop farming cash crops D Efforts at assisting enslaved people in escaping from the South

D Efforts at assisting enslaved people in escaping from the South Douglass' argument asserts that enslaved people maintained inalienable rights and that one of those included the right to attempt to obtain their freedom by running away.

"[In Virginia] the Negroes live in small cottages called quarters . . . under the direction of an overseer, who takes care that they tend such land as the owner allots and orders. . . . Their greatest hardship [is] consisting in that they and their posterity are not at their own liberty or disposal, but the property of their owners. . . . The children belong to the master of the woman that bears them. . . . "[The] abundance of [the] English entertain . . . that they are all fools and beggars that live in any [other] country but theirs. This home fondness has been very prejudicial [harmful] to the common sort of English, and has in a great measure [slowed] the plantations from being stocked with such inhabitants as are skillful, industrious, and laborious. . . . "These [English] servants are but an insignificant number, when compared with the vast shoals [mass] of Negroes who are employed as slaves there to do the hardest and most part of the work." Hugh Jones, The Present State of Virginia, 1724 The economy of the Middle Colonies differed from the economy of Virginia described in the excerpt in that the Middle Colonies more often A relied on enslaved labor in cities B imported enslaved Africans C engaged in trans-Atlantic commerce D purchased land from Native Americans

A relied on enslaved labor in cities Cities in the Middle Colonies and New England had larger populations of enslaved people than did the few cities in the Chesapeake and Carolinas.

Which of the following best describes the trend in the number of enslaved Africans brought to the Carolinas and Georgia depicted in the graph? A It sharply decreased before the 1770s and then returned to the level of 1751-1760. B It increased before the 1770s and then declined between 1771 and 1790. C It remained steady before 1770 and reached zero slave imports by 1771. D It consistently decreased until reaching zero slave imports by 1781

B It increased before the 1770s and then declined between 1771 and 1790. The data in the graph show that the number of enslaved Africans brought to the Carolinas and Georgia increased from 1751 to 1770 before declining in the 1770s and 1780s and remaining constant after 1790.

"In the time of the late war, being desirous to defend, secure, and promote the Rights and Liberties of the people, we spared no pains but freely granted all the aid and assistance of every kind that our civil fathers [political leaders] required of us. "We are sensible also that a great debt is justly brought upon us by the War, and we are as willing to pay our share towards it as we are to enjoy our shares in independency. . . . "But with the greatest submission we beg leave to inform your Honors that unless something takes place more favorable to the people, in a little time at least one half of our inhabitants in our opinion will become bankrupt. . . . When we compute the taxes laid upon us the five preceding years, the State and County, town, and class taxes, the amount is equal to what our farms will rent for. Sirs in this situation, what have we to live on: No money to be had; our estates daily posted and sold. . . . Surely your Honors are no strangers to the distresses of the people but do know that many of our good inhabitants are now confined in jail for debt and for taxes. . . . Will not the people in the neighboring states say of this state: although the Massachusetts [people] boast of their fine Constitution, their government is such that it devours their inhabitants? ". . . If your Honors find anything above mentioned worthy of notice, we earnestly pray that . . . [the state legislature] would point out some way whereby the people might be relieved." Petition from the town of Greenwich to the Massachusetts state legislature, 1786 Concerns about domestic political unrest in the early United States were lessened by which of the following developments? A The outbreak of the Haitian Revolution B The creation of a stronger central government C The attempts by Native American peoples to limit White settlement D The conflict between Great Britain and France after the French Revolution

B The creation of a stronger central government One of the key motivations behind the strengthening of the central government under the Constitution was to lessen the possibility of domestic political revolts, such as Shays' Rebellion in Massachusetts.

"I . . . write an account to Your Majesty as the first [person] to come among these natives. . . . "These Indian people of New Spain [Mexico] are vassals of Your Majesty. . . . I dare plead with you for a remedy because, for their people to be saved, they are in great need of relief in order to devote themselves at least somewhat to matters of Faith. After all, it is the struggle for their salvation that justifies their discovery. . . . "I firmly believe that if the decrees Your Majesty sent here for their benefit were implemented, and if the governors and judges did more than pretend to do so, great good would have come to these people. Even more firmly I believe that Your Majesty's intention is that they be saved and that they know God. For this to happen, they must have some relief, so that with the moderate labor needed to meet their tribute obligation, they can still give themselves wholeheartedly to our teachings. . . . Otherwise, God will have good reason to complain, for Spaniards came to this land and have taken their property for their own benefit, and Your Majesty has extracted great benefit from them, too. . . . ". . . Your Majesty . . . should know that the Indians who are required to labor for a master in Mexico City in domestic service and bring firewood, fodder, and chickens leave their pueblo for a month at a time. . . . And the poor Indians often have to buy these things because they are not to be found in their pueblos. . . . Take pity on them and consider what is happening to the poor Indian woman who is in her house with no one to support her and her children, for her husband is hard pressed to meet his tribute requirement. . . . ". . . I advise you that if Your Majesty does not establish that . . . [the Indians] be required to pay tribute only from what they have, within thirty years these parts will be as deserted as the [Caribbean] islands, and so many souls will be lost." Fray (Friar) Pedro de Gante, Spanish Catholic friar and missionary, letter to Emperor Charles V, king of Spain, 1552 In the third paragraph of the excerpt, which of the following pieces of evidence does de Gante use to support his argument about the emperor of Spain's obligations to Native Americans? A Native Americans have their own religious practices. B Native American workers were eager to fulfill the emperor's wishes. C The emperor has benefited from the riches acquired in the Americas. D The royal governors have followed the crown's decrees in the Americas.

C The emperor has benefited from the riches acquired in the Americas. In the third paragraph, de Gante supported the argument that the Spanish emperor should protect Native Americans from abuses with evidence that the benefits from colonization such as spiritual salvation imposed the obligation to protect the Native Americans and convert them to Christianity. Otherwise God would become angry.

Which of the following best describes an overall trend depicted in the graph for the time period between 1751 and 1800 ? A The slave trade to the North increased. B The slave trade was halted as a result of the American Revolution. C The number of enslaved Africans brought to the United States decreased. D The number of enslaved Africans brought to the Carolinas and Georgia steadily increased.

C The number of enslaved Africans brought to the United States decreased. Except for a slight increase in the international slave trade to the Carolinas and Georgia in the 1760s, the overall trend of the data in the graph indicates that the number of enslaved Africans brought to British North America and the United States in this period decreased.

Which of the following describes a trend shown in the graph of the regional distribution of the slave trade before the American Revolution? A The northern colonies did not participate in the slave trade. B The slave trade to Virginia and Maryland increased. C Fewer enslaved Africans were brought to the southern colonies than to the northern colonies. D More enslaved Africans were brought to the Carolinas and Georgia than to Virginia and Maryland.

D More enslaved Africans were brought to the Carolinas and Georgia than to Virginia and Maryland. The data in the graph indicate that throughout the period prior to the American Revolution (1775-1783), more enslaved Africans were brought to the Carolinas and Georgia than to Virginia and Maryland.

"Every British Subject born on the continent of America . . . is by the law of God and nature, by the common law, and by act of parliament, . . . entitled to all the natural, essential, inherent and inseparable rights of our fellow subjects in Great-Britain. Among those rights are the following . . . : ". . . Taxes are not to be laid on the people, but by their consent in person, or by [representatives]. ". . . I can see no reason to doubt, but that the imposition of taxes, whether on trade, or on land, or houses, or ships, . . . in the colonies is absolutely irreconcilable with the rights of the Colonists, as British subjects. . . . "The power of parliament is uncontrollable, . . . and we must obey. . . . Therefore let the parliament lay what burthens they please on us, we must, it is our duty to submit and patiently bear them till they . . . afford us relief by repealing such acts, as through mistake, or other human infirmities, have been suffered to pass, if they can be convinced that their proceedings are not constitutional." James Otis, The Rights of the British Colonies Asserted and Proved, pamphlet, 1764 In the excerpt, Otis was responding to which of the following developments? A The publication of Thomas Paine's Common Sense B The threat of a French invasion of British North America C The introduction of widespread boycotts against imported British goods D The British government's attempts to pay for the costs of the Seven Years' War (French and Indian War)

D The British government's attempts to pay for the costs of the Seven Years' War (French and Indian War) After the end of the Seven Years' War in 1763, Great Britain began to pursue new policies to raise revenue in its North American colonies, including the stricter enforcement of the Navigation Acts and the passage of new taxes for the colonies. Colonists such as Otis opposed these measures, claiming they violated the rights of colonists as British subjects.

"I . . . write an account to Your Majesty as the first [person] to come among these natives. . . . "These Indian people of New Spain [Mexico] are vassals of Your Majesty. . . . I dare plead with you for a remedy because, for their people to be saved, they are in great need of relief in order to devote themselves at least somewhat to matters of Faith. After all, it is the struggle for their salvation that justifies their discovery. . . . "I firmly believe that if the decrees Your Majesty sent here for their benefit were implemented, and if the governors and judges did more than pretend to do so, great good would have come to these people. Even more firmly I believe that Your Majesty's intention is that they be saved and that they know God. For this to happen, they must have some relief, so that with the moderate labor needed to meet their tribute obligation, they can still give themselves wholeheartedly to our teachings. . . . Otherwise, God will have good reason to complain, for Spaniards came to this land and have taken their property for their own benefit, and Your Majesty has extracted great benefit from them, too. . . . ". . . Your Majesty . . . should know that the Indians who are required to labor for a master in Mexico City in domestic service and bring firewood, fodder, and chickens leave their pueblo for a month at a time. . . . And the poor Indians often have to buy these things because they are not to be found in their pueblos. . . . Take pity on them and consider what is happening to the poor Indian woman who is in her house with no one to support her and her children, for her husband is hard pressed to meet his tribute requirement. . . . ". . . I advise you that if Your Majesty does not establish that . . . [the Indians] be required to pay tribute only from what they have, within thirty years these parts will be as deserted as the [Caribbean] islands, and so many souls will be lost." Fray (Friar) Pedro de Gante, Spanish Catholic friar and missionary, letter to Emperor Charles V, king of Spain, 1552 Which of the following best describes an argument made by de Gante in the letter? A The Catholic Church should abandon conversion efforts in the Americas. B The emperor should sponsor voyages to discover new American kingdoms. C De Gante should be rewarded for his missionary work in the Americas. D The Spanish should require less tribute after conquest to avoid Native American depopulation.

D The Spanish should require less tribute after conquest to avoid Native American depopulation. In the excerpt, de Gante argues that the emperor should reduce Native Americans' tribute requirements to Spanish conquerors in order to avoid the depopulation experienced in the Caribbean. Payments of tribute and labor through encomiendas were imposed by the Spanish on Native Americans after the conquest of Mexico in the 1520s. These requirements were criticized by some Spanish religious leaders as harmful to Native Americans and counterproductive to Christian missionary efforts.

"Barbarians [are] . . . , in the proper and strict sense of the word, dull witted and lacking in the reasoning powers necessary for self-government. They are without laws, without king, etc. For this reason they are by nature unfitted for rule. "[Some] barbarians . . . have a lawful, just, and natural government. Even though they lack the art and use of writing, they are not wanting in the capacity and skill to rule and govern themselves. . . . Thus they have kingdoms, communities, and cities that they govern wisely according to their laws and customs. . . . ". . . It does not necessarily follow that [Native Americans] are incapable of government and have to be ruled by others, except to be taught about the Catholic faith and to be admitted to the holy sacraments. They are not ignorant, inhuman, or bestial. Rather, long before they had heard the word Spaniard they had properly organized states, wisely ordered by excellent laws, religion, and custom. They cultivated friendship and . . . lived together in populous cities in which they wisely administered the affairs of both peace and war justly and equitably, truly governed by laws that at very many points surpass ours. . . . ". . . [Was] the war of the Romans against the [ancient] Spanish justified in order to free them from barbarism? . . . Do you think that the Romans, once they had [conquered] the wild and barbaric peoples of Spain, could with secure right divide all of you [Spaniards] among themselves [in encomiendas] . . . ? And do you then conclude that the Romans could have stripped your rulers of their authority and consigned all of you, after you had been deprived of your liberty, to wretched labors, especially in searching for gold and silver [mines]. . . ? Is this the way to impose the yoke of Christ on Christian men?" Bartolomé de Las Casas, Spanish Catholic religious leader, In Defense of the Indians, circa 1550 Which of the following describes a piece of evidence for Las Casas' claim in the fourth paragraph about the similarity between ancient Spanish people and Native Americans? A The ancient Spanish were used by the Romans as laborers. B The ancient Spanish were Christians when the Romans arrived. C The ancient Spanish had no laws or political rulers. D The ancient Spanish had a right to their own freedom.

D The ancient Spanish had a right to their own freedom. Las Casas provided evidence that the ancient Spanish had a right to their own freedom to support his claim that the Native Americans should be protected from abuses by Spanish colonizers just as the ancient Spanish were not subjected to similar abuses by the Romans.

The change in White and Black populations in the various regions shown in the table most strongly suggests the influence of which of the following? A The eventual decline of slave labor in the Chesapeake and Carolinas B The colonists' growing resistance to British imperial control in New England C The spread of epidemic diseases across North America D The demand in Europe for commodities such as tobacco and sugar

D The demand in Europe for commodities such as tobacco and sugar The expansion of populations in colonies in the South and West Indies, especially the extreme increase in the population of enslaved Africans, was a result of the widespread expansion of the production of crops for European consumption.

"I have heard it asserted by some, that as America hath flourished under her former connection with Great Britain, that the same connection is necessary towards her future happiness, and will always have the same effect. Nothing can be more fallacious [untrue] than this kind of argument. We may as well assert that because a child has thrived upon milk, that it is never to have meat, or that the first twenty years of our lives is to become a precedent for the next twenty. But even this is admitting more than is true, for I answer roundly, that America would have flourished as much, and probably much more, had no European power had any thing to do with her. . . . "But Britain is the parent country, say some. Then the more shame upon her conduct. Even brutes do not devour their young, nor savages make war upon their families. . . . Europe, and not England, is the parent country of America. This new world hath been the asylum for the persecuted lovers of civil and religious liberty from every part of Europe. Hither have they fled, not from the tender embraces of the mother, but from the cruelty of the monster; and it is so far true of England, that the same tyranny which drove the first emigrants from home, pursues their descendants still." Thomas Paine, Common Sense, 1776 The ideas expressed in the excerpt best reflect which of the following historical processes? A The desire to fund transportation improvements to settle the interior of North America B The persistence of aspects of African culture among enslaved people C The creation of alliances between Europeans and Native Americans D The transmission of Enlightenment ideals across the Atlantic

D The transmission of Enlightenment ideals across the Atlantic Writings like those of Paine and John Locke reflect the emergence of a trans-Atlantic print culture that resulted in the exchange of ideas that challenged European monarchies.

"The next matter I shall recommend to you is the providing more effectively for the security of your frontiers against [American] Indians, who notwithstanding the many parties of Rangers [militia, or local men who volunteered for colonial defense] have . . . killed and carried off at least twenty of our outward inhabitants and Indian allies; I have attempted by several ways to oppose those [invasions] but after some trouble and expense have only experienced that our people are not ready for warlike undertakings. . . . The [condition of our Indian allies has] of late approved themselves to be ready and faithfully allied, and I am persuaded that setting them along our frontiers without all our inhabitants . . . would be a better and cheaper safeguard to the country than the old method of Rangers." Virginia Governor Alexander Spotswood, addressing the members of the House of Burgesses, 1713 The point of view expressed by the Virginia governor in the excerpt is that he A opposes the expansion of colonial settlements into western lands where White settlers could encounter violent opposition from America Indians B believes that White Virginians must learn to protect themselves if they are to expand their settlements C values the lives of White Virginians equally with those of allied American Indians D feels a responsibility to protect White frontier settlers from violent encounters with American Indians

D feels a responsibility to protect White frontier settlers from violent encounters with American Indians Spotswood's narrative and proposal focus on better protecting White settlers from violent clashes in frontier areas.

"'I ordered my company to fire,' [George] Washington reported. . . . This incident . . . led to massive French retaliation and the outbreak of what was soon a world war. It raged in North America for six years, 1754-60, in Central and South America, in the Caribbean and the Atlantic, in India and the East, and not least in Europe, where it was known as the Seven Years War (1756-63). . . . Horace Walpole [stated]: 'The volley fired by a young Virginian in the backwoods of America set the world on fire.'" Paul Johnson, historian, A History of the American People, 1997 Which of the following contributed to the outbreak of the Seven Years' War (French and Indian War) in North America? A Intensified competition between France and Britain over colonies B Efforts by Britain to monopolize tobacco sales in Europe C Spanish attempts to end British control of the trans-Atlantic slave trade D Britain's desire to enact new taxes on Native American nations

A Intensified competition between France and Britain over colonies Increasing competition over territory, particularly in the Great Lakes area, resulted in the outbreak of combat between British and French forces in North America.

Developments such as that depicted in the image most directly led to which of the following? A The importation of enslaved Africans to the Caribbean B The increasing use of indentured servants in the Caribbean C The spread of Spanish missionaries into portions of Florida D The settling of the eastern North American seaboard by the Spanish

A The importation of enslaved Africans to the Caribbean To meet the growing demand for sugar cultivation, the Spanish began importing enslaved Africans to the Caribbean to labor on sugar plantations and in sugar production. The image depicts the beginning of this transition to enslaved African labor in the Caribbean.

The trend from 1660 to 1710 depicted in the table most strongly indicates which of the following? A The rise of the trans-Atlantic trade of enslaved Africans B The increase in the practice of indentured servitude C The escalation of conflict between the British and Native Americans D The continued Anglicization in the British colonies

A The rise of the trans-Atlantic trade of enslaved Africans The table depicts a steady rise in the Black population, especially in the West Indies, due to the development of an Atlantic economy that increasingly relied on the labor of enslaved Africans.

Which of the following most directly contributed to the development depicted in the image? A The emerging European naval capabilities in the Caribbean B The search for new sources of wealth in the Caribbean C The North American reliance on imports from the Caribbean D The spread of diseases native to the Americas to enslaved African plantation laborers

B The search for new sources of wealth in the Caribbean The search for new sources of wealth led the Spanish and other nations to establish plantations for the cultivation of sugar and other crops throughout the Caribbean.

"In Carolina, the instances of Negroes murdering . . . their own masters or overseers are not rare . . . . [Runaways] escape by water, past Frederica [in Georgia] to St. Augustine [in Florida], where they receive freedom, be it war or peace [with Spain]. Many just run into the woods, get along miserably, [or] are secretly looked after by other Negroes. . . . "Those Negroes who have served the [colony of Georgia] well are bought and freed by the government, receive their own land, and enjoy the English rights. If a private party wants to release a Negro he must have the consent of the governor or get him out of the colony. For the free Negroes abuse their freedom, and it is feared they seduce others [to freedom]. . . . ". . . Mixings or marriages [between Black and White colonists] are not allowed by the laws; but . . . I have learned of 2 white women, one French and one German, who have secretly been with Negroes and have borne black children. . . . And all too common [are] white men . . . [who with Negro women] father half-black children. [The children] are perpetual slaves just like their mothers." Johann Martin Bolzius, German minister, report to a correspondent in Europe on life in Georgia and the Carolinas, 1751 The experience of enslaved people in the southern British colonies as described in the excerpt was similar to the experience of enslaved people in the northern British colonies because A many enslaved people in both regions had rights that were recognized under colonial law B many enslaved people in both regions developed strategies to resist slavery C most enslaved people in both regions performed skilled labor in port cities D most enslaved people in both regions had opportunities to earn their freedom

B many enslaved people in both regions developed strategies to resist slavery The experience of many enslaved people in both the northern and southern British colonies was similar because they developed strategies to resist slavery, which included work slow downs, running away, and violent revolts.

"Today, two hundred and fifty years after the French and Indian War, most Americans are no more familiar with its events and significance than they are with those of the Peloponnesian War. Few know that George Washington struck the first spark of a war that set the British North American frontier ablaze from the Carolinas to Nova Scotia, then spread to Europe, Canada, the Caribbean, West Africa, India, and, finally, the Philippines. Historians call this immense conflict the Seven Years' War; . . . Winston Churchill described it as 'the first world war.'" Fred Anderson, The War That Made America: A Short History of the French and Indian War, 2005 The conclusion of the Seven Years' War (French and Indian War) had which of the following effects on Native American societies? A Native Americans allied with Great Britain gained the right to become British citizens, angering the colonists. B British and French officials agreed to force Native Americans to move to reservations west of the Mississippi River. C Native Americans gained control of all of the western fur trade with British colonists. D The British government attempted to restrict western settlement to reduce tensions between colonists and Native Americans.

D The British government attempted to restrict western settlement to reduce tensions between colonists and Native Americans. In order to reduce conflicts between Native Americans and colonists, the British government passed but failed to enforce the Proclamation of 1763, prohibiting further westward settlement in the colonies.

"In Carolina, the instances of Negroes murdering . . . their own masters or overseers are not rare . . . . [Runaways] escape by water, past Frederica [in Georgia] to St. Augustine [in Florida], where they receive freedom, be it war or peace [with Spain]. Many just run into the woods, get along miserably, [or] are secretly looked after by other Negroes. . . . "Those Negroes who have served the [colony of Georgia] well are bought and freed by the government, receive their own land, and enjoy the English rights. If a private party wants to release a Negro he must have the consent of the governor or get him out of the colony. For the free Negroes abuse their freedom, and it is feared they seduce others [to freedom]. . . . ". . . Mixings or marriages [between Black and White colonists] are not allowed by the laws; but . . . I have learned of 2 white women, one French and one German, who have secretly been with Negroes and have borne black children. . . . And all too common [are] white men . . . [who with Negro women] father half-black children. [The children] are perpetual slaves just like their mothers." Johann Martin Bolzius, German minister, report to a correspondent in Europe on life in Georgia and the Carolinas, 1751 The reaction to the situation described in the third paragraph represented a continuity with which of the following earlier colonial developments? A A strict racial system was established that separated enslaved people from European colonists. B Some enslaved Native Americans were incorporated into colonial society. C Some colonists used Enlightenment principles to argue for racial equality between Europeans and Africans. D The institution of slavery was transferred to colonial America unaltered from West Africa.

A A strict racial system was established that separated enslaved people from European colonists. The reaction against ethnic mixing in the third paragraph of the excerpt represented a continuity with the racial system that developed in British America in the later seventeenth century that enforced the separation of Europeans and enslaved Africans.

"His Catholic Majesty [of Spain] and the United States of America desiring to consolidate on a permanent basis the Friendship and good correspondence which happily prevails between the two Parties, have determined to establish by a convention several points. . . . "Article IV "It is likewise agreed that the Western boundary of the United States which separates them from the Spanish Colony of Louisiana, is in the middle of the channel or bed of the River Mississippi . . . ; and his Catholic Majesty has likewise agreed that the navigation of the said River in its whole breadth from its source to the Ocean shall be free only to his Subjects, and the Citizens of the United States, unless he should extend this privilege to the Subjects of other Powers by special convention. . . . "Article XXII "The two high contracting Parties hoping that the good correspondence and friendship which happily reigns between them will be further increased by this Treaty, and that it will contribute to augment their prosperity and opulence, will in future give to their mutual commerce all the extension and favor which the advantage of both Countries may require; . . . his Catholic Majesty will permit the Citizens of the United States for the space of three years from this time to deposit their merchandise and effects in the Port of New Orleans." Treaty of Friendship, Limits, and Navigation Between Spain and the United States, known as Pinckney's Treaty or the Treaty of San Lorenzo, 1795 The agreements made in the excerpt best reflect which of the following concerns in the United States during this period? A How to support western settlers beyond the Appalachian Mountains B How to avoid the establishment of political parties C Whether to assist the French in their war against Great Britain D Whether to allow slavery to expand into new territories

A How to support western settlers beyond the Appalachian Mountains In the 1790s, navigation rights to the Mississippi River and trade access to New Orleans were primary economic concerns for United States settlers living west of the Appalachian Mountains. These settlers relied on river routes to transport their produce and threatened war with Spain if they could not gain access to them.

Which of the following was the most important reason that Native American relations with English settlers differed from Native American relations with other groups of European settlers in the 1600s? A Larger numbers of English colonists settled on land taken from Native Americans. B English settlers were technologically more advanced than other European settlers. C Native Americans understood the English language better than other European languages. D English colonization along the Eastern Seaboard provided fewer opportunities for conflict between the two sides than did colonization in the interior.

A Larger numbers of English colonists settled on land taken from Native Americans. The larger number of English settlers than of settlers from other nations created more instances of tension with Native Americans, whose land was rapidly being populated by the English.

"The second chief and principal end [of colonization] . . . consists in the [sale] of the mass of our clothes and other commodities of England, and in receiving back of the needful commodities that we now receive from all other places of the world. . . . This one thing is to be done, without which it were in vain to go about this; and that is the matter of planting [colonies] and fortification. . . . We are to plant upon the mouths of the great navigable rivers which are there [in America], by strong order of fortification, and there to plant our colonies. . . . And these fortifications shall keep the [native] people of [America] in obedience and good order. . . . ". . . Without this planting in due time, we shall never be able to have full knowledge of the language, manners, and customs of the people of those regions. . . . And although by other means we might attain to the knowledge thereof, yet being not there fortified and strongly seated, the French that swarm with [a] multitude of people, or other nations, might secretly fortify and settle themselves before us." Richard Hakluyt, English government official, A Discourse on Western Planting, 1584 Hakluyt's call for the English to learn about Native American "language, manners, and customs" best represents which of the following developments in the 1500s? A Native Americans and Europeans partnered for trade. B Europeans introduced maize cultivation to the Americas. C Native Americans were sent in large numbers into slavery in Europe. D Europeans refused defensive military alliances with Native Americans.

A Native Americans and Europeans partnered for trade. In order to pursue commercial relationships with Native Americans, European officials such as Hakluyt advocated that Europeans learn Native American languages and study Native American cultures.

"In Carolina, the instances of Negroes murdering . . . their own masters or overseers are not rare . . . . [Runaways] escape by water, past Frederica [in Georgia] to St. Augustine [in Florida], where they receive freedom, be it war or peace [with Spain]. Many just run into the woods, get along miserably, [or] are secretly looked after by other Negroes. . . . "Those Negroes who have served the [colony of Georgia] well are bought and freed by the government, receive their own land, and enjoy the English rights. If a private party wants to release a Negro he must have the consent of the governor or get him out of the colony. For the free Negroes abuse their freedom, and it is feared they seduce others [to freedom]. . . . ". . . Mixings or marriages [between Black and White colonists] are not allowed by the laws; but . . . I have learned of 2 white women, one French and one German, who have secretly been with Negroes and have borne black children. . . . And all too common [are] white men . . . [who with Negro women] father half-black children. [The children] are perpetual slaves just like their mothers." Johann Martin Bolzius, German minister, report to a correspondent in Europe on life in Georgia and the Carolinas, 1751 Which of the following represented a change in the labor force of the southern British colonies by the 1700s as depicted in the excerpt? A Slavery became more widespread than indentured servitude. B The rise of manufacturing led to an increased need for skilled artisans and craftsmen. C The encomienda was adopted from the Spanish to provide Native American workers. D People newly freed from slavery frequently worked on plantations.

A Slavery became more widespread than indentured servitude. By 1700, the southern colonies in British North America had largely shifted from using European indentured servants to using enslaved Africans to provide labor for plantation agriculture.

"The next matter I shall recommend to you is the providing more effectively for the security of your frontiers against [American] Indians, who notwithstanding the many parties of Rangers [militia, or local men who volunteered for colonial defense] have . . . killed and carried off at least twenty of our outward inhabitants and Indian allies; I have attempted by several ways to oppose those [invasions] but after some trouble and expense have only experienced that our people are not ready for warlike undertakings. . . . The [condition of our Indian allies has] of late approved themselves to be ready and faithfully allied, and I am persuaded that setting them along our frontiers without all our inhabitants . . . would be a better and cheaper safeguard to the country than the old method of Rangers." Virginia Governor Alexander Spotswood, addressing the members of the House of Burgesses, 1713 Which of the following best describes a purpose of the excerpt? A The Virginia governor is seeking support from the colonial legislature for his plan to address conflict between settlers and American Indians in frontier areas. B The Virginia governor is warning American Indians that he will authorize military action if they continue assaults on settlers. C The Virginia governor is seeking pardon from the House of Burgesses for his inability to protect them from violent conflict with American Indians. D The Virginia governor is soliciting the support of allied American Indians for his plan to provide security in the West.

A The Virginia governor is seeking support from the colonial legislature for his plan to address conflict between settlers and American Indians in frontier areas. The governor is describing a plan to address conflict between settlers and American Indians in frontier areas to the members of the House of Burgesses because he wants their support to carry it out.

"Threatened by popular political victories [in the states] and widespread resistance, many elite Pennsylvanians launched an effort to remake the state and national governments so that they were less democratic. . . . Popular policies and resistance . . . threatened elite ideals. . . . Popular calls for a revaluation of war debt certificates, bans on for-profit corporations, progressive taxation, limits on land speculation, and every other measure designed to make property more equal promised to take wealth away from the elite. . . . It was also threatening that popular politics frightened off potential European investors. . . . [They] were alarmed by the Pennsylvania legislature's 1785 [cancellation] of the Bank of North America's corporate charter. . . . . . . The push for the Constitution was based in part on the belief that state governments across the new nation had been too democratic and, as a result, had produced policies . . . that threatened elite interests. Most of the men who assembled at the Constitutional Convention in Philadelphia in 1787 were also convinced that the national government under the Articles of Confederation was too weak to counter the rising tide of democracy in the states." Terry Bouton, historian, Taming Democracy: "The People," the Founders, and the Troubled Ending of the American Revolution, 2007 The relationship established between the federal government and the states under the United States Constitution was a long-term response to which of the following earlier developments? A The claims of parliamentary authority over colonial legislatures B George Washington's warning against the formation of permanent alliances C Frontier conflicts between Native Americans and British colonists D Democratic-Republican opposition to Alexander Hamilton's economic policies

A The claims of parliamentary authority over colonial legislatures The establishment of a federal system under the Constitution, whereby specific government powers were designated for the central government and others reserved for the states, was a response to the earlier conflict between the British North American colonies and the British Parliament. In the 1760s and 1770s, the central government of the British Empire in London claimed all power to pass laws for the colonies and to overrule the colonial legislatures, and the Constitution was intended to prevent a repeat of this kind of conflict.

"[In Virginia] the Negroes live in small cottages called quarters . . . under the direction of an overseer, who takes care that they tend such land as the owner allots and orders. . . . Their greatest hardship [is] consisting in that they and their posterity are not at their own liberty or disposal, but the property of their owners. . . . The children belong to the master of the woman that bears them. . . . "[The] abundance of [the] English entertain . . . that they are all fools and beggars that live in any [other] country but theirs. This home fondness has been very prejudicial [harmful] to the common sort of English, and has in a great measure [slowed] the plantations from being stocked with such inhabitants as are skillful, industrious, and laborious. . . . "These [English] servants are but an insignificant number, when compared with the vast shoals [mass] of Negroes who are employed as slaves there to do the hardest and most part of the work." Hugh Jones, The Present State of Virginia, 1724 The development described in the excerpt represented which of the following long-term trends in Virginia? A The hardening of racial divisions B The oversupply of indentured laborers C The dominance of subsistence farming D The Anglicization of colonial culture

A The hardening of racial divisions The excerpt describes the decrease of indentured servitude, the increase in African slavery in Virginia, and the inherited nature of chattel slavery in the colony, which represents an aspect of the hardening of racial divisions in colonial Virginian society.

"To understand political power right, and derive it from its original, we must consider what state all men are naturally in, and that is, a state of perfect freedom to order their actions, and dispose of their possessions and persons, as they think fit, within the bounds of the law of nature, without asking leave, or depending upon the will of any other man. "A state also of equality, wherein all the power and jurisdiction is reciprocal, no one having more than another. . . . "The state of nature has a law of nature to govern it which obliges every one . . . that being all equal and independent, no one ought to harm another in his life, health, liberty, or possessions." John Locke, Two Treatises of Government, 1689 The excerpt from Locke's Two Treatises of Government could best be used as evidence by historians studying which of the following topics? A The impact of the Enlightenment on Revolutionary political thought B The use of Revolutionary pamphlets to mobilize colonial resistance to British policies C The attempts by African Americans to seek freedom during the American Revolution D The influence of the American Revolution on ideas about republicanism overseas

A The impact of the Enlightenment on Revolutionary political thought Many historians have cited the transmission of Enlightenment thinkers such as Locke as inspiring revolutionary thought in Britain's North American colonies.

"The New England settlers more closely resembled the non-migrating English population than they did other English colonists in the New World. . . . While the composition of the emigrant populations in the Chesapeake and the Caribbean hindered the successful transfer of familiar patterns of social relationships, the character of the New England colonial population ensured it. The prospect of colonizing distant lands stirred the imaginations of young people all over England but most of these young adults made their way to the tobacco and sugar plantations of the South. Nearly half of a sample of Virginia residents in 1625 were between the ages of twenty and twenty-nine, and groups of emigrants to the Chesapeake in the seventeenth century consistently included a majority of people in their twenties. In contrast, only a quarter of the New England settlers belonged to this age group. "Similarly, the sex ratio of the New England emigrant group resembled that of England's population. If women were . . . scarce in the Chesapeake . . . they were comparatively abundant in the northern colonies. In the second decade of Virginia's settlement, there were four or five men for each woman; by the end of the century, there were still about three men for every two women. Among the emigrants [in New England], however, nearly half were women and girls. Such a high proportion of females in the population assured the young men of New England greater success than their southern counterparts in finding spouses." Virginia DeJohn Anderson, historian, "Migrants and Motives: Religion and the Settlement of New England, 1630-1640," published in 1985 Which of the following best describes an overall argument of the excerpt? A The makeup of emigrant populations led to greater reconstruction of English family life in New England than it did in the Chesapeake. B British colonial settlers were disproportionately young and male compared to the overall population of England. C Tobacco and sugar plantations in the southern colonies attracted the most settlers in British North America. D By the end of the seventeenth century, the population makeup of the British colonies resembled the population makeup of England.

A The makeup of emigrant populations led to greater reconstruction of English family life in New England than it did in the Chesapeake. The excerpt argues that the age and sex structure of the New England emigrant population ensured the "successful transfer of familiar patterns of social relationships," unlike the age and sex structure of the Chesapeake settler population.

"On the western side of the ocean, movements of people and ideas . . . preceded the Atlantic connection. Great empires—in the Valley of Mexico, on the Mississippi River . . . —had collapsed or declined in the centuries before 1492. . . . As Columbus embarked on his first transatlantic voyage, the Mexica, or Aztecs, were consolidating their position [in Mexico]; their city was a center of both trade and military might. Tenochtitlán [the Aztec capital] . . . held 200,000 people, a population greater than in the largest city in contemporary Europe. ". . . The Mississippian culture spread east and west from its center, the city of Cahokia, on the Mississippi River near the site of modern St. Louis. It was a successor to earlier cultures, evidence of which can be seen in the great ceremonial mounds they built. Cahokia declined and was ultimately abandoned completely in the later thirteenth century. . . . Throughout the Southeast, smaller mound-building centers continued." Karen Ordahl Kupperman, historian, The Atlantic in World History, 2012 Which of the following best characterizes the Mississippian societies described in the excerpt? A They had mixed agricultural and hunter-gatherer economies that favored the development of permanent villages. B They were nomadic peoples who utilized river systems to move throughout the region. C They lived in isolated, impermanent communities and left very little trace of their presence once a settlement had been abandoned. D They used advanced agricultural practices like irrigation to support economic growth.

A They had mixed agricultural and hunter-gatherer economies that favored the development of permanent villages. Mississippians had mixed agricultural and hunter-gatherer economies that allowed the development of permanent villages and cities such as Cahokia.

"I have heard it asserted by some, that as America hath flourished under her former connection with Great Britain, that the same connection is necessary towards her future happiness, and will always have the same effect. Nothing can be more fallacious [untrue] than this kind of argument. We may as well assert that because a child has thrived upon milk, that it is never to have meat, or that the first twenty years of our lives is to become a precedent for the next twenty. But even this is admitting more than is true, for I answer roundly, that America would have flourished as much, and probably much more, had no European power had any thing to do with her. . . . "But Britain is the parent country, say some. Then the more shame upon her conduct. Even brutes do not devour their young, nor savages make war upon their families. . . . Europe, and not England, is the parent country of America. This new world hath been the asylum for the persecuted lovers of civil and religious liberty from every part of Europe. Hither have they fled, not from the tender embraces of the mother, but from the cruelty of the monster; and it is so far true of England, that the same tyranny which drove the first emigrants from home, pursues their descendants still." Thomas Paine, Common Sense, 1776 Paine's rhetoric in the excerpt would have most likely been interpreted at the time as promoting the A independence of the American colonies B sale of British colonies to France C elimination of the trans-Atlantic slave trade to the colonies D election of representatives from the colonies to Parliament

A independence of the American colonies Paine's arguments were widely used to support advocacy for complete independence of the North American colonies from Great Britain.

"To understand political power right, and derive it from its original, we must consider what state all men are naturally in, and that is, a state of perfect freedom to order their actions, and dispose of their possessions and persons, as they think fit, within the bounds of the law of nature, without asking leave, or depending upon the will of any other man. "A state also of equality, wherein all the power and jurisdiction is reciprocal, no one having more than another. . . . "The state of nature has a law of nature to govern it which obliges every one . . . that being all equal and independent, no one ought to harm another in his life, health, liberty, or possessions." John Locke, Two Treatises of Government, 1689 During the period of the American Revolution, Locke's point of view in the excerpt would most likely have been interpreted as promoting a form of government based on A natural rights B military rule C hereditary privilege D religious beliefs

A natural rights Locke's statement in the excerpt that "we must consider what state all men are naturally in" and that men are bound by the laws of nature promoted the concept of a government based on natural rights.

"The emancipation of slaves in New England, beginning around 1780, was a gradual process, whether by post nati statute [laws freeing enslaved people born after a certain date], as in Rhode Island and Connecticut, or by effect, as in Massachusetts and New Hampshire, where ambiguous judicial decisions and constitutional interpretations discouraged slaveholding without clearly outlawing it. The gradual nature of the process encouraged Whites to transfer a language and set of practices shaped in the context of slavery to their relations with a slowly emerging population of free people of color. The rhetoric of antislavery and revolutionary republicanism fostered this transfer, undergirding Whites' assumptions that emancipated slaves, likely to be dependent and disorderly, would constitute a problem requiring firm management in the new republic. . . . "Even more problematic was the promise implicit in antislavery rhetoric that abolition, by ending 'the problem'—the sin of slavery and the troublesome presence of slaves—would result in the eventual absence of people of color themselves. In other words, Whites anticipated that free people of color, would, by some undefined moment (always imminent), have disappeared." Joanne Pope Melish, historian, Disowning Slavery: Gradual Emancipation and "Race" in New England, 1780-1860, published in 1998 The author claims in the excerpt that antislavery rhetoric in the late eighteenth century was based on A the belief that emancipated people would not be a presence in society B religious ideals formed during the First Great Awakening C legal precedent established during the colonial period D the concern that revolutionary conflict would spread from France to the United States

A the belief that emancipated people would not be a presence in society The author argues that antislavery rhetoric was based on the idea that "people of color would . . . have disappeared" after emancipation.

"[Before European contact] Cahokia [in present-day Missouri] and such other major centers as those now known as Coosa and Etowah in Georgia, Moundville in Alabama, and Natchez in Mississippi were home to highly stratified societies, organized as chiefdoms and characterized by a sharp divide between elites and commoners. . . . Surrounding networks of agricultural hamlets provided food to support the urban centers. . . . "From the Ohio River through most of present-day Canada and down the coast to the Chesapeake were speakers of Algonquian languages. . . . Nearly everywhere [here], villages composed of 500 to 2,000 people were the norm. . . . "[This] Indian country was decentralized and diverse, but not disconnected. . . . Routes of trade and communication, most of them millennia old and following the great river systems, crisscrossed the continent. The goods that moved along them were, for the most part, few and rare. . . . Some closely neighboring people might exchange crucial resources—corn, for instances, for meat or fish." Daniel K. Richter, historian, Facing East from Indian Country: A Native History of Early America, 2001 Which of the following most supported the development of the commerce described in the third paragraph? A Maize cultivation spread northward from Mexico. B Native Americans constructed extensive road networks. C Native Americans in the Northeast of North America formed extensive empires. D Metal tools became valuable objects in Native American religions.

A- Maize cultivation spread northward from Mexico. The spread of maize cultivation north from present-day Mexico in preceding centuries supported economic development, settlement, and social diversification.

"[Before European contact] Cahokia [in present-day Missouri] and such other major centers as those now known as Coosa and Etowah in Georgia, Moundville in Alabama, and Natchez in Mississippi were home to highly stratified societies, organized as chiefdoms and characterized by a sharp divide between elites and commoners. . . . Surrounding networks of agricultural hamlets provided food to support the urban centers. . . . "From the Ohio River through most of present-day Canada and down the coast to the Chesapeake were speakers of Algonquian languages. . . . Nearly everywhere [here], villages composed of 500 to 2,000 people were the norm. . . . "[This] Indian country was decentralized and diverse, but not disconnected. . . . Routes of trade and communication, most of them millennia old and following the great river systems, crisscrossed the continent. The goods that moved along them were, for the most part, few and rare. . . . Some closely neighboring people might exchange crucial resources—corn, for instances, for meat or fish." Daniel K. Richter, historian, Facing East from Indian Country: A Native History of Early America, 2001 Which of the following best characterizes the process described in the first paragraph of the excerpt? A Native American agriculture encouraged the growth of socially diversified urban areas. B An arid climate led Native Americans to construct irrigation projects in the Mississippi River valley. C The climate in North America discouraged the development of large settlements. D Military power helped Native American hunters and gathers to conquer expansive cities.

A- Native American agriculture encouraged the growth of socially diversified urban areas. The excerpt describes the development of major population centers. Prior to contact with Europeans, Native Americans in the Mississippi River valley developed large communities and highly stratified societies.

"I have heard it asserted by some, that as America hath flourished under her former connection with Great Britain, that the same connection is necessary towards her future happiness, and will always have the same effect. Nothing can be more fallacious [untrue] than this kind of argument. We may as well assert that because a child has thrived upon milk, that it is never to have meat, or that the first twenty years of our lives is to become a precedent for the next twenty. But even this is admitting more than is true, for I answer roundly, that America would have flourished as much, and probably much more, had no European power had any thing to do with her. . . . "But Britain is the parent country, say some. Then the more shame upon her conduct. Even brutes do not devour their young, nor savages make war upon their families. . . . Europe, and not England, is the parent country of America. This new world hath been the asylum for the persecuted lovers of civil and religious liberty from every part of Europe. Hither have they fled, not from the tender embraces of the mother, but from the cruelty of the monster; and it is so far true of England, that the same tyranny which drove the first emigrants from home, pursues their descendants still." Thomas Paine, Common Sense, 1776 Which of the following historical situations most directly shaped Paine's argument that Britain's policies were economically harming its colonies? A Increased immigration to the Americas B Continued enforcement of mercantilism C Limitations on indentured servitude D Demand for raw materials

B Continued enforcement of mercantilism Paine is arguing that, although "America hath flourished under her former connection with Great Britain," the limitations caused by mercantilist policies (which compares to a child being fed milk instead of meat) harmed the colonies' economies.

Today, two hundred and fifty years after the French and Indian War, most Americans are no more familiar with its events and significance than they are with those of the Peloponnesian War. Few know that George Washington struck the first spark of a war that set the British North American frontier ablaze from the Carolinas to Nova Scotia, then spread to Europe, Canada, the Caribbean, West Africa, India, and, finally, the Philippines. Historians call this immense conflict the Seven Years' War; . . . Winston Churchill described it as 'the first world war.'" Fred Anderson, The War That Made America: A Short History of the French and Indian War, 2005 Which of the following best explains a result of the Seven Years' War (French and Indian War) ? A France sold the Louisiana Territory to Great Britain. B Great Britain gained a claim to land extending to the Mississippi River. C Great Britain and Spain established an alliance. D British influence over its North American colonies decreased.

B Great Britain gained a claim to land extending to the Mississippi River. In the treaty that ended the Seven Years' War, France transferred its North American territorial claims east of the Mississippi River to Great Britain.

"The isolation of the [native peoples] of the Americas . . . from Old World germs prior to the last few hundred years was nearly absolute. Not only did very few people of any origin cross the great oceans, but those who did must have been healthy or they would have died on the way, taking their pathogens with them. . . . [Native Americans] were not without their own infections, of course. [But Native Americans] seem to have been without any experience with such Old World maladies as smallpox [and] measles. . . . "Indications of the susceptibility of [Native Americans] . . . to Old World infections appear almost immediately after the intrusion of the whites. In 1492, Columbus kidnapped a number of [Arawak Indians] to train as interpreters and to show to King Ferdinand and Queen Isabella. Several of them seem to have died on the stormy voyage to Europe [in 1493]. . . . In 1495, Columbus . . . sent 550 [Arawak] slaves . . . off across the Atlantic. . . to be put to work in Spain. The majority of these soon were also dead. . . . ". . . What killed the Arawaks in 1493 and 1495? . . . Columbus certainly did not want to kill his interpreters, and slavers and slaveholders have no interest whatever in the outright slaughter of their property. . . . The most likely candidates for the role of exterminator of the first [Native Americans] in Europe were those that killed so many other Arawaks in the decades immediately following: Old World pathogens." Alfred W. Crosby, historian, Ecological Imperialism: The Biological Expansion of Europe, 900-1900, published in 1986 In the excerpt, Crosby makes which of the following claims about the transmission of Old World diseases to the Americas? A It had minimal effect on Native Americans. B It was an unintended consequence of contact between the New World and the Old World. C It was significant in the centuries prior to Columbus' arrival in the Americas. D It was a deliberate act on the part of Columbus.

B It was an unintended consequence of contact between the New World and the Old World. In the first paragraph of the excerpt, Crosby claims that because Native Americans had previously had little to no exposure to Europeans, they were susceptible to pandemic spread of Old World diseases.

"Every British Subject born on the continent of America . . . is by the law of God and nature, by the common law, and by act of parliament, . . . entitled to all the natural, essential, inherent and inseparable rights of our fellow subjects in Great-Britain. Among those rights are the following . . . : ". . . Taxes are not to be laid on the people, but by their consent in person, or by [representatives]. ". . . I can see no reason to doubt, but that the imposition of taxes, whether on trade, or on land, or houses, or ships, . . . in the colonies is absolutely irreconcilable with the rights of the Colonists, as British subjects. . . . "The power of parliament is uncontrollable, . . . and we must obey. . . . Therefore let the parliament lay what burthens they please on us, we must, it is our duty to submit and patiently bear them till they . . . afford us relief by repealing such acts, as through mistake, or other human infirmities, have been suffered to pass, if they can be convinced that their proceedings are not constitutional." James Otis, The Rights of the British Colonies Asserted and Proved, pamphlet, 1764 By the 1770s, to which of the following groups would Otis' argument that the colonies "must obey" Parliament most appeal? A Farmers in the New England colonies B Loyalists in New York C Enslaved people in the southern colonies D Artisans in port cities

B Loyalists in New York By the 1770s, Loyalists, many of whom were concentrated in New York, most supported the argument that the British North American colonies were bound to obey Parliament.

"In the time of the late war, being desirous to defend, secure, and promote the Rights and Liberties of the people, we spared no pains but freely granted all the aid and assistance of every kind that our civil fathers [political leaders] required of us. "We are sensible also that a great debt is justly brought upon us by the War, and we are as willing to pay our share towards it as we are to enjoy our shares in independency. . . . "But with the greatest submission we beg leave to inform your Honors that unless something takes place more favorable to the people, in a little time at least one half of our inhabitants in our opinion will become bankrupt. . . . When we compute the taxes laid upon us the five preceding years, the State and County, town, and class taxes, the amount is equal to what our farms will rent for. Sirs in this situation, what have we to live on: No money to be had; our estates daily posted and sold. . . . Surely your Honors are no strangers to the distresses of the people but do know that many of our good inhabitants are now confined in jail for debt and for taxes. . . . Will not the people in the neighboring states say of this state: although the Massachusetts [people] boast of their fine Constitution, their government is such that it devours their inhabitants? ". . . If your Honors find anything above mentioned worthy of notice, we earnestly pray that . . . [the state legislature] would point out some way whereby the people might be relieved." Petition from the town of Greenwich to the Massachusetts state legislature, 1786 Which of the following evidence used in the petition supports the claim that the Massachusetts government "devours their inhabitants"? A Many people living in Massachusetts fought in the Revolutionary War. B Many Massachusetts farmers were held in debtor's prison. C Massachusetts property owners typically rented land to tenants. D Massachusetts incurred a state debt during the American Revolution.

B Many Massachusetts farmers were held in debtor's prison. Among the list of developments that the petition used as evidence of how the Massachusetts government had harmed Greenwich was the imprisonment of people in debtor's prison, mentioned in the third paragraph.

"Barbarians [are] . . . , in the proper and strict sense of the word, dull witted and lacking in the reasoning powers necessary for self-government. They are without laws, without king, etc. For this reason they are by nature unfitted for rule. "[Some] barbarians . . . have a lawful, just, and natural government. Even though they lack the art and use of writing, they are not wanting in the capacity and skill to rule and govern themselves. . . . Thus they have kingdoms, communities, and cities that they govern wisely according to their laws and customs. . . . ". . . It does not necessarily follow that [Native Americans] are incapable of government and have to be ruled by others, except to be taught about the Catholic faith and to be admitted to the holy sacraments. They are not ignorant, inhuman, or bestial. Rather, long before they had heard the word Spaniard they had properly organized states, wisely ordered by excellent laws, religion, and custom. They cultivated friendship and . . . lived together in populous cities in which they wisely administered the affairs of both peace and war justly and equitably, truly governed by laws that at very many points surpass ours. . . . ". . . [Was] the war of the Romans against the [ancient] Spanish justified in order to free them from barbarism? . . . Do you think that the Romans, once they had [conquered] the wild and barbaric peoples of Spain, could with secure right divide all of you [Spaniards] among themselves [in encomiendas] . . . ? And do you then conclude that the Romans could have stripped your rulers of their authority and consigned all of you, after you had been deprived of your liberty, to wretched labors, especially in searching for gold and silver [mines]. . . ? Is this the way to impose the yoke of Christ on Christian men?" Bartolomé de Las Casas, Spanish Catholic religious leader, In Defense of the Indians, circa 1550 Which of the following claims in the first and second paragraph of the excerpt did Las Casas use to support his overall argument about the capabilities of Native Americans? A Spain's behavior toward Native Americans could be called "barbarian." B Native American societies did not meet the definition of "barbarian." C Spain's European rivals were more like "barbarians" than Native Americans were. D Native Americans believed that the Spanish were "barbarians."

B Native American societies did not meet the definition of "barbarian." Las Casas used the claim in the excerpt that Native Americans did not fit the definition of "barbarian" as evidence for the argument that the Spanish should respect their governments and not abuse them as laborers.

In which of the following ways did the Spanish impose racial hierarchies in the regions of the Americas that they controlled during the 1500s and 1600s?

B The Spanish created a caste system that incorporated people of European, Native American and African descent. The Spanish developed a caste system defining each person's social level based on racial backgrounds in which Native Americans and African Americans ranked below Europeans in order to regulate a diverse mixed-raced population.

"To understand political power right, and derive it from its original, we must consider what state all men are naturally in, and that is, a state of perfect freedom to order their actions, and dispose of their possessions and persons, as they think fit, within the bounds of the law of nature, without asking leave, or depending upon the will of any other man. "A state also of equality, wherein all the power and jurisdiction is reciprocal, no one having more than another. . . . "The state of nature has a law of nature to govern it which obliges every one . . . that being all equal and independent, no one ought to harm another in his life, health, liberty, or possessions." John Locke, Two Treatises of Government, 1689 Interpretations of Locke's assertions regarding a "state of freedom" and a "state also of equality" most directly influenced which of the following? A The grievances of American colonists during the Seven Years' War (French and Indian War) B The political rhetoric of Patriots during the American Revolution C The end of American involvement in the international slave trade D The demands by colonists to settle beyond the current frontier boundaries

B The political rhetoric of Patriots during the American Revolution The political rhetoric of Patriots during the American Revolution was strongly influenced by the concept of natural rights and freedom from tyranny.

"The New England settlers more closely resembled the non-migrating English population than they did other English colonists in the New World. . . . While the composition of the emigrant populations in the Chesapeake and the Caribbean hindered the successful transfer of familiar patterns of social relationships, the character of the New England colonial population ensured it. The prospect of colonizing distant lands stirred the imaginations of young people all over England but most of these young adults made their way to the tobacco and sugar plantations of the South. Nearly half of a sample of Virginia residents in 1625 were between the ages of twenty and twenty-nine, and groups of emigrants to the Chesapeake in the seventeenth century consistently included a majority of people in their twenties. In contrast, only a quarter of the New England settlers belonged to this age group. "Similarly, the sex ratio of the New England emigrant group resembled that of England's population. If women were . . . scarce in the Chesapeake . . . they were comparatively abundant in the northern colonies. In the second decade of Virginia's settlement, there were four or five men for each woman; by the end of the century, there were still about three men for every two women. Among the emigrants [in New England], however, nearly half were women and girls. Such a high proportion of females in the population assured the young men of New England greater success than their southern counterparts in finding spouses." Virginia DeJohn Anderson, historian, "Migrants and Motives: Religion and the Settlement of New England, 1630-1640," published in 1985 The first paragraph of the excerpt makes which of the following claims? A Emigrants to the Chesapeake came as family groups more often than emigrants to New England did. B The settlers of New England varied in age more than emigrants to the Chesapeake did. C The settlers of both New England and the Chesapeake were disproportionately young adults. D More young adults migrated to New England than to the Chesapeake colonies.

B The settlers of New England varied in age more than emigrants to the Chesapeake did. The excerpt explains that young adults predominated in groups of emigrants to the Chesapeake, whereas only a quarter of migrants to New England were in their twenties, making for a settlement population in New England that varied in age.

"The emancipation of slaves in New England, beginning around 1780, was a gradual process, whether by post nati statute [laws freeing enslaved people born after a certain date], as in Rhode Island and Connecticut, or by effect, as in Massachusetts and New Hampshire, where ambiguous judicial decisions and constitutional interpretations discouraged slaveholding without clearly outlawing it. The gradual nature of the process encouraged Whites to transfer a language and set of practices shaped in the context of slavery to their relations with a slowly emerging population of free people of color. The rhetoric of antislavery and revolutionary republicanism fostered this transfer, undergirding Whites' assumptions that emancipated slaves, likely to be dependent and disorderly, would constitute a problem requiring firm management in the new republic. . . . "Even more problematic was the promise implicit in antislavery rhetoric that abolition, by ending 'the problem'—the sin of slavery and the troublesome presence of slaves—would result in the eventual absence of people of color themselves. In other words, Whites anticipated that free people of color, would, by some undefined moment (always imminent), have disappeared." Joanne Pope Melish, historian, Disowning Slavery: Gradual Emancipation and "Race" in New England, 1780-1860, published in 1998 The author makes which of the following arguments in the excerpt about the perceptions Whites maintained regarding emancipated people in the North? A They expected emancipated people to assimilate into society. B They believed that emancipated people were unable to take care of themselves. C They hoped that emancipated people would provide a source of inexpensive labor. D They assumed that emancipated people would advocate for abolition in the South.

B They believed that emancipated people were unable to take care of themselves. The author claims that White people in the North generally believed that emancipated people were "likely to be dependent and disorderly."

"Every British Subject born on the continent of America . . . is by the law of God and nature, by the common law, and by act of parliament, . . . entitled to all the natural, essential, inherent and inseparable rights of our fellow subjects in Great-Britain. Among those rights are the following . . . : ". . . Taxes are not to be laid on the people, but by their consent in person, or by [representatives]. ". . . I can see no reason to doubt, but that the imposition of taxes, whether on trade, or on land, or houses, or ships, . . . in the colonies is absolutely irreconcilable with the rights of the Colonists, as British subjects. . . . "The power of parliament is uncontrollable, . . . and we must obey. . . . Therefore let the parliament lay what burthens they please on us, we must, it is our duty to submit and patiently bear them till they . . . afford us relief by repealing such acts, as through mistake, or other human infirmities, have been suffered to pass, if they can be convinced that their proceedings are not constitutional." James Otis, The Rights of the British Colonies Asserted and Proved, pamphlet, 1764 Which of the following was a major purpose of Otis' pamphlet? A To recruit foreign allies to support the claims of the colonists against Great Britain B To encourage opposition to Parliament's regulation of colonial commerce C To propose a plan to send colonial delegates to sit in Parliament D To advocate for colonial independence from Great Britain

B To encourage opposition to Parliament's regulation of colonial commerce As Otis described in the excerpt, he believed parliamentary taxation of colonial trade to violate the principles of the British constitution. When Otis wrote the pamphlet in 1764, Great Britain was attempting to more strictly enforce the Navigation Acts in North America by halting smuggling, collecting customs duties, and passing new laws such as the Sugar Act.

"His Catholic Majesty [of Spain] and the United States of America desiring to consolidate on a permanent basis the Friendship and good correspondence which happily prevails between the two Parties, have determined to establish by a convention several points. . . . "Article IV "It is likewise agreed that the Western boundary of the United States which separates them from the Spanish Colony of Louisiana, is in the middle of the channel or bed of the River Mississippi . . . ; and his Catholic Majesty has likewise agreed that the navigation of the said River in its whole breadth from its source to the Ocean shall be free only to his Subjects, and the Citizens of the United States, unless he should extend this privilege to the Subjects of other Powers by special convention. . . . "Article XXII "The two high contracting Parties hoping that the good correspondence and friendship which happily reigns between them will be further increased by this Treaty, and that it will contribute to augment their prosperity and opulence, will in future give to their mutual commerce all the extension and favor which the advantage of both Countries may require; . . . his Catholic Majesty will permit the Citizens of the United States for the space of three years from this time to deposit their merchandise and effects in the Port of New Orleans." Treaty of Friendship, Limits, and Navigation Between Spain and the United States, known as Pinckney's Treaty or the Treaty of San Lorenzo, 1795 Which of the following groups would have most likely supported the agreements made in the excerpt? A Fur trappers on the Pacific coast B White farmers in Kentucky C Merchants in New England D Native American leaders in Louisiana

B White farmers in Kentucky White Farmers who settled in new states such as Kentucky and other locations west of the Appalachian Mountains increasingly relied on the Mississippi River and its tributaries for the transportation of their goods to coastal markets via New Orleans. They would have supported the access this agreement gave them to the river and its outlet on the Gulf of Mexico.

"Barbarians [are] . . . , in the proper and strict sense of the word, dull witted and lacking in the reasoning powers necessary for self-government. They are without laws, without king, etc. For this reason they are by nature unfitted for rule. "[Some] barbarians . . . have a lawful, just, and natural government. Even though they lack the art and use of writing, they are not wanting in the capacity and skill to rule and govern themselves. . . . Thus they have kingdoms, communities, and cities that they govern wisely according to their laws and customs. . . . ". . . It does not necessarily follow that [Native Americans] are incapable of government and have to be ruled by others, except to be taught about the Catholic faith and to be admitted to the holy sacraments. They are not ignorant, inhuman, or bestial. Rather, long before they had heard the word Spaniard they had properly organized states, wisely ordered by excellent laws, religion, and custom. They cultivated friendship and . . . lived together in populous cities in which they wisely administered the affairs of both peace and war justly and equitably, truly governed by laws that at very many points surpass ours. . . . ". . . [Was] the war of the Romans against the [ancient] Spanish justified in order to free them from barbarism? . . . Do you think that the Romans, once they had [conquered] the wild and barbaric peoples of Spain, could with secure right divide all of you [Spaniards] among themselves [in encomiendas] . . . ? And do you then conclude that the Romans could have stripped your rulers of their authority and consigned all of you, after you had been deprived of your liberty, to wretched labors, especially in searching for gold and silver [mines]. . . ? Is this the way to impose the yoke of Christ on Christian men?" Bartolomé de Las Casas, Spanish Catholic religious leader, In Defense of the Indians, circa 1550 One piece of evidence Las Casas used to support his claim about Native American societies in the third paragraph is that they A did not practice Christianity B had developed large urban areas C sought friendly trading relations with the Spanish D were ignorant of Europe before they encountered the Spanish

B had developed large urban areas Las Casas used the claim that Native Americans lived in populous cities as evidence that they had civilized societies whose integrity should be respected by the Spanish. This claim reflected Las Casas' understanding of Native American peoples living in such places as Mexico and Peru.

"Threatened by popular political victories [in the states] and widespread resistance, many elite Pennsylvanians launched an effort to remake the state and national governments so that they were less democratic. . . . Popular policies and resistance . . . threatened elite ideals. . . . Popular calls for a revaluation of war debt certificates, bans on for-profit corporations, progressive taxation, limits on land speculation, and every other measure designed to make property more equal promised to take wealth away from the elite. . . . It was also threatening that popular politics frightened off potential European investors. . . . [They] were alarmed by the Pennsylvania legislature's 1785 [cancellation] of the Bank of North America's corporate charter. . . . . . . The push for the Constitution was based in part on the belief that state governments across the new nation had been too democratic and, as a result, had produced policies . . . that threatened elite interests. Most of the men who assembled at the Constitutional Convention in Philadelphia in 1787 were also convinced that the national government under the Articles of Confederation was too weak to counter the rising tide of democracy in the states." Terry Bouton, historian, Taming Democracy: "The People," the Founders, and the Troubled Ending of the American Revolution, 2007 One piece of evidence Bouton uses to support his argument about why some United States political leaders sought to replace the Articles of Confederation in 1787 was that they A thought that a strong central government threatened Americans' liberties B opposed the economic policies that some state legislatures pursued C believed that the national government should be more democratic D sought to discourage Europeans from investing in the United States

B opposed the economic policies that some state legislatures pursued In the first paragraph of the excerpt, Bouton provides evidence that some United States political leaders opposed the policies of the state legislatures in the 1780s because of state laws that, for example, limited land speculation and the creation of business corporations. Bouton uses this evidence to support his argument that elite economic interests motivated the pursuit of restrictions on the power of the states at the Constitutional Convention.

"[Before European contact] Cahokia [in present-day Missouri] and such other major centers as those now known as Coosa and Etowah in Georgia, Moundville in Alabama, and Natchez in Mississippi were home to highly stratified societies, organized as chiefdoms and characterized by a sharp divide between elites and commoners. . . . Surrounding networks of agricultural hamlets provided food to support the urban centers. . . . "From the Ohio River through most of present-day Canada and down the coast to the Chesapeake were speakers of Algonquian languages. . . . Nearly everywhere [here], villages composed of 500 to 2,000 people were the norm. . . . "[This] Indian country was decentralized and diverse, but not disconnected. . . . Routes of trade and communication, most of them millennia old and following the great river systems, crisscrossed the continent. The goods that moved along them were, for the most part, few and rare. . . . Some closely neighboring people might exchange crucial resources—corn, for instances, for meat or fish." Daniel K. Richter, historian, Facing East from Indian Country: A Native History of Early America, 2001 Which of the following best describes the economic system that supported the Native American villages discussed in the second paragraph of the excerpt? A Semi Nomadic hunting B Settled subsistence farming C Trade and manufacturing of luxury goods D Migration and colonization of new territories

B- Settled subsistence farming Villages and settlements of the size described in the excerpt typically relied upon economies that involved settled subsistence farming in which they produced their own food.

On the western side of the ocean, movements of people and ideas . . . preceded the Atlantic connection. Great empires—in the Valley of Mexico, on the Mississippi River . . . —had collapsed or declined in the centuries before 1492. . . . As Columbus embarked on his first transatlantic voyage, the Mexica, or Aztecs, were consolidating their position [in Mexico]; their city was a center of both trade and military might. Tenochtitlán [the Aztec capital] . . . held 200,000 people, a population greater than in the largest city in contemporary Europe. ". . . The Mississippian culture spread east and west from its center, the city of Cahokia, on the Mississippi River near the site of modern St. Louis. It was a successor to earlier cultures, evidence of which can be seen in the great ceremonial mounds they built. Cahokia declined and was ultimately abandoned completely in the later thirteenth century. . . . Throughout the Southeast, smaller mound-building centers continued." Karen Ordahl Kupperman, historian, The Atlantic in World History, 2012 Which of the following contributed most significantly to the population trend in pre-Columbian Mexico described in the excerpt? A Migration in pursuit of fertile lands B Trade and settlement resulting from maize cultivation C Low birth rates and high death tolls as a result of European diseases D Internal conflict between groups causing political instability

B- Trade and settlement resulting from maize cultivation Maize cultivation supported trade, settlement, advanced irrigation, and social diversification, all of which contributed to population growth and the development of a large urban center.

"The second chief and principal end [of colonization] . . . consists in the [sale] of the mass of our clothes and other commodities of England, and in receiving back of the needful commodities that we now receive from all other places of the world. . . . This one thing is to be done, without which it were in vain to go about this; and that is the matter of planting [colonies] and fortification. . . . We are to plant upon the mouths of the great navigable rivers which are there [in America], by strong order of fortification, and there to plant our colonies. . . . And these fortifications shall keep the [native] people of [America] in obedience and good order. . . . ". . . Without this planting in due time, we shall never be able to have full knowledge of the language, manners, and customs of the people of those regions. . . . And although by other means we might attain to the knowledge thereof, yet being not there fortified and strongly seated, the French that swarm with [a] multitude of people, or other nations, might secretly fortify and settle themselves before us." Richard Hakluyt, English government official, A Discourse on Western Planting, 1584 Which of the following developments in the 1500s is best illustrated by the excerpt? A European settlers faced resistance from Native Americans. B Europeans transported enslaved Africans to the Americas to produce sugar. C Europeans sought new sources of wealth in the Americas. D European missionaries traveled to the Americas seeking religious converts.

C Europeans sought new sources of wealth in the Americas. In his Discourse, Hakluyt argued that England should establish fortified colonies in the Americas in order to trade for valuable commercial commodities, which was one means by which Europeans sought new wealth from the Americas in the 1500s.

"The isolation of the [native peoples] of the Americas . . . from Old World germs prior to the last few hundred years was nearly absolute. Not only did very few people of any origin cross the great oceans, but those who did must have been healthy or they would have died on the way, taking their pathogens with them. . . . [Native Americans] were not without their own infections, of course. [But Native Americans] seem to have been without any experience with such Old World maladies as smallpox [and] measles. . . . "Indications of the susceptibility of [Native Americans] . . . to Old World infections appear almost immediately after the intrusion of the whites. In 1492, Columbus kidnapped a number of [Arawak Indians] to train as interpreters and to show to King Ferdinand and Queen Isabella. Several of them seem to have died on the stormy voyage to Europe [in 1493]. . . . In 1495, Columbus . . . sent 550 [Arawak] slaves . . . off across the Atlantic. . . to be put to work in Spain. The majority of these soon were also dead. . . . ". . . What killed the Arawaks in 1493 and 1495? . . . Columbus certainly did not want to kill his interpreters, and slavers and slaveholders have no interest whatever in the outright slaughter of their property. . . . The most likely candidates for the role of exterminator of the first [Native Americans] in Europe were those that killed so many other Arawaks in the decades immediately following: Old World pathogens." Alfred W. Crosby, historian, Ecological Imperialism: The Biological Expansion of Europe, 900-1900, published in 1986 Which of the following describes Crosby's overall argument in the excerpt about the reason for the change in Native American populations after 1492 ? A Slaveholders overworked enslaved Native Americans in silver and gold mines. B King Ferdinand and Queen Isabella commanded Native Americans to immigrate to Spain. C Native Americans had no immunity to new diseases introduced by Europeans. D Christopher Columbus pursued a policy of genocide against Native Americans.

C Native Americans had no immunity to new diseases introduced by Europeans Crosby argues in the excerpt that the introduction of new diseases to the Americas by Europeans, against which Native Americans had no immunity, led to a large reduction in Native American populations at the outset of European colonization.

"The isolation of the [native peoples] of the Americas . . . from Old World germs prior to the last few hundred years was nearly absolute. Not only did very few people of any origin cross the great oceans, but those who did must have been healthy or they would have died on the way, taking their pathogens with them. . . . [Native Americans] were not without their own infections, of course. [But Native Americans] seem to have been without any experience with such Old World maladies as smallpox [and] measles. . . . "Indications of the susceptibility of [Native Americans] . . . to Old World infections appear almost immediately after the intrusion of the whites. In 1492, Columbus kidnapped a number of [Arawak Indians] to train as interpreters and to show to King Ferdinand and Queen Isabella. Several of them seem to have died on the stormy voyage to Europe [in 1493]. . . . In 1495, Columbus . . . sent 550 [Arawak] slaves . . . off across the Atlantic. . . to be put to work in Spain. The majority of these soon were also dead. . . . ". . . What killed the Arawaks in 1493 and 1495? . . . Columbus certainly did not want to kill his interpreters, and slavers and slaveholders have no interest whatever in the outright slaughter of their property. . . . The most likely candidates for the role of exterminator of the first [Native Americans] in Europe were those that killed so many other Arawaks in the decades immediately following: Old World pathogens." Alfred W. Crosby, historian, Ecological Imperialism: The Biological Expansion of Europe, 900-1900, published in 1986 Which of the following best describes evidence used by Crosby to support his argument about the change in Native American populations after 1492 ? A The Spanish sought to use Native Americans as laborers on encomiendas. B Columbus trained Native Americans as translators and guides. C Native Americans who were taken to Europe as slaves experienced high mortality rates. D Some diseases affecting Native Americans originated in the Americas.

C Native Americans who were taken to Europe as slaves experienced high mortality rates. In the second paragraph, Crosby provides evidence that few Native Americans survived being transported as enslaved laborers to Europe by Columbus in 1492 and 1495 in order to support his argument that exposure to European diseases was the main factor causing the decline in Native American populations after 1492.

"Today, two hundred and fifty years after the French and Indian War, most Americans are no more familiar with its events and significance than they are with those of the Peloponnesian War. Few know that George Washington struck the first spark of a war that set the British North American frontier ablaze from the Carolinas to Nova Scotia, then spread to Europe, Canada, the Caribbean, West Africa, India, and, finally, the Philippines. Historians call this immense conflict the Seven Years' War; . . . Winston Churchill described it as 'the first world war.'" Fred Anderson, The War That Made America: A Short History of the French and Indian War, 2005 Britain's victory in the Seven Years' War (French and Indian War) had which of the following economic consequences for its American colonies? A The British government granted certain American colonists a monopoly on the fur trade without French competition. B The British government paid for the construction of canals to encourage trade between the Great Lakes region and the East Coast. C The British government increased taxation of colonial goods to help pay off the debt created by the war. D The British government forced American merchants to supply the British Royal Navy without payment.

C The British government increased taxation of colonial goods to help pay off the debt created by the war. Following the conclusion of the war, Great Britain increased its control and taxation of its colonies to help pay for the debt caused by the conflict.

"To understand political power right, and derive it from its original, we must consider what state all men are naturally in, and that is, a state of perfect freedom to order their actions, and dispose of their possessions and persons, as they think fit, within the bounds of the law of nature, without asking leave, or depending upon the will of any other man. "A state also of equality, wherein all the power and jurisdiction is reciprocal, no one having more than another. . . . "The state of nature has a law of nature to govern it which obliges every one . . . that being all equal and independent, no one ought to harm another in his life, health, liberty, or possessions." John Locke, Two Treatises of Government, 1689 The ideas in the excerpt were most likely interpreted by American colonists in the 1770s as a call to reject which of the following? A The holding of private property B The establishment of representative democracy C The acceptance of the divine right of kings D The creation of political parties

C The acceptance of the divine right of kings Locke's ideas in the excerpt that men are in a "state of freedom" and not dependent upon the will of others correspond to the rejection of the concept of the divine right of kings, the belief that rulers had a god-given authority to rule.

"Threatened by popular political victories [in the states] and widespread resistance, many elite Pennsylvanians launched an effort to remake the state and national governments so that they were less democratic. . . . Popular policies and resistance . . . threatened elite ideals. . . . Popular calls for a revaluation of war debt certificates, bans on for-profit corporations, progressive taxation, limits on land speculation, and every other measure designed to make property more equal promised to take wealth away from the elite. . . . It was also threatening that popular politics frightened off potential European investors. . . . [They] were alarmed by the Pennsylvania legislature's 1785 [cancellation] of the Bank of North America's corporate charter. . . . . . . The push for the Constitution was based in part on the belief that state governments across the new nation had been too democratic and, as a result, had produced policies . . . that threatened elite interests. Most of the men who assembled at the Constitutional Convention in Philadelphia in 1787 were also convinced that the national government under the Articles of Confederation was too weak to counter the rising tide of democracy in the states." Terry Bouton, historian, Taming Democracy: "The People," the Founders, and the Troubled Ending of the American Revolution, 2007 The ratification of the United States Constitution despite the "resistance" described in the excerpt is best understood in the context of which of the following developments? A The threat to the neutral trading rights of the United States B The persistence of regional cultures in the new United States C The agreement by some state delegates to pursue a Bill of Rights D The mistrust of centralized authority by some Anti-Federalist politicians

C The agreement by some state delegates to pursue a Bill of Rights While many Americans resisted the increased centralization of political power in the United States, the agreement among delegates at some state ratification conventions to pursue the addition of a Bill of Rights to the Constitution helped win majorities for ratification in these states.

"I have heard it asserted by some, that as America hath flourished under her former connection with Great Britain, that the same connection is necessary towards her future happiness, and will always have the same effect. Nothing can be more fallacious [untrue] than this kind of argument. We may as well assert that because a child has thrived upon milk, that it is never to have meat, or that the first twenty years of our lives is to become a precedent for the next twenty. But even this is admitting more than is true, for I answer roundly, that America would have flourished as much, and probably much more, had no European power had any thing to do with her. . . . "But Britain is the parent country, say some. Then the more shame upon her conduct. Even brutes do not devour their young, nor savages make war upon their families. . . . Europe, and not England, is the parent country of America. This new world hath been the asylum for the persecuted lovers of civil and religious liberty from every part of Europe. Hither have they fled, not from the tender embraces of the mother, but from the cruelty of the monster; and it is so far true of England, that the same tyranny which drove the first emigrants from home, pursues their descendants still." Thomas Paine, Common Sense, 1776 Paine's argument "that the same tyranny which drove the first emigrants from home, pursues their descendants still" was most likely in reference to which of the following situations? A The hope that religious dissenters would agree to join the Church of England B The challenges faced by settlers who moved west of the Appalachian Mountains C The belief among colonists that they had earned a right to greater liberty from Britain D The concern that increased debt following the Seven Years' War (French and Indian War) would decrease the value of American goods

C The belief among colonists that they had earned a right to greater liberty from Britain Paine's argument reflects increasing desire among colonists to assert their independence from British control and influence.

Hail Columbia! Happy Land! Hail ye heroes, heaven-born band, Who fought and bled in freedom's cause, Who fought and bled in freedom's cause, And when the storm of war was gone, Enjoy'd the peace your valor won— Let Independence be our boast, Ever mindful what it cost; Ever grateful for the prize, Let its altar reach the skies. Firm, united let us be, Rallying round our Liberty, As a band of brothers join'd, Peace and safety we shall find. . . . Sound, sound the trump of fame, Let [George] Washington's great name Ring round the world with loud applause, Ring round the world with loud applause, Let every clime to freedom dear, Listen with a joyful ear, With equal skill, with godlike power, He governs in the fearful hour Of horrid war, or guides with ease, The happier times of honest peace. Firm, united let us be, Rallying round our Liberty, As a band of brothers join'd, Peace and safety we shall find. Hail Columbia, popular song, 1798 Which of the following developments best explains the sentiment expressed in the first verse of the song lyrics? A The conflicts between the federal army and Native Americans on western frontiers B The resumption of commerce with Great Britain after the American Revolution C The growth among people in the United States of a sense of national identity D The partisan clash between Democratic-Republicans and Federalists

C The growth among people in the United States of a sense of national identity The celebration of the winning of independence and the fight for freedom during the Revolutionary War in the verse reflected a developing sense of American national identity in the United States in the 1790s.

"The New England settlers more closely resembled the non-migrating English population than they did other English colonists in the New World. . . . While the composition of the emigrant populations in the Chesapeake and the Caribbean hindered the successful transfer of familiar patterns of social relationships, the character of the New England colonial population ensured it. The prospect of colonizing distant lands stirred the imaginations of young people all over England but most of these young adults made their way to the tobacco and sugar plantations of the South. Nearly half of a sample of Virginia residents in 1625 were between the ages of twenty and twenty-nine, and groups of emigrants to the Chesapeake in the seventeenth century consistently included a majority of people in their twenties. In contrast, only a quarter of the New England settlers belonged to this age group. "Similarly, the sex ratio of the New England emigrant group resembled that of England's population. If women were . . . scarce in the Chesapeake . . . they were comparatively abundant in the northern colonies. In the second decade of Virginia's settlement, there were four or five men for each woman; by the end of the century, there were still about three men for every two women. Among the emigrants [in New England], however, nearly half were women and girls. Such a high proportion of females in the population assured the young men of New England greater success than their southern counterparts in finding spouses." Virginia DeJohn Anderson, historian, "Migrants and Motives: Religion and the Settlement of New England, 1630-1640," published in 1985 The second paragraph of the excerpt makes which of the following claims about the populations of men and women in the colonies? A New England settlers included more young men than Chesapeake settlers did. B Emigrants to New England were predominantly unmarried women. C The populations of men and women in New England were roughly equal from the time of its founding. D The population of the Chesapeake achieved a balanced sex ratio more quickly than New England did.

C The populations of men and women in New England were roughly equal from the time of its founding. The excerpt claims that the sex ratio of New England's emigrant population resembled England's and nearly half of the settlers in New England were female, unlike the Chesapeake where men outnumbered women throughout the seventeenth century.

"It was painful for me, on a subject of such national importance, to differ from the respectable members who signed the Constitution; but conceiving, as I did, that the liberties of America were not secured by the system, it was my duty to oppose it. "My principal objections to the plan are, that there is no adequate provision for a representation of the people; . . . that some of the powers of the legislature are ambiguous . . . ; that the executive is blended with, and will have an undue influence over, the legislature; that the judicial department will be oppressive; . . . and that the system is without the security of a bill of rights. These are objections which are not local, but apply equally to all the states. "As the Convention was called for the 'sole and express purpose of revising the Articles of Confederation . . . ,' I did not conceive that these powers extend to the formation of the plan proposed; but the Convention being of a different opinion, I acquiesced [agreed] in it, being fully convinced that, to preserve the Union, an efficient government was indispensably necessary, and that it would be difficult to make proper amendments to the Articles of Confederation. "The Constitution proposed has few, if any, federal features, but is rather a system of national government. Nevertheless, in many respects, I think it has great merit, and, by proper amendments, may be adapted. . . . "Others may suppose that the Constitution may be safely adopted, because therein provision is made to amend it. But cannot this object be better attained before a ratification than after it? And should a free people adopt a form of government under conviction that it wants [needs] amendment?" Elbridge Gerry, letter to the Massachusetts state legislature, 1787 Gerry made which of the following arguments in the excerpt about the Articles of Confederation? A The Articles needed to be amended to make slavery illegal. B The Articles were mostly effective at funding the central government. C The problems of the Articles could not be fixed by the state delegates. D The state governments should be given increased power under the Articles.

C The problems of the Articles could not be fixed by the state delegates. In the excerpt Gerry conceded that although he was not sent to the Constitutional Convention with permission to abandon the Articles of Confederation entirely, the creation of a new system of government would more effectively provide for a capable central government than simply amending the existing Articles.

"In the time of the late war, being desirous to defend, secure, and promote the Rights and Liberties of the people, we spared no pains but freely granted all the aid and assistance of every kind that our civil fathers [political leaders] required of us. "We are sensible also that a great debt is justly brought upon us by the War, and we are as willing to pay our share towards it as we are to enjoy our shares in independency. . . . "But with the greatest submission we beg leave to inform your Honors that unless something takes place more favorable to the people, in a little time at least one half of our inhabitants in our opinion will become bankrupt. . . . When we compute the taxes laid upon us the five preceding years, the State and County, town, and class taxes, the amount is equal to what our farms will rent for. Sirs in this situation, what have we to live on: No money to be had; our estates daily posted and sold. . . . Surely your Honors are no strangers to the distresses of the people but do know that many of our good inhabitants are now confined in jail for debt and for taxes. . . . Will not the people in the neighboring states say of this state: although the Massachusetts [people] boast of their fine Constitution, their government is such that it devours their inhabitants? ". . . If your Honors find anything above mentioned worthy of notice, we earnestly pray that . . . [the state legislature] would point out some way whereby the people might be relieved." Petition from the town of Greenwich to the Massachusetts state legislature, 1786 The concern in the petition about the effect of taxation is best understood in continuity with which of the following earlier developments? A The increased competition between the British and French in North America B The passage of gradual emancipation laws in some northern states C The revolt against British imperial control of the North American colonies D The development of opposition to the United States Constitution

C The revolt against British imperial control of the North American colonies The concerns about unfair and ruinous taxation on the part of common American farmers after the American Revolution reflected the continued unpopularity of taxation by political leaders that began during the imperial crisis of the 1760s.

The expansion of European settlement in the Americas most directly led to which of the following developments?

C The use of enslaved Native Americans and Africans to meet the labor demands of colonial agricultural production As more European settlements were established, the demand in agricultural production required more laborers, resulting in the enslavement of some Native Americans and the importation of enslaved people from Africa by both the Spanish and the British.

"His Catholic Majesty [of Spain] and the United States of America desiring to consolidate on a permanent basis the Friendship and good correspondence which happily prevails between the two Parties, have determined to establish by a convention several points. . . . "Article IV "It is likewise agreed that the Western boundary of the United States which separates them from the Spanish Colony of Louisiana, is in the middle of the channel or bed of the River Mississippi . . . ; and his Catholic Majesty has likewise agreed that the navigation of the said River in its whole breadth from its source to the Ocean shall be free only to his Subjects, and the Citizens of the United States, unless he should extend this privilege to the Subjects of other Powers by special convention. . . . "Article XXII "The two high contracting Parties hoping that the good correspondence and friendship which happily reigns between them will be further increased by this Treaty, and that it will contribute to augment their prosperity and opulence, will in future give to their mutual commerce all the extension and favor which the advantage of both Countries may require; . . . his Catholic Majesty will permit the Citizens of the United States for the space of three years from this time to deposit their merchandise and effects in the Port of New Orleans." Treaty of Friendship, Limits, and Navigation Between Spain and the United States, known as Pinckney's Treaty or the Treaty of San Lorenzo, 1795 Which of the following was a primary reason that the United States and Spain agreed to the articles outlined in the excerpt? A To force Great Britain to give up its colonial claims in Canada B To provide land for the establishment of Catholic missions C To minimize tensions caused by United States expansion into western territory D To prevent France from reclaiming the territory it lost after the Seven Years' War

C To minimize tensions caused by United States expansion into western territory As United States settlement continued to expand westward toward the Mississippi River, Spain and the United States sought to address concerns that debates over American navigation rights would lead to future conflicts. White farmers in the Northwest Territory at this time partly depended on access to the Mississippi River and New Orleans, via tributaries such as the Ohio River, to sell their commercial produce.

"'I ordered my company to fire,' [George] Washington reported. . . . This incident . . . led to massive French retaliation and the outbreak of what was soon a world war. It raged in North America for six years, 1754-60, in Central and South America, in the Caribbean and the Atlantic, in India and the East, and not least in Europe, where it was known as the Seven Years War (1756-63). . . . Horace Walpole [stated]: 'The volley fired by a young Virginian in the backwoods of America set the world on fire.'" Paul Johnson, historian, A History of the American People, 1997 Britain attempted to pay for the debt resulting from the Seven Years' War (French and Indian War) by A charging immigrants a fee to relocate to North America B selling ships from its navy to colonial merchants C increasing taxes on goods bought and sold in the colonies D sponsoring expeditions to locate valuable resources

C increasing taxes on goods bought and sold in the colonies As a result of the cost of the war, Great Britain enacted a series of taxes—particularly on its colonies—and began to enforce many rules that had been generally ignored.

"The emancipation of slaves in New England, beginning around 1780, was a gradual process, whether by post nati statute [laws freeing enslaved people born after a certain date], as in Rhode Island and Connecticut, or by effect, as in Massachusetts and New Hampshire, where ambiguous judicial decisions and constitutional interpretations discouraged slaveholding without clearly outlawing it. The gradual nature of the process encouraged Whites to transfer a language and set of practices shaped in the context of slavery to their relations with a slowly emerging population of free people of color. The rhetoric of antislavery and revolutionary republicanism fostered this transfer, undergirding Whites' assumptions that emancipated slaves, likely to be dependent and disorderly, would constitute a problem requiring firm management in the new republic. . . . "Even more problematic was the promise implicit in antislavery rhetoric that abolition, by ending 'the problem'—the sin of slavery and the troublesome presence of slaves—would result in the eventual absence of people of color themselves. In other words, Whites anticipated that free people of color, would, by some undefined moment (always imminent), have disappeared." Joanne Pope Melish, historian, Disowning Slavery: Gradual Emancipation and "Race" in New England, 1780-1860, published in 1998 The author argues that emancipation in northern states occurred A as a result of Supreme Court rulings B through amendments to state constitutions C over a long period of time D in response to widespread protests

C over a long period of time The author argues that emancipation was a "gradual" process that occurred in different ways in different states at different points in time.

"The development of a plantation economy, beginning in the sixteenth century, transformed Africa, America, Europe, and Asia, too. It displaced the old silk trade and shifted the increasingly dynamic center of the world economy westward to the Atlantic. . . . "The Atlantic economy supplied eager European consumers with mildly addictive . . . crops like tobacco and coffee, along with sugar. . . . The Atlantic plantation system transformed these three [products] into items of general consumption. . . . Investors prospered, and capital for further economic development accumulated in the [home country]. The governments found funding and motive to develop sea power. The Americas had lucrative export crops and developed a society based on a system of labor exploitation of Africans, and Africa suffered the transport of eleven million of its people to the New World." Thomas Bender, historian, A Nation Among Nations: America's Place in World History, 2006 The second paragraph of the excerpt makes which of the following claims about the introduction to Europe of new crops from the Americas? A They remained relatively unpopular. B They required little labor to produce. C They stimulated economies across Europe. D They replaced traditional agricultural products.

C- They stimulated economies across Europe. The excerpt explains that the trade in goods produced in the Americas such as tobacco, coffee, and sugar contributed to economic prosperity in European countries engaged in colonialism.

"The isolation of the [native peoples] of the Americas . . . from Old World germs prior to the last few hundred years was nearly absolute. Not only did very few people of any origin cross the great oceans, but those who did must have been healthy or they would have died on the way, taking their pathogens with them. . . . [Native Americans] were not without their own infections, of course. [But Native Americans] seem to have been without any experience with such Old World maladies as smallpox [and] measles. . . . "Indications of the susceptibility of [Native Americans] . . . to Old World infections appear almost immediately after the intrusion of the whites. In 1492, Columbus kidnapped a number of [Arawak Indians] to train as interpreters and to show to King Ferdinand and Queen Isabella. Several of them seem to have died on the stormy voyage to Europe [in 1493]. . . . In 1495, Columbus . . . sent 550 [Arawak] slaves . . . off across the Atlantic. . . to be put to work in Spain. The majority of these soon were also dead. . . . ". . . What killed the Arawaks in 1493 and 1495? . . . Columbus certainly did not want to kill his interpreters, and slavers and slaveholders have no interest whatever in the outright slaughter of their property. . . . The most likely candidates for the role of exterminator of the first [Native Americans] in Europe were those that killed so many other Arawaks in the decades immediately following: Old World pathogens." Alfred W. Crosby, historian, Ecological Imperialism: The Biological Expansion of Europe, 900-1900, published in 1986 Which of the following developments in the late 1400s and early 1500s is depicted in the excerpt? A Native Americans adapted to diverse geographical environments and developed complex societies. B Alliances with Europeans aided some Native American societies in their efforts to conquer rival powers. C Europeans persecuted for their religious beliefs established new separatist settlements in the Americas. D Europeans undertook voyages across the Atlantic to the Americas in search of new sources of wealth.

D Europeans undertook voyages across the Atlantic to the Americas in search of new sources of wealth. The encounters between the Arawak Indians and Christopher Columbus described in the excerpt resulted from voyages made by Columbus and other Europeans seeking westerly access to eastern luxury goods or sources of material wealth in the Americas beginning in 1492.

"It was painful for me, on a subject of such national importance, to differ from the respectable members who signed the Constitution; but conceiving, as I did, that the liberties of America were not secured by the system, it was my duty to oppose it. "My principal objections to the plan are, that there is no adequate provision for a representation of the people; . . . that some of the powers of the legislature are ambiguous . . . ; that the executive is blended with, and will have an undue influence over, the legislature; that the judicial department will be oppressive; . . . and that the system is without the security of a bill of rights. These are objections which are not local, but apply equally to all the states. "As the Convention was called for the 'sole and express purpose of revising the Articles of Confederation . . . ,' I did not conceive that these powers extend to the formation of the plan proposed; but the Convention being of a different opinion, I acquiesced [agreed] in it, being fully convinced that, to preserve the Union, an efficient government was indispensably necessary, and that it would be difficult to make proper amendments to the Articles of Confederation. "The Constitution proposed has few, if any, federal features, but is rather a system of national government. Nevertheless, in many respects, I think it has great merit, and, by proper amendments, may be adapted. . . . "Others may suppose that the Constitution may be safely adopted, because therein provision is made to amend it. But cannot this object be better attained before a ratification than after it? And should a free people adopt a form of government under conviction that it wants [needs] amendment?" Elbridge Gerry, letter to the Massachusetts state legislature, 1787 In the excerpt, Gerry claimed that he did not sign the United States Constitution for which of the following reasons? A He required approval from the Massachusetts legislature. B He demanded that he be appointed to the new government. C He believed that the United States should return to British rule. D He believed the new government insufficiently protected the rights of citizens.

D He believed the new government insufficiently protected the rights of citizens. Gerry noted at the beginning of the excerpt that the primary reason for his refusal to sign the new Constitution was that he believed "the liberties of America were not secured by the system."

Hail Columbia! Happy Land! Hail ye heroes, heaven-born band, Who fought and bled in freedom's cause, Who fought and bled in freedom's cause, And when the storm of war was gone, Enjoy'd the peace your valor won— Let Independence be our boast, Ever mindful what it cost; Ever grateful for the prize, Let its altar reach the skies. Firm, united let us be, Rallying round our Liberty, As a band of brothers join'd, Peace and safety we shall find. . . . Sound, sound the trump of fame, Let [George] Washington's great name Ring round the world with loud applause, Ring round the world with loud applause, Let every clime to freedom dear, Listen with a joyful ear, With equal skill, with godlike power, He governs in the fearful hour Of horrid war, or guides with ease, The happier times of honest peace. Firm, united let us be, Rallying round our Liberty, As a band of brothers join'd, Peace and safety we shall find. Hail Columbia, popular song, 1798 Which of the following best explains the depiction of George Washington in the third verse of the song? A Washington's public advocacy for abolition B The movement to elect Washington president for life C Washington's policy of intervening in European conflicts D The popular image in national culture of Washington as a virtuous leader

D The popular image in national culture of Washington as a virtuous leader In the early United States, Washington became a celebrated national figure for his leadership during the Revolutionary War and for embodying republican values.

"In the time of the late war, being desirous to defend, secure, and promote the Rights and Liberties of the people, we spared no pains but freely granted all the aid and assistance of every kind that our civil fathers [political leaders] required of us. "We are sensible also that a great debt is justly brought upon us by the War, and we are as willing to pay our share towards it as we are to enjoy our shares in independency. . . . "But with the greatest submission we beg leave to inform your Honors that unless something takes place more favorable to the people, in a little time at least one half of our inhabitants in our opinion will become bankrupt. . . . When we compute the taxes laid upon us the five preceding years, the State and County, town, and class taxes, the amount is equal to what our farms will rent for. Sirs in this situation, what have we to live on: No money to be had; our estates daily posted and sold. . . . Surely your Honors are no strangers to the distresses of the people but do know that many of our good inhabitants are now confined in jail for debt and for taxes. . . . Will not the people in the neighboring states say of this state: although the Massachusetts [people] boast of their fine Constitution, their government is such that it devours their inhabitants? ". . . If your Honors find anything above mentioned worthy of notice, we earnestly pray that . . . [the state legislature] would point out some way whereby the people might be relieved." Petition from the town of Greenwich to the Massachusetts state legislature, 1786 Which of the following claims did the residents of Greenwich use to most support their argument that they should be "relieved" by the Massachusetts legislature from the situation described in the petition? A They believed that Massachusetts should pay off its war debts. B They earned enough money to pay their state taxes. C They were eager to gain the benefits of independence. D They had aided the government during the Revolutionary War.

D They had aided the government during the Revolutionary War. In the first paragraph, the claim that the people of Greenwich had aided the cause of the American Revolution was used to support the overall argument in the petition that they deserved to be relieved by the Massachusetts legislature from debt and burdensome taxes.

"I . . . write an account to Your Majesty as the first [person] to come among these natives. . . . "These Indian people of New Spain [Mexico] are vassals of Your Majesty. . . . I dare plead with you for a remedy because, for their people to be saved, they are in great need of relief in order to devote themselves at least somewhat to matters of Faith. After all, it is the struggle for their salvation that justifies their discovery. . . . "I firmly believe that if the decrees Your Majesty sent here for their benefit were implemented, and if the governors and judges did more than pretend to do so, great good would have come to these people. Even more firmly I believe that Your Majesty's intention is that they be saved and that they know God. For this to happen, they must have some relief, so that with the moderate labor needed to meet their tribute obligation, they can still give themselves wholeheartedly to our teachings. . . . Otherwise, God will have good reason to complain, for Spaniards came to this land and have taken their property for their own benefit, and Your Majesty has extracted great benefit from them, too. . . . ". . . Your Majesty . . . should know that the Indians who are required to labor for a master in Mexico City in domestic service and bring firewood, fodder, and chickens leave their pueblo for a month at a time. . . . And the poor Indians often have to buy these things because they are not to be found in their pueblos. . . . Take pity on them and consider what is happening to the poor Indian woman who is in her house with no one to support her and her children, for her husband is hard pressed to meet his tribute requirement. . . . ". . . I advise you that if Your Majesty does not establish that . . . [the Indians] be required to pay tribute only from what they have, within thirty years these parts will be as deserted as the [Caribbean] islands, and so many souls will be lost." Fray (Friar) Pedro de Gante, Spanish Catholic friar and missionary, letter to Emperor Charles V, king of Spain, 1552 One piece of evidence that de Gante used in the excerpt to support his overall argument about the treatment of the people of Mexico is that Native Americans A were not subjects of the Spanish crown B were first encountered by de Gante himself C had sufficient time to learn about Christianity D did not have enough supplies to support their families

D did not have enough supplies to support their families In order to argue that the Native Americans were ill-treated by the Spanish, de Gante provided evidence that Native Americans were unable to support their families while seeking tribute for the Spanish.

"It was painful for me, on a subject of such national importance, to differ from the respectable members who signed the Constitution; but conceiving, as I did, that the liberties of America were not secured by the system, it was my duty to oppose it. "My principal objections to the plan are, that there is no adequate provision for a representation of the people; . . . that some of the powers of the legislature are ambiguous . . . ; that the executive is blended with, and will have an undue influence over, the legislature; that the judicial department will be oppressive; . . . and that the system is without the security of a bill of rights. These are objections which are not local, but apply equally to all the states. "As the Convention was called for the 'sole and express purpose of revising the Articles of Confederation . . . ,' I did not conceive that these powers extend to the formation of the plan proposed; but the Convention being of a different opinion, I acquiesced [agreed] in it, being fully convinced that, to preserve the Union, an efficient government was indispensably necessary, and that it would be difficult to make proper amendments to the Articles of Confederation. "The Constitution proposed has few, if any, federal features, but is rather a system of national government. Nevertheless, in many respects, I think it has great merit, and, by proper amendments, may be adapted. . . . "Others may suppose that the Constitution may be safely adopted, because therein provision is made to amend it. But cannot this object be better attained before a ratification than after it? And should a free people adopt a form of government under conviction that it wants [needs] amendment?" Elbridge Gerry, letter to the Massachusetts state legislature, 1787 Arguments for creating a stronger federal government arose primarily as the result of long-term concerns about the A attempts by Great Britain to regain its former North American colonies B use of congressional funds to build road networks between the states C challenge of establishing rules to allow territories to become new states D inability of the national government to maintain order under the Articles

D inability of the national government to maintain order under the Articles Internal conflicts throughout the 1780s such as Shays' Rebellion led many American leaders to believe that the federal government needed to be strengthened by replacing the Articles of Confederation.

"'I ordered my company to fire,' [George] Washington reported. . . . This incident . . . led to massive French retaliation and the outbreak of what was soon a world war. It raged in North America for six years, 1754-60, in Central and South America, in the Caribbean and the Atlantic, in India and the East, and not least in Europe, where it was known as the Seven Years War (1756-63). . . . Horace Walpole [stated]: 'The volley fired by a young Virginian in the backwoods of America set the world on fire.'" Paul Johnson, historian, A History of the American People, 1997 The British government attempted to restrict westward settlement following the Seven Years' War (French and Indian War) in order to A support the fur trade east of the Appalachian Mountains B discourage immigration to the North American colonies C give French settlers time to leave Canada D minimize conflicts with Native Americans

D minimize conflicts with Native Americans The Proclamation of 1763 was enacted with the intended purpose of limiting the westward movement of colonists so that the British government could spend less money and effort on securing them from conflicts with Native Americans whose land they were claiming. Related Content & Skills

"The development of a plantation economy, beginning in the sixteenth century, transformed Africa, America, Europe, and Asia, too. It displaced the old silk trade and shifted the increasingly dynamic center of the world economy westward to the Atlantic. . . . "The Atlantic economy supplied eager European consumers with mildly addictive . . . crops like tobacco and coffee, along with sugar. . . . The Atlantic plantation system transformed these three [products] into items of general consumption. . . . Investors prospered, and capital for further economic development accumulated in the [home country]. The governments found funding and motive to develop sea power. The Americas had lucrative export crops and developed a society based on a system of labor exploitation of Africans, and Africa suffered the transport of eleven million of its people to the New World." Thomas Bender, historian, A Nation Among Nations: America's Place in World History, 2006 Which of the following claims does the excerpt make about changes that occurred as a result of new interactions in the Atlantic region? A Merchants from Asia dominated trade throughout the sixteenth century. B Europeans developed new methods of conducting trade and making profits. C African kingdoms were the largest purchasers of goods produced in the Americas. D Native Americans amassed fortunes as Europeans paid high prices for rare goods.

Europeans developed new methods of conducting trade and making profits. The excerpt claims that financial investors prospered and accumulated capital, which would eventually become the basis for further economic development and, in the long term, industrialization.

Which of the following was a major difference between the encomienda system and slave labor in the Spanish colonies?

The encomienda system exploited the labor of Native Americans, whereas slavery more typically extracted labor from enslaved Africans. The encomienda system was developed as a way of marshaling Native American labor to support agriculture and mining in the Spanish colonies, whereas the use of slave labor primarily resulted from Spanish trade with West Africa. Native Americans were a minority of the slave population.

"The development of a plantation economy, beginning in the sixteenth century, transformed Africa, America, Europe, and Asia, too. It displaced the old silk trade and shifted the increasingly dynamic center of the world economy westward to the Atlantic. . . . "The Atlantic economy supplied eager European consumers with mildly addictive . . . crops like tobacco and coffee, along with sugar. . . . The Atlantic plantation system transformed these three [products] into items of general consumption. . . . Investors prospered, and capital for further economic development accumulated in the [home country]. The governments found funding and motive to develop sea power. The Americas had lucrative export crops and developed a society based on a system of labor exploitation of Africans, and Africa suffered the transport of eleven million of its people to the New World." Thomas Bender, historian, A Nation Among Nations: America's Place in World History, 2006 The excerpt makes the overall argument that the Atlantic economy A eliminated competition between European nations B drove long-lasting economic shifts across Europe, Africa, and the Americas C contributed to the persistence of feudal economic systems D contributed to the collapse of European empires

drove long-lasting economic shifts across Europe, Africa, and the Americas The excerpt outlines how the growth in the Atlantic economy resulted in changes in every region—whether positive in Europe as a result of increased wealth, or negative in Africa as a result of the growth of the trans-Atlantic slave trade.


Set pelajaran terkait

13-4 Paying Withholding and Payroll Taxes

View Set

Logistics Test 3: Network Design

View Set